Select Financial Statement Accounts Flashcards
Hilltop Co.’s monthly bank statement shows a balance of $54,200. Reconciliation of the statement with company books reveals the following information:
Bank service charge $ 10
Insufficient funds check 650
Checks outstanding 1,500
Deposits in transit 350
Check deposited by Hilltop and cleared by the bank for $125, but improperly recorded by Hilltop as $152
What is the net cash balance after the reconciliation?
A $52,363 B $53,023 C $53,050 D $53,077
Explanation:
The correct answer is (C).
A common format of the bank reconciliation statement is to reconcile both book and bank balances to a common amount known as the true balance or net cash balance. This approach has the advantage of providing the cash figure to be reported in the balance sheet. Furthermore, journal entries necessary to adjust the books can be taken directly from the book balance section of the reconciliation. Net cash is the bank balance adjusted for outstanding checks and deposits in transit ($54,200 + $350 - $1,500 = $53,050).
Net cash is also the book balance adjusted for unrecorded or misrecorded items, such as service charges, insufficient funds, and errors. A normal book to bank reconciliation will compute the unadjusted book balance of $53,737. Net cash from the book side is $53,050 ($53,737 - $10 service charge - $650 insufficient funds - $27 net effect from the error).
Options (A), (B) and (D) are incorrect based on the above explanation.
Poe, Inc., had the following bank reconciliation at March 31: Balance per bank statement, 3/31 $ 46,500 Add deposit in transit 10,300 $ 56,800 Less outstanding checks (12,600) Balance per books, 3/31 $ 44,200 Data per bank for the month of April: Deposits $ 58,400 Disbursements 49,700All reconciling items at March 31 cleared the bank in April. Outstanding checks at April 30 totaled $7,000. There were no deposits in transit at April 30. What is the amount of cash disbursements per books in April 30? A $44,100 B $49,200 C $54,300 D $56,700
Answer is A
Explanation:
Disbursements per bank, April $49,700
Less: Outstanding checks at 3/31 (12,600)
Add Outstanding checks at 4/30 7,000
Cash disbursement per books, April $44,100
At June 30, Almond Co.'s cash balance was $10,012 before adjustments, while its ending bank statement balance was $10,772. Check number 101 was issued June 2 in the amount of $95, but was erroneously recorded in Almond's general ledger balance as $59. The check was correctly listed in the bank statement at $95. The bank statement also included a credit memo for interest earned in the amount of $35, and a debit memo for monthly service charges in the amount of $50. What was Almond's adjusted cash balance at June 30? A $9,598 B $9,961 C $10,048 D $10,462
Answer is B
Explanation:
When the check was originally recorded the journal entry was:
dr. Some Account……………………………….59
cr. Cash…………………………………………………59
The correcting entry would require an additional $36 credit to cash to get to the correct amount that should have been recorded in the first transaction ($95).
dr. Some Account……………………………….36
cr. Cash………………………………………………..36
Therefore, you should subtract $36 from the Cash balance reported in the general ledger.
=10012-36+35-50
=9,961
Trans Co. had the following balances at December 31, year 4: Cash in checking account $35,000 Cash in money market account 75,000 U. S. Treasury bill, purchased 11/1 year 4, maturing 1/31, year 5 350,000 U. S. Treasury bill, purchased 12/1 year 4, maturing 3/31, year 5 400,000Trans's policy is to treat as cash equivalents all highly-liquid investments with a maturity of three months or less when purchased. What amount should Trans report as cash and cash equivalents in its December 31, year 4, balance sheet? A $110,000 B $385,000 C $460,000 D $860,000
Answer is C
Explanation:
The $400,000 U.S. Treasury bill purchased 12/1, year 4 and maturing 3/31, year 5 is not included as a cash equivalent because the maturity was more than three months at the time of purchase. Cash and cash equivalents reported at December 31, year 4, are as follows:
Cash in checking account $ 35,000
Cash in money market account 75,000
U. S. Treasury bill, purchased 11/1 year 4, maturing 1/31, year 5 350,000
Total cash and cash equivalents $ 460,000
________________________ is (are) defined as money or a claim to receive a sum of money, the amount of which is fixed or determinable without reference to future prices of specific goods or services.
A Monetary assets B Monetary liabilities C The value in use D The recoverable amount
Answer is A
Explanation:
Monetary assets are defined as money or a claim to receive a sum of money, the amount of which is fixed or determinable without reference to future prices of specific goods or services.
Alton Co. had a cash balance of $32,300 recorded in its general ledger at the end of the month, prior to receiving its bank statement. Reconciliation of the bank statement reveals the following information:
Bank service charge $15
Check deposited and returned for insufficient funds check $120
Deposit recorded in the general ledger as $258 but should be $285
Checks outstanding $1,800
After reconciling its bank statement, what amount should Alton report as its cash account balance?
A $30,338 B $30,392 C $32,138 D $32,192
Explanation:
The correct answer is (D).
There are two types of reconciling items.
Type A reconciling items do not require adjustment on books. These include items that have already been recorded on books and require correction from the bank’s side. Outstanding checks, deposits in transit and bank errors are type A items.
Type B adjustment entries are those entries which have been recorded correctly by the bank but have not been correctly recorded in the books of accounts and thus require adjustment on books. The items that are classified as type B reconciling items that require adjustment on books are unrecorded returned insufficient fund checks, unrecorded bank reconciliation of notes receivables, unrecorded bank charges and cash account errors.
Cash Balance $32,300 (-)Bank Fees ($15) (-) NSF check ($120) (+) Deposit Error ($285 -$258) $27 Corrected cash balance $32,192
he following are held by Smite Co.: Cash in checking account $20,000 Cash in bond sinking fund account 30,000 Post-dated check from customer dated one month from balance sheet date 250 Petty cash 200 Commercial paper (matures in two months) 7,000 Certificate of deposit (matures in six months) 5,000What amount should be reported as cash and cash equivalents on Smite's balance sheet? A $57,200 B $32,200 C $27,450 D $27,200
Answer is D
Explanation:
Items in this question that are not considered cash equivalents are the cash in the bond sinking fund account ($30,000), the postdated check from a customer ($250), and the certificate of deposit that matures in six months ($5,000).
$20,000 Cash in checking account
200 Petty cash
7,000 Commercial paper (matures in two months)
$27,200 Total cash and cash equivalents
The following information pertains to Grey Co. at December 31 of the previous year: Checkbook balance $ 12,000 Bank statement balance 16,000 Check drawn on Grey's account, payable to a vendor, dated and recorded last 12/31 but not mailed until 1/10 this year 1,800 On Grey's December 31, previous year balance sheet, what amount should be reported as cash? A $12,000 B $13,800 C $14,200 D $16,000
Answer is B
Explanation: Checkbook balance $ 12,000 Unmailed checks 1,800 Cash $ 13,800 Answers (a), (c), and (d) are incorrect because the $1,800 check was not mailed as of 12/31 and, therefore, needs to be added back to the checkbook balance to arrive at the true cash balance as of 12/31. In addition, the starting point is the checkbook, not the bank statement, because the bank statement does not include checks in transit.
Poe, Inc., had the following bank reconciliation at March 31 of the current year:
Balance per bank statement, 3/31 $ 46,500
Add deposit in transit 10,300
$ 56,800
Less outstanding checks (12,600)
Balance per books, 3/31 $ 44,200
Data per bank for the month of April:
Deposits $ 58,400
Disbursements 49,700
All reconciling items at March 31 cleared the bank in April. Outstanding checks at April 30 totaled $7,000. There were no deposits in transit at April 30.
What is the cash balance per books at April 30?
A $48,200 B $52,900 C $55,200 D $58,500
Explanation:
The correct answer is (A).
The cash balance per books at April 30 is computed by subtracting the outstanding checks at April 30 from the balance per bank. First, adjust the bank balance from March 31 to April 30:
Cash balance per 3/31 bank statement $ 46,500
Add April deposits per bank 58,400
Less April disbursements per bank (49,700)
Cash balance per bank, 4/30 (all reconciling items at 3/31 cleared the bank in April) 55,200
Less outstanding checks, 4/30 (7,000)
Cash balance per books, 4/30 $ 48,200
Option (B) is incorrect because this takes the closing balance as per books as of 3/31 and adds the deposits and deducts disbursements for the month of April without taking into consideration the other impacts ($44,200 + $58,400 - $49,700 = $52,900).
Option (C) is incorrect because this does not take into account the outstanding checks on April 30 of $7,000.
Option (D) is incorrect because of inaccurate calculations
In preparing its August 31, Year 2 bank reconciliation, Apex Corp. has available the following information: Balance per bank statement, 8/31 $18,050 Deposit in transit, 8/31 3,250 Return of customer's check for insufficient funds, 8/31 600 Outstanding checks, 8/31 2,750 Bank service charges for August 100At August 31, Year 2, Apex's correct cash balance is A $18,550 B $17,950 C $17,850 D $17,550
Answer is A
Explanation: Bank balance ($18,050) + DIT ($3,250) - Outstanding checks ($2,750) = Correct Cash Balance of $18,550 [Editor's note: returned check and service charges are already reflected in the 8/31 bank balance.]
Smith Co. has a checking account at Small Bank and an interest-bearing savings account at Big Bank. On December 31, year 1, the bank reconciliations for Smith are as follows: Big Bank Bank balance $150,000 Deposit in transit 5,000 Book balance $155,000 Small Bank Bank balance $ 1,500 Outstanding checks (8,500) Book balance $(7,000)What amount should be classified as cash on Smith's balance sheet at December 31, year 1? A $148,000 B $151,000 C $155,000 D $156,000
Answer is C
Explanation:
Smith would classify the $150,000 bank balance and $5,000 deposit in transit for Big Bank as cash on the balance sheet. The bank balance of $1,500 in Small Bank is negated by the $8,500 in outstanding checks. Overdrafts in accounts with no available cash in another account at the same bank to offset are classified as current liabilities. They are not deducted from the total amount of cash at another bank.
On March 31, Vale Co. had an unadjusted credit balance of $1,000 in its allowance for uncollectible accounts. An analysis of Vale's trade accounts receivable at that date revealed the following: Age Amount Estimated uncollectible 0 - 30 days $60,000 5% 31- 60 days 4,000 10% Over 60 days 2,000 $1,400What amount should Vale report as allowance for uncollectible accounts in its March 31 balance sheet? A $4,800 B $4,000 C $3,800 D $3,000
Answer is A
Explanation:
While the unadjusted credit balance in the allowance accounts would be used to compute the uncollectible account expense for the period ending 3/31, the amount is not used to compute the balance of the allowance for uncollectible accounts at 3/31.
Age Accounts receivable Estimated % uncollectible Allowance for uncollectible accounts
0 - 30 days $60,000 5% $3,000
31 - 60 days 4,000 10% 400
Over 60 days 2,000 * 1,400
$4,800*The dollar amount of the estimated uncollectible accounts over 60 days is given in the data.
When the allowance method of recognizing uncollectible accounts is used, the entry to record the write-off of a specific account
A
Decreases both accounts receivable and the allowance for uncollectible accounts.
B
Decreases accounts receivable and increases the allowance for uncollectible accounts.
C
Increases the allowance for uncollectible accounts and decreases net income.
D
Decreases both accounts receivable and net income.
Answer is A
Explanation:
The entry to record the write-off of a specific account receivable using the allowance method involves a debit to allowance for uncollectible accounts and a credit to accounts receivable. This affects balance sheet accounts, not net income
On June 1 of the current year, Pitt Corp. sold merchandise with a list price of $5,000 to Burr on account. Pitt allowed trade discounts of 30% and 20%. Credit terms were 2/15, n/40 and the sale was made FOB shipping point. Pitt prepaid $200 of delivery costs for Burr as an accommodation. On June 12, Pitt received from Burr a remittance in full payment amounting to A $2,744 B $2,940 C $2,944 D $3,140
Answer is C
Explanation:
List price of merchandise $ 5,000
Less: trade discount—30% (1,500)
Balance $ 3,500
Less: trade discount—20% (700)
Balance $ 2,800
Less: 2% cash discount, remittance received within 15 days of sale ($2,800 × 2%) (56)
Amount received for merchandise $ 2,744
Add: Reimbursement of prepayment of delivery costs 200
Remittance received from Burr in full payment $ 2,944
Jole Co. lent $10,000 to a major supplier in exchange for a non interest bearing note due in three years and a contract to purchase a fixed amount of merchandise from the supplier at a 10% discount from prevailing market prices over the next three years. The market rate for a note of this type is 10%. On issuing the note, Jole should record
Discount on Note Receivable Deferred Charge
A Yes Yes
B Yes No
C No Yes
D No No
Answer is A
Explanation:
Notes receivable are reported net of any discount.If a non-interest bearing (or low) note is exchanged for cash and a promise to provide future goods at lower-than-usual market prices, the issuer values the note at present value. The difference between present value and the cash payments is to be recognized as a part of the future goods’ cost, i.e., a deferred charge.
According to the percentage-of-outstanding-receivables method, which of the following is (are) true?
A
A percentage of uncollectible accounts in gross accounts receivable is determined based on the entity’s overall experience with uncollectible accounts over a period of time, adjusted for any relevant conditions.
B
The percentage of uncollectible accounts is applied to the ending balance of gross accounts receivable, to obtain the desired ending balance of the allowance for uncollectible accounts.
C
This method is balance-sheet oriented because it attempts to achieve a proper carrying amount for the accounts receivable at the end of a period, at net realizable value.
D
All of the above
Answer is D
Explanation:
A percentage of uncollectible accounts in gross accounts receivable is determined based on the entity’s overall experience with uncollectible accounts over a period of time, adjusted for any relevant conditions. The percentage of uncollectible accounts is applied to the ending balance of gross accounts receivable, to obtain the desired ending balance of the allowance for uncollectible accounts. This method is balance-sheet oriented because it attempts to achieve a proper carrying amount for the accounts receivable at the end of a period, at net realizable value. Answer D., all of the above, is the best choice.
During the year, Hauser Co. wrote off a customer's account receivable. Hauser used the allowance method for uncollectable accounts. What impact would the write-off have on net income and total assets? Net income Total assets A Decrease Decrease B Decrease No effect C No effect Decrease D No effect No effect
Answer is D
Explanation:
The journal entry to record the write-off of an account is as follows:
Allowance for Uncollectible Accounts XX
Accounts Receivable—Joe Doe XXThis entry would decrease both accounts receivable and allowance for uncollectible accounts. It has no impact on net income or total assets.
Which of the following describes portfolio segment disclosure in regards to credit losses?
A The level used by the entity in developing and documenting a systematic method for determining the allowance for credit losses B The level based on initial measurement attributes, risk characteristics of the financing receivables, and methods used by reporting entities related to monitoring and assessing credit risk C A fully aggregated basis of disclosure D None of the above
Answer is A
Explanation: In order to achieve the disclosure objective, reporting entities need to provide disclosures on two levels of disaggregation: portfolio segment and class of financing receivable. A portfolio segment is defined as the level used by the entity in developing and documenting a systematic method for determining the allowance for credit losses. Class of financing receivables generally represents a disaggregation of a portfolio segment, based on initial measurement attributes, risk characteristics of the financing receivables, and methods used by reporting entities related to monitoring and assessing credit risk.
Based on the aging of its accounts receivable at December 31 Terry Company determined that the net realizable value of the receivables at that date is $190,000. Additional information is as follows: Accounts receivable at 12/31 $220,000 Allowance for doubtful accounts at 1/1—credit balance 32,000 Accounts written off as uncollectible at 9/30 24,000Terry's doubtful accounts expense for the year ended December 31 is: A $38,000 B $30,000 C $26,000 D $22,000
Answer is D
Explanation:
The allowance account needs an ending balance of $30,000 [$220,000 - $190,000]. Currently, the allowance balance is $8,000 [$32,000 - $24,000], so an adjustment to bad debt expense and the allowance account for $22,000 is required.
Frame Co. has an 8% note receivable dated June 30, year 1, in the original amount of $150,000. Payments of $50,000 in principal plus accrued interest are due annually on July 1, year 2, year 3, and year 4. In its June 30, year 3, balance sheet, what amount should Frame report as a current asset for interest on the note receivable? A $0 B $ 4,000 C $ 8,000 D $12,000
Answer is C
Explanation:
The note can be recorded at its face amount of $150,000 because there is no indication that the rate of interest (8%) stipulated by the parties to the transaction does not represent fair and adequate compensation for the use of the funds. Payments of $50,000 in principal plus accrued interest are due annually on July 1, year 2, year 3, and year 4. Frame should report the interest receivable as a current asset in its 6/30, year 3 balance sheet, because the amount is to be received within one year of the balance sheet date (i.e., it is to be received 7/1, year 3).
Carrying amount of note, 6/30, yr1 $150,000
Less: Principal payment, 7/1, yr2 (50,000)
Carrying amount of note, 7/1, yr2 100,000
Times: Stated interest rate × 8%
Interest receivable, 6/30, year 3 $ 8,000
Rue Co.'s allowance for uncollectible accounts had a credit balance of $12,000 at December 31, year 2. During year 3, Rue wrote-off uncollectible accounts of $48,000. The aging of accounts receivable indicated that a $50,000 allowance for uncollectible accounts was required at December 31, year 3. What amount of uncollectible accounts expense should Rue report for year 3? A $48,000 B $50,000 C $60,000 D $86,000
Answer is D
Explanation:
Under the aging of accounts receivable method, after the desired ending balance of the allowance group is determined, the amount of uncollectible accounts (bad debt) expense recognized is the difference between the existing balance in the allowance account and the desired ending balance. The allowance account started the year with a credit balance of $12,000. The write-offs during the year would have been a credit to accounts receivable and a debit to allowance for uncollectible accounts of $48,000 thus bringing the allowance account to a $36,000 debit balance at year-end. To get the allowance account to the desired $50,000 credit balance there would need to be a credit to allowance for uncollectible accounts and debit to uncollectible accounts expense for $86,000.
Which of the following is true regarding an entity that records a note receivable?
A
When a note is exchanged for cash and a promise to provide merchandise at a discount from market price, the note is presumed to have a present value at issuance equal to the cash proceeds exchanged.
B
Noninterest-bearing notes receivable and those with an unrealistic stated rate of interest are not reported on the balance sheet but disclosed in the notes to the financial statements.
C
Loan origination fees are expensed in full in the period incurred.
D
For interest-bearing notes calling for the prevailing rate of interest at the time of issuance, the present value of the note is the same as the face amount of the note.
Answer is D
Explanation:
The correct answer is (D).
For interest-bearing notes calling for the prevailing rate of interest at the time of issuance, the present value of the note is the same as the face amount of the note.
When a note is exchanged for cash and a promise to provide merchandise at a discount from market price, the issuer records the note at present value.
The difference between fair value and cash payments is recognized as interest revenue over the contract life and is recorded as part of the cost of the related merchandise.
For non-interest-bearing notes and those with an unrealistic stated rate of interest, the receivable must be reported at its present value or the fair value of the property, good, or service exchanged, whichever is more clearly determinable.
In its December 31 balance sheet, Butler Co. reported trade accounts receivable of $250,000 and related allowance for uncollectible accounts of $20,000.
What is the total amount of risk of accounting loss related to Butler’s trade accounts receivable, and what amount of that risk is off-balance sheet risk?
Risk of accounting loss Off-balance sheet risk A $0 $0 B $230,000 $0 C $230,000 $20,000 D $250,000 $20,000
Answer is B
Explanation:
The total risk of accounting loss is the amount of potential loss the entity would suffer if all parties to the financial instruments failed completely to perform and the amounts due proved to be of no value to the entity. Butler Co. had already recorded an allowance for uncollectible accounts of $20,000 on its trade accounts receivable of $250,000, so the net of $230,000 is the risk of accounting loss. The entire amount is shown on the balance sheet, thus there is no off-balance sheet risk involved.
CLOSE
On July 1 of the previous year, Kay Corp. sold equipment to Mando Co. for $100,000. Kay accepted a 10% note receivable for the entire sales price. This note is payable in two equal installments of $50,000 plus accrued interest on December 31 of the previous and current year. On July 1 of the current year, Kay discounted the note at a bank at an interest rate of 12%. Kay's proceeds from the discounted note were A $48,400 B $49,350 C $50,350 D $51,700
Answer is D
Explanation:
Face amount of note, 7/1 of previous year $100,000
Less: payment of first installment, 12/31, previous year (50,000)
Face amount of note, 12/31 of previous year (due 12/31 of the current year) $ 50,000
Add interest to maturity ($50,000 x 10% x 12/12) 5,000
Maturity value of remaining portion of note $ 55,000
Less: Bank discount ($55,000 x 12% x 6/12) (3,300)
Proceeds from discounted note $ 51,700
Rand, Inc. accepted from a customer a $40,000, 90-day, 12% interest-bearing note dated August 31 of the current year. On September 30, Rand discounted the note at the Apex State Bank at 15%. However, the proceeds were not received until October 1. In Rand's September 30 balance sheet, the amount receivable from the bank, based on a 360-day year, includes accrued interest revenue of A $170 B $200 C $300 D $400
Answer is A
Explanation: Face amount of note $ 40,000 Add interest to maturity ($40,000 x 12% × 90/360) 1,200 Maturity value of note $ 41,200 Less bank discount ($41,200 x 15% x 60/360) (1,030) Proceeds from discounted note $ 40,170 Less face amount of note (40,000) Accrued interest revenue, 9/30 $ 170
On the December 31 balance sheet of Mann Co., the current receivables consisted of the following:
Trade accounts receivable $ 93,000
Allowance for uncollectible accounts (2,000)
Claim against shipper for goods lost in transit (November) 3,000
Selling price of unsold goods sent by Mann on consignment at 130% of cost (not included in Mann’s ending inventory) 26,000
Security deposit on lease of warehouse used for storing some inventories 30,000
Total $150,000At December 31 the correct total of Mann’s current net receivables was
A
$ 94,000
B
$120,000
C
$124,000
D
$150,000
Answer is A
Explanation:
The goods out on consignment have not yet been sold and, thus, must be included in Mann’s inventory at their cost of $20,000 ($26,000 / 130%). The security deposit on lease of the warehouse should be classified as a noncurrent asset.
Trade accounts receivable 93,000
Less: Allowance for uncollectible accounts (2,000)
Plus: Claim against shipper for goods lost in transit 3,000
Current net receivables, 12/31 $94,000
Marr Co. had the following sales and accounts receivable balances, prior to any adjustments at year end: Credit sales $10,000,000 Accounts receivable 3,000,000 Allowance for uncollectible accounts (debit balance) 50,000Marr uses 3% of accounts receivable to determine its allowance for uncollectible accounts at year end. By what amount should Marr adjust its allowance for uncollectible accounts at year end? A $0 B $40,000 C $90,000 D $140,000
Answer is D
Explanation:
The amount of the adjustment to the uncollectible account is the difference between the existing balance and the desired ending balance. The uncollectible account currently has a debit balance of $50,000. To get the allowance account to the desired $90,000 credit balance (3% × $3,000,000) there would need to be a credit to allowance for uncollectible accounts and debit to uncollectible accounts expense for $140,000.
At the end of year one, Boller Co. had an ending balance in the allowance for uncollectible accounts of $30,000. During year two, Boller wrote-off $40,000 of accounts receivable. At the end of year two, Boller had $300,000 in accounts receivable and determined that 8% of these would be uncollectible. What amount should be reported as uncollectible accounts expense on Boller’s year two income statement?
A $64,000 B $34,000 C $24,000 D $14,000
Answer is B
Explanation:
The correct answer is (B).
Boller Co. should report uncollectible account expense in the income statement for year 2 will be at $34,000.
Allowance for Uncollectible Account
Write Off
$40,000
Opening Balance
$30,000
Allowance for Uncollectible
$24,000
Bad Debt Expense (Plug)
$34,000
Alternative Method:
Summary
Amount
Write-offs in the year 2
$40,000
Bad debt expense
$34,000
Ending balance from year 1
$(30,000)
Allowance of uncollectible in year 2 (i.e. $300,000 x 8%)
The following information relates to Jay Co.’s accounts receivable for the current year:
Accounts receivable, 1/1 $ 650,000
Credit sales for the year 2,700,000
Sales returns for the year 75,000
Accounts written off during year 40,000
Collections from customers during year 2,150,000
Estimated future sales returns at 12/31 50,000
Estimated uncollectible accounts at 12/31 110,000
What amount should Jay report for accounts receivable, before allowances for sales returns and uncollectible accounts, at December 31?
A $1,200,000 B $1,125,000 C $1,085,000 D $ 925,000
Answer is C
Explanation:
Credit sales increase accounts receivable (A/R). Collections from customers, accounts written-off, and sales returns decrease A/R. Since the estimated future sales returns and the estimated uncollectible accounts are recorded in allowance accounts to A/R, they do not directly decrease the balance of the Accounts Receivable account.
Accounts Receivable
Balance, 1/1 650,000
Credit sales 2,700,000
2,150,000
40,000
75,000
Collections from customers
Accounts written-off
Sales returns
Balance, 12/31 1,085,000
When the allowance method of recognizing uncollectible accounts is used, how would the collection of an account previously have written off affect accounts receivable and the allowance for uncollectible accounts?
Accounts receivable Allowance for uncollectible accounts A Increase Decrease B Increase No effect C No effect Decrease D No effect Increase
Answer is D
Explanation:
Journal entries to record the collection of an account previously written off as uncollectible are as follows.
Accounts Receivable—Joe Doe XX
Allowance for Uncollectible Accounts XX
To reopen account to the balance it had when written off.
Cash XX
Accounts Receivable—Joe Doe XX
To record receipt of cash in payment of the receivable
These entries would increase cash and allowance for uncollectible accounts. They would have no net effect on net accounts receivable, net income, current assets, or working capital.
Gar Co. factored its receivables without recourse with Ross Bank. Gar received cash as a result of this transaction, which is best described as a
A
Loan from Ross collateralized by Gar’s accounts receivable.
B
Loan from Ross to be repaid by the proceeds from Gar’s accounts receivable.
C
Sale of Gar’s accounts receivable to Ross, with the risk of uncollectible accounts retained by Gar.
D
Sale of Gar’s accounts receivable to Ross, with the risk of uncollectible accounts transferred to Ross.
Answer is D
Explanation:
Factoring of receivables is in substance a sale of receivables when the transfer is without recourse (i.e., the financing institution or “factor” assumes the risk of collection).
The following information pertains to Oro Corp:
Credit sales for the year ended December 31 $450,000
Credit balance in allowance for uncollectible accounts at January 1 10,800
Bad debts written off during the year 18,000According to past experience 3% of Oro’s credit sales have been uncollectible. After provision is made for bad debt expense for the year ended December 31, the allowance for uncollectible accounts balance would be:
A
$6,300
B
$13,500
C
$24,300
D
$31,500
Answer is A
Explanation:
Allowance credit balance of $10,800 - Write offs $18,000 + 3% credit sales of $13,500 (450,000 * 3%) = $6,300
Delta, Inc. sells to wholesalers on terms of 2/15, net 30. Delta has no cash sales but 50% of Delta’s customers take advantage of the discount. Delta uses the gross method of recording sales and trade receivables. An analysis of Delta’s trade receivables balances at December 31 revealed the following:
Age Amount Collectible 0-15 days $100,000 100% 16-30 days $60,000 95% 31-60 days $5,000 90% over 60 days 2,500 $500 $ 167,500 In its December 31 balance sheet, what amounts should Delta report for allowance for discounts?
A $1,000 B $1,620 C $1,675 D $2,000
Answer is A
Explanation:
Only the receivables which have aged 0-15 days are eligible for the discount. The discount is computed using 50% of the dollar amount eligible, not 50% of
the discount. Only 50% of the customers take advantage of the discount.
Amount eligible for discount $100,000 % of customers that take discount × 50% Amount of eligible amount taken 50,000 Discount allowed × 2% Allowance for discount $ 1,000
Tinsel Co.'s balances in allowance for uncollectible accounts were $70,000 at the beginning of the current year and $55,000 at year end. During the year, receivables of $35,000 were written off as uncollectible. What amount should Tinsel report as uncollectible accounts expense at year end? A $15,000 B $20,000 C $35,000 D $50,000
Answer is B
Explanation:
The allowance account has an ending balance of $55,000. Prior to the bad debt adjustment, the allowance balance is $35,000 ($70,000 - $35,000 in writeoffs), so an adjustment to bad debt expense and the allowance account for $20,000 is required.
On December 30 of the current year, Chang Co. sold a machine to Door Co. in exchange for a non-interest-bearing note requiring ten annual payments of $10,000. Door made the first payment on that same date. The market interest rate for similar notes at date of issuance was 8%. Information on present value factors is as follows: Period Present value of $1 at 8% Present value of ordinary annuity of 1$ at 8% 9 0.50 6.25 10 0.46 6.71In its December 31 year-end balance sheet, what amount should Chang report as note receivable? A $45,000 B $46,000 C $62,500 D $67,100
Answer is C
Explanation:
Notes receivable are generally required to be recorded at their present value. At December 31, Chang is owed 9 more annual payments of $10,000. The appropriate factor to apply is the present value of ordinary annuity of $1 at 8% for 9 periods, which is given as 6.25. $10,000 x 6.25 = $62,500.
For the current year ended December 31, Beal Co. estimated its allowance for uncollectible accounts using the year-end aging of accounts receivable. The following data are available:
Allowance for uncollectible accounts, 1/1 $42,000
Provision for uncollectible accounts (2% on credit sales of $2,000,000) 40,000
Uncollectible accounts written-off, 11/30 46,000
Estimated uncollectible accounts per aging, 12/31 52,000After the year-end adjustment, the uncollectible accounts expense for the current year should be
A
$46,000
B
$48,000
C
$52,000
D
$56,000
Answer is D
Explanation:
Since Beal uses the aging method, the credit sales information is irrelevant. The balance in the allowance account was a debit of $4,000 (42,000 - 46,000) prior to the year-end adjustment. Given that the year-end allowance balance should be $52,000, bad debt expense would be debited for $56,000, and the allowance account would be credited for $56,000.
In its previous year-end balance sheet, Fleet Co. reported accounts receivable of $100,000 before allowance for uncollectible accounts of $10,000. Credit sales during the current year were $611,000, and collections from customers, excluding recoveries, totaled $591,000. During the year , accounts receivable of $45,000 were written off and $17,000 were recovered. Fleet estimated that $15,000 of the accounts receivable at December 31 were uncollectible.
In its December 31 current year balance sheet, what amount should Fleet report as accounts receivable before allowance for uncollectible accounts?
A $58,000 B $67,000 C $75,000 D $82,000
Answer is C
Explanation:
The A/R balance was increased by credit sales and decreased by collections from customers and by accounts written off. Accounts recovered resulted in an increase and a decrease of equal amount to the A/R balance.
Accounts Receivable
Balance, 1/1 100,000
Credit sales 611,000
591,000
Customer collections
Accounts reinstated 17,000
45,000
Accounts written-off
17,000
Accounts recovered
Balance, 12/31 75,000
On December 31, Key Co. received two $10,000 non-interest bearing notes from customers in exchange for services rendered.The note from Alpha Co.,which is due in nine months,was made under customary trade terms, but the note from Omega Co., which is due in two years, was not. The market interest rate for both note sat the date of issuance is 8%. The present value of $1 due in nine months at 8% is .944. The present value of $1 due in two years at 8% is .857. At what amounts should these two notes receivable be reported in Key’s December 31 balance sheet?
Alpha Omega A $9,440 $8,570 B $10,000 $8,570 C $9,440 $10,000 D $10,000 $10,000
Answer is B
Explanation:
Notes receivable are claims usually not arising from sales in the ordinary course of business.Legally,the claim is evidenced by a note representing an unconditional promise to pay. The recording of notes receivable is at their present value. This is not intended to apply to receivables and payables arising from transactions with customers or suppliers in the normal course of business which are due in customary trade terms not exceeding approximately one year. Thus, the note from Alpha Co. would be reported at the $10,000 face value. The note from Omega would be discounted at $8,570 ($10,000 × .857).
Red Co. had $3 million in accounts receivable recorded on its books. Red wanted to convert the $3 million in receivables to cash in a more timely manner than waiting the 45 days for payment as indicated on its invoices. Which of the following would alter the timing of Red's cash flows for the $3 million in receivables already recorded on its books? A Change the due date of the invoice B Factor the receivables outstanding C Discount the receivables outstanding D Demand payment from customers before the due date
Answer is B
Explanation:
Only factoring the receivables would ensure altering the timing of cash flows for the $3 million in receivables already recorded on its books. Factoring the receivables outstanding is a transfer of the receivables to a factor (transferee) without recourse and is accounted for as any other sale of an asset: debit cash, credit the receivables, and record a gain or loss for the difference. Changing the due date of the invoice, discounting the receivables outstanding, and demanding payment from customers before the due date will not necessarily get the cash for receivables any quicker and alter the timing of cash flows.
On December 31 of the current year, Jet Co. received two $10,000 notes receivable from customers in exchange for services rendered. On both notes, interest is calculated on the outstanding principal balance at the annual rate of 3% and payable at maturity. The note from Hart Corp., made under customary trade terms, is due in nine months and the note from Maxx, Inc. is due in five years. The market interest rate for similar notes on this date was 8%. The compound interest factors to convert future values into present values at 8% follow:
Present value of $1 due in nine months: .944
Present value of $1 due in five years: .680At what amounts should these two notes receivable be reported in Jet’s December 31 balance sheet?
Hart Maxx
A $9,440 $6,800
B $9,652 $7,820
C $10,000 $6,800
D $10,000 $7,820
Answer is D
Explanation:
Both notes were received on the balance sheet date. Since the note from Hart arose from a transaction with a customer in the normal course of business and is due in customary trade terms not exceeding one year, it can be reported at its face amount of $10,000 despite the fact that the 3% stated interest rate of the note differs from the prevailing market interest rate of 8% for similar notes at the transaction date. On the other hand, the note from Maxx is due in more than one year. Therefore, the note from Maxx cannot be reported at its face amount because its 3% stated interest rate differs from the prevailing market interest rate of 8% for similar notes at the transaction date. Because neither the fair value of the services performed by Jet nor the fair value of the note received from Maxx is indicated, the note is reported at its present value, determined by discounting all future cash payments of the note at the prevailing (i.e., market) rate of interest for a note of this type.
Principal amount $ 10,000
Interest on outstanding principal balance due on maturity date of note [($10,000 × 3%) × 5] 1,500
Amount due on maturity date of note $ 11,500
Present value factor of $1 at 8% for 5 periods × 0.680
Present value of note received from Maxx $ 7,820
Milton Co. pledged some of its accounts receivable to Good Neighbor Financing Corporation in return for a loan. Which of the following statements is correct?
A
Good Neighbor Financing cannot take title to the receivables if Milton does not repay the loan. Title can only be taken if the receivables are factored.
B
Good Neighbor Financing will assume the responsibility of collecting the receivables.
C
Milton will retain control of the receivables.
D
Good Neighbor Financing will take title to the receivables, and will return title to Milton after the loan is paid.
Answer is C
Explanation:
Receivables may be pledged as security for loans. Control of the receivables is retained and collections on the receivables are usually required to be applied to a reduction of the loan. Good Neighbor Financing could take title to the receivables if Milton doesn’t pay the loan, does not assume the responsibility of collecting the receivables, and does not take title to the receivables until the loan is repaid.
On April 1, Aloe, Inc. factored $80,000 of its accounts receivable without recourse. The factor retained 10% of the accounts receivable as an allowance for sales returns and charged a 5% commission on the gross amount of the factored receivables. What amount of cash did Aloe receive from the factored receivables? A $68,000 B $68,400 C $72,000 D $76,000
Answer is A
Explanation:
Factoring is similar to a sale of receivables because it is generally without recourse and the factor generally handles the billing and collection function. Aloe would receive $68,000 from the factored receivables. The $80,000 factored amount less $8,000 (10% as an allowance for sales returns) and less $4,000 (for the 5% commission charged) equals $68,000.
Bee Co. uses the direct write-off method to account for uncollectible accounts receivable. During an accounting period, Bee’s cash collections from customers equal sales adjusted for the addition or deduction of the following amounts:
Accounts written-off Increase in accounts receivable balance
A Deduction Deduction
B Addition Deduction
C Deduction Addition
D Addition Addition
Answer is A
Explanation:
An increase in A/R indicates that there was an excess of sales over cash collections. A write-off of receivables will offset the gross increase in A/R without affecting cash. Regardless of the accrual method used to account for uncollectible accounts, the cash collections from customers equal sales adjusted by a deduction for an increase in A/R balance and a deduction for accounts written-off during the period.
Foster Co. adjusted its allowance for uncollectible accounts at year end. The general ledger balances for the accounts receivable and the related allowance account were $1,000,000 and $40,000, respectively. Foster uses the percentage-of-receivables method to estimate its allowance for uncollectible accounts. Accounts receivable were estimated to be 5% uncollectible. What amount should Foster record as an adjustment to its allowance for uncollectible accounts at year end? A $10,000 decrease B $10,000 increase C $50,000 decrease D $50,000 increase
Answer is B
Explanation:
The allowance account balance would increase by the difference between the required amount and the balance at the end of the year.
$1,000,000 Accounts receivable balance 5% Uncollectible 50,000 Balance required - 40,000 End of year balance in allowance account $ 10,000 Adjustment to increase balance
Clear Co.’s trial balance has the following selected accounts:
Cash (includes $10,000 in a bond sinking fund for long-term bond payable) $50,000 Accounts receivable $20,000 Allowance for doubtful accounts $5,000 Deposits received from customers $3,000 Merchandise inventory $7,000 Unearned rent $1,000 Investment in trading securities $2,000 What amount should clear report as total current assets in its balance sheet?
A $64,000 B $67,000 C $72,000 D $74,000
Answer is A
Explanation:
The correct answer is (A).
Ref Summary Amount
A Total Cash $50,000
B Less: Cash used to pay Long-Term Debt $10,000
C Available Cash (A-B) $40,000
D Accounts Receivable $20,000
E Less: Allowance for doubtful debts $5,000
F Net Receivable (D-E) $15,000
G Merchandise Inventory $7,000
H Trading securities $2,000
I Total Current Assets (C + F + G + H) $64,000
The bond sinking fund is not a current asset. The bond sinking fund is reported right after current assets on the balance sheet. The bond sinking fund is part of long-term assets.
Customer deposits and unearned rents represent amounts already received and included in cash in exchange for goods or services that have not yet been delivered. These items are reported in financial statements in liabilities, and not under current assets.
A note receivable bearing a reasonable interest rate is sold to a bank with recourse. At the date of the discounting transaction, the notes receivable discounted account should be
A
Decreased by the proceeds from the discounting transaction.
B
Increased by the proceeds from the discounting transaction.
C
Increased by the face amount of the note.
D
Decreased by the face amount of the note.
Answer is C
Explanation:
A company that discounts a note receivable with recourse is contingently liable to the lender. It must pay the lender the amount due at maturity if the maker of the note fails to pay the obligation. The contingent liability is usually shown in the accounts by recording the note discounted in a Notes Receivable Discounted account at the note’s face amount. The Notes Receivable Discounted account is reported as a contra asset and deducted from Notes Receivable in the balance sheet.
Notes receivable $XXX
Less: Notes receivable discounted (XXX) $XXXExplanation:
A company that discounts a note receivable with recourse is contingently liable to the lender. It must pay the lender the amount due at maturity if the maker of the note fails to pay the obligation. The contingent liability is usually shown in the accounts by recording the note discounted in a Notes Receivable Discounted account at the note’s face amount. The Notes Receivable Discounted account is reported as a contra asset and deducted from Notes Receivable in the balance sheet.
Notes receivable $XXX
Less: Notes receivable discounted (XXX) $XXX
Inge Co. determined that the net value of its accounts receivable at December 31 of the current year, based on an aging of the receivables, was $325,000.
Additional information is as follows:
Allowance for uncollectible accounts, 1/1 $30,000
Uncollectible accounts written-off during the year 18,000
Uncollectible accounts recovered during the year 2,000
Accounts receivable at 12/31 350,000
For the current year ending December 31, what would be Inge’s uncollectible accounts expense?
A $5,000 B $11,000 C $15,000 D $21,000
ANswer is B
Explanation:
The correct answer is (B).
After the allowance for uncollectible accounts at 12/31 is determined, uncollectible accounts expense can be analyzed.
Current year write-offs 18,000 Balance - 1/1 30,000
Uncollectible accounts recovered 2,000
Uncollectible accounts expense (plug) 11,000
Balance - 12/31 25,000
A method of estimating uncollectible accounts that emphasizes asset valuation rather than income measurement is the allowance method based on A Aging the receivables. B Direct write off. C Gross sales. D Credit sales less returns and allowances.
Answer is A
Explanation:
The allowance method based on aging the receivables attempts to value the receivables at their future collectible amounts. Thus, it emphasizes asset valuation rather than income measurement.
At January 1, Jam Co. had a credit balance of $260,000 in its allowance for uncollectible accounts. Based on past experience, 2% of Jam’s credit sales have been uncollectible. During the year, Jam wrote off $325,000 of uncollectible accounts. Credit sales for the year were $9,000,000. In its December 31 year end balance sheet, what amount should Jam report as allowance for uncollectible accounts?
A $115,000 B $180,000 C $245,000 D $440,000
ANswer is A
Explanation:
Based on the information given, Jamin uses the percentage of sales method, under which bad debt expense is calculated as a percentage of credit sales (credit sales of $9,000,000 × 2% = $180,000), charged to Bad Debt Expense, and credited to the Allowance for Uncollectible Accounts.The allowance account balance is not considered in determining the amount of bad debt expense.
Allowance for Uncollectible Accounts
260,000
Beg. bal. 1/1
Write-offs 325,000
180,000
Bad debt expense
115,000
12/31 balance
___________ refers to the sale of a note to a third party, usually a bank or other financial institution.
A Factoring B Assigning C Discounting D None of the above
Answer is C
Explanation:
The correct answer is (C).
Discounting refers to the sale of a note to a third party, usually a bank or other financial institution.
The sale of a Note Receivable at a value less than the face value is Discounting.
The sale of Accounts Receivables for a fee is known as Factoring.
Assigning is s a lending agreement whereby the borrower assigns Accounts Receivable to the lending institution in exchange for a loan, where the Accounts Receivable serves as collateral.
Could current cost financial statements report holding gains for goods sold during the period and holding gains on inventory at the end of the period? Goods sold Inventory A Yes Yes B Yes No C No Yes D No No
Answer is A
Explanation:
An increase in the current cost of inventory items sold is a realized holding gain. An increase in the current cost of inventory items on hand is an unrealized holding gain. Current cost financial statements will measure and report both realized and unrealized holding gains.
Wilson Corp. experienced a $50,000 decline in the market value of its inventory in the first quarter of its fiscal year. Wilson had expected this decline to reverse in the third quarter, and in fact, the third quarter recovery exceeded the previous decline by $10,000. Wilson's inventory did not experience any other declines in market value during the fiscal year. What amounts of loss and/or gain should Wilson report in its interim financial statements for the first and third quarters? First quarter Third quarter A $0 $0 B $0 $10,000 gain C $50,000 loss $50,000 gain D $50,000 loss $60,000 gain
Answer is A
Explanation:
The use of lower of cost or market may result in inventory losses that should not be deferred beyond the interim period in which the decline occurs. Recoveries of these losses in subsequent periods should be recognized as gains, but only to the extent of losses recognized in previous interim periods of the same fiscal year. Temporary market declines, however, need not be recognized at the interim dates since no loss is expected to be incurred in the fiscal year. Because Wilson expected the decline to reverse within the fiscal year, no loss should be recorded for the first quarter. Even though the recovery exceeded the previous decline by $10,000, gains are recognized only to the extent of losses recognized in previous interim periods of the same fiscal year.
Which of the following statements describes a perpetual inventory system?
A
A perpetual inventory system is characterized by no entries being made to the inventory account during the period.
B
The balance in the inventory account at any time reveals the inventory that should be on hand.
C
Acquisitions of inventory goods are debited to “Purchases” while issuances are not recorded.
D
Cost of goods sold (CGS) is a residual amount obtained by subtracting the ending inventory from the sum of beginning inventory and net purchases.
Answer is B
Explanation:
In a periodic inventory system, acquisitions of inventory goods are debited to “Purchases” while issuances are not recorded. At any point in time the balance in the inventory account reflects the amount at the “beginning” of the period. The inventory on hand is “periodically” determined by physical count. In a periodic inventory system, cost of goods sold (CGS) is a residual amount obtained by subtracting the ending inventory from the sum of beginning inventory and net purchases. In a perpetual inventory system, the balance in the inventory account at any time reveals the inventory that should be on hand.
A company manufactures and distributes replacement parts for various industries. As of December 31, year 1, the following amounts pertain to the company’s inventory:
Item Cost Net
replacement
cost Sale price Cost to sell
or dispose Normal
profit margin
Blades $41,000 $ 38,000 $ 50,000 $ 2,000 $15,000
Towers 52,000 40,000 54,000 4,000 14,000
Generators 20,000 24,000 30,000 2,000 6,000
Gearboxs 80,000 105,000 120,000 12,000 8,000
What is the total carrying value of the company’s inventory as of December 31, year 1, under IFRS?
A $178,000 B $191,000 C $193,000 D $207,000
Answer is B
Explanation:
Under IFRS, inventory is carried at the lower of cost or net realizable value (best estimate of the net amounts inventories are expected to realize).
Item Cost Sale price Cost to sell or dispose Lower of cost/NRV Blades $41,000 $ 50,000 $ 2,000 $ 41,000 Towers 52,000 54,000 4,000 50,000 Generators 20,000 30,000 2,000 20,000 Gearboxes 80,000 120,000 12,000 \_\_\_80,000 12/31, yr 1 $191,000
Assuming constant inventory quantities, which of the following inventory-costing methods will produce a lower inventory turnover ratio in an inflationary economy?
A FIFO (first in, first out) B LIFO (last in, first out) C Moving average D Weighted average
Answer is A
Explanation:
Inventory turnover ratio = COGS/Average inventory. In an inflationary economy, COGS reported using FIFO is low, leading to higher net profits. Inventory would be higher since it is reported at current prices. Therefore, higher Inventories and lower COGS will result in a lower inventory turnover ratio. In turnover ratios, turn-it-over to the denominator. So, higher inventory and lower COGS would lead to lower inventory ratio.
Option (B) is incorrect because the inventory turnover ratio is lower as the COGS reported are higher because the costs of the units purchased are higher when compared to the cost of the units first purchased and inventory reported are lower as it consists of earlier purchases.
Option (C) and (D) are incorrect as per the above explanation.
Brock Co. adopted the dollar-value LIFO inventory method as of January 1, year 1. A single inventory pool and an internally computed price index are used
to compute Brock’s LIFO inventory layers. Information about Brock’s dollar value inventory follows:
Inventory
Date At base year cost At current year cost At dollar value LIFO
1/1, year 1 $40,000 $40,000 $40,000
Year 1 layer 5,000 14,000 6,000
12/31, year 1 $45,000 $54,000 $46,000
Year 2 layer 15,000 26,000 ?
12/31, year 2 $60,000 $80,000 ?
What was Brock’s dollar value LIFO inventory at December 31, year 2?
A $80,000 B $74,000 C $66,000 D $60,000
Answer is C
Explanation:
The price index is computed by dividing the ending inventory at current year cost by its base year cost.
Date Layers at base year cost Price index Ending inventory at LIFO cost
01/01, year 1 $40,000 1.0000 [1] $40,000
12/31, year 1 5,000 1.2000 [2] 6,000
12/31, year 2 15,000 1.3333 [3] 20,000
$60,000 $66,000
[1] $40,000 / $40,000 [2] $54,000 / $45,000 [3] $80,000 / $60,000
Trans Co. uses a periodic inventory system. The following are inventory transactions for the month of January:
1/1 Beginning inventory 10,000 units at $3
1/5 Purchase 5,000 units at $4
1/15 Purchase 5,000 units at $5
1/20 Sales at $10 per unit 10,000 units
Trans uses the average pricing method to determine the value of its inventory. What amount should Trans report as cost of goods sold on its income statement for the month of January?
A $ 30,000 B $ 37,500 C $ 40,000 D $100,000
Answer is B
Explanation:
Using the average cost method, Weighted average cost = Cost of goods available for sale/Number of units available for sale = $75,000/20,000 units = 3.75. COGS reported would be, Units sold x Weighted average cost = 10,000 units x $3.75 = 37,500.
Date
Inventory Units
Cost per Unit
Amount
1 – Jan
10,000
$3
$30,000
5 - Jan
5,000
$4
$20,000
15 - Jan
5,000
$5
$25,000
Total
20,000
$75,000
Option (A) is incorrect because the COGS are calculated based on the unit price of the beginning inventory.
Option (C) is incorrect because it is calculated based on the average price of the units [($3 + $4 + $5)/3 = $4]; 10,000 units x $4= $40,000.
Option (D) is incorrect because it represents Sales (10,000 x $10) and not COGS.
Town Inc. is preparing its financial statements for the current year ended December 31. At December 31, Town had outstanding purchase orders in the ordinary course of business for purchase of a raw material to be used in its manufacturing process. The market price is currently higher than the purchase price and is not anticipated to change within the next year. What is the reporting requirement? A Disclosure only B Accrual only C Both accrual and disclosure D Neither accrual nor disclosure
Answer is D
Explanation:
Because the outstanding purchase orders occurred in the ordinary course of business and the raw materials have not yet been received, the purchase is not required to be accrued. Since the price difference is only a market price difference occurring in the ordinary course of business, disclosure is not required.
During periods of rising prices, when the FIFO inventory method is used, a perpetual inventory system results in an ending inventory cost that is
A
The same as in a periodic inventory system.
B
Higher than in a periodic inventory system.
C
Lower than in a periodic inventory system.
D
Higher or lower than in a periodic inventory system, depending on whether physical quantities have increased or decreased.
Answer is A
Explanation:
In all cases where FIFO is used, the inventory and cost of goods sold would be the same at the end of a period whether a perpetual or a periodic system is used. This is true because the same costs will always be first in and, therefore, first out, whether cost of goods sold is computed as cost of goods sold throughout the accounting period (the perpetual system) or as a residual at the end of the accounting period (the periodic system).This is true regardless of whether inventory prices rose or fell during the period.
Hutch Inc. uses the conventional retail inventory method to account for inventory. The following information relates to current year operations:
Average Cost Retail
Beginning inventory and purchases
$ 600,000 $920,000
Net markups 40,000
Net markdowns 60,000
Sales 780,000
What amount should be reported as cost of sales for the current year?
A $480,000 B $487,500 C $520,000 D $525,000
Explanation:
The correct answer is (D)
Cost of sales is determined by subtracting the estimated ending inventory at cost from the cost of the beginning inventory and purchases (i.e., $600,000 – $75,000 = $525,000). Under the conventional retail inventory method, the beginning inventory and the net markups are included in the cost-to-retail ratio. The net markdowns are not included in the ratio.
=(920K + 40K- 780K +60K) x 62.5%***
= $ 75,000
***Cost-to-retail ratio ($600,000 / $960,000)= 62.5%
Thread Co. is selecting its inventory system in preparation for its first year of operations. Thread intends to use either the periodic weighted average method or the perpetual moving average method, and to apply the lower of cost or market rule either to individual items or to the total inventory. Inventory prices are expected to generally increase throughout the year, although a few individual prices will decrease. What inventory system should Thread select if it wants to maximize the inventory carrying amount at December 31?
Inventory method Cost or market application
A Perpetual Total inventory
B Perpetual Individual item
C Periodic Total inventory
D Periodic Individual item
Answer is A
Explanation:
In a period of rising prices, a moving weighted average method (perpetual system) will give a higher ending inventory figure than the weighted average method with a periodic system because with the moving weighted average method, more of the early (low) costs are released to cost of goods sold with that method than with a periodic system, leaving the higher costs for the ending inventory figure. Also, the item-by-item approach to applying the lower of cost or market rule is the more conservative method. Therefore, a higher inventory figure is obtained by use of the total inventory approach.
Kauf Co. had the following amounts related to the sale of consignment inventory:
Cost of merchandise shipped to consignee $72,000
Sales value for two-thirds of inventory sold by consignee 80,000
Freight cost for merchandise shipped 7,500
Advertising paid for by consignee, to be reimbursed 4,500
10% commission due the consignee for the sale 8,000
What amount should Kauf report as net profit(loss) from this transaction for the year?
A $(12,000) B $ 8,000 C $ 14,500 D $ 32,000
Answer is C
Explanation:
Cost of goods sold will be two-thirds of the goods shipped (including two-thirds of the freight), but all of the advertising and commission will be deducted from gross profit to determine net profit.
Revenue $ 80,000
Less: Cost of consigned goods $ 72,000
Plus Freight for consigned goods ____7,500
Total cost of consigned goods $ 79,500
Portion sold ______2/3
Cost of Goods Sold (rounded) _(53,000)
Gross Profit $ 27,000
Less advertising (4,500)
Less commission __(8,000)
Net Profit $ 14,500
Option (A) is incorrect because it is net profit as per above and not loss.
Option (B) is incorrect because $8,000 is net of sales value of the 2/3rd inventory sold by the consignee and the cost of the merchandise shipped to the consignee ($80,000 - $72,000).
Option (D) is incorrect because it does not deduct 2/3rd portion of the freight charges, advertising charges and 10% commission.
A firm’s ending inventory balance was overstated by $1,000. Which of the following statements is correct according to a periodic inventory system?
A
The retained earnings were overstated by $1,000.
B
The cost of goods sold was overstated by $1,000.
C
The cost of goods available for sale was overstated by $1,000.
D
The gross margin was understated by $1,000.
ANswer is A
Explanation:
In a periodic inventory system the inventory on hand is periodically determined by physical count. No entries are made to the inventory account during the period; the account reflects the amount at the beginning of the period until inventory is counted. A firm’s ending inventory balance being overstated by $1,000 would cause the retained earnings to be overstated by $1,000. Cost of goods sold (COGS) is obtained by subtracting the ending inventory from the sum of beginning inventory and net purchases, so COGS would be understated by $1,000. The cost of goods available is the sum of the beginning inventory and net purchases, and thus it is not affected by the ending inventory being overstated. The gross margin is a percentage used as part of the gross margin method to determine COGS. It may be used to verify the accuracy of the year-end physical count but otherwise it is not affected by the periodic inventory system.
Which of the following is not an indicator that an entity is an agent?
A
The entity does not have inventory risk before or after the goods have been ordered by a customer, during shipping, or on return.
B
The entity does not have discretion in establishing prices for the other party’s goods or services and, thus, the benefit that the entity can receive from those goods or services is limited.
C
The entity is exposed to credit risk for the amount receivable from a customer in exchange for the other party’s goods or services.
D
Another party is primarily responsible for fulfilling the contract.
ANswer is C
Explanation:
An agent is not exposed to credit risk for the amount receivable from a customer in exchange for the other party’s goods or services. The following are indicators that an entity is an agent –
Another party is primarily responsible for fulfilling the contract.
The entity does not have inventory risk before or after the goods have been ordered by a customer, during shipping, or on return.
The entity does not have discretion in establishing prices for the other party’s goods or services and, thus, the benefit that the entity can receive from those goods or services is limited.
The entity’s consideration is in the form of a commission.
The entity is not exposed to credit risk for the amount receivable from a customer in exchange for the other party’s goods or services.
Options (A), (B) and (D) are incorrect as they are all indicators that an entity is an agent.
The original cost of an inventory item is below both replacement cost and net realizable value. The net realizable value less normal profit margin is below the original cost. Under the lower of cost or market method, the inventory item should be valued at A Replacement cost. B Net realizable value. C Net realizable value less normal profit margin. D Original cost.
Answer is D
Explanation:
According to the lower of cost or market rule, market is defined as replacement cost. Market cannot exceed net realizable value and cannot be less than net realizable value less normal profit margin. In this instance, original cost is between net realizable value and net realizable value less normal profit margin. Since original cost is within the parameters for replacement cost and is less than replacement cost, the inventory should be reported at original cost.
The following information was derived from the current year accounting records of Clem Co.: Clem's Central Warehouse Clem's goods held by consignees Beginning inventory $110,000 $12,000 Purchases $480,000 $60,000 Freight in $10,000 Transportation to consignees $ 5,000 Freight out $ 30,000 $ 8,000 Ending inventory $145,000 $20,000Clem's cost of sales for the year was: A $455,000 B $485,000 C $507,000 D $512,000
Answer is D
Explanation:
Goods out on consignment remain the property of the consignor and must be included in the consignor’s inventory at purchase price or production cost, including freight in and other costs incurred to process the goods up to the time of sale.
Beginning inventory ($110,000 + $12,000) $122,000
Add: Purchases ($480,000 + $60,000) $540,000
Freight in 10,000
Transportation to consignees 5,000
Add: Total inventoriable costs 555,000
Goods available for sale 677,000
Less ending inventory ($145,000 + $20,000) 165,000
Cost of sales for the year $512,000
Estimates of price-level changes for specific inventories are required for which of the following inventory methods?
A Conventional retail B Dollar-value LIFO C Weighted average cost D Average cost retail
Answer is B
Explanation:
The dollar value LIFO method requires estimates of price-level changes for specific inventories. Under this method, goods are combined into pools and are traced by their dollar value, corrected for inflation. The annual layer added to the pool is tracked by calculating the ending inventory @ base year-end price. This is done to remove the effects of inflation from annual LIFO layers to gauge whether the increase or decreases to the inventory are real or due to inflation. The annual layer added @ base is then adjusted @current year price.
Option (A) is incorrect because the conventional retail inventory method is used by retailers to estimate the cost of their ending inventory by using the relationship between the cost of merchandise and its retail price (cost-to-retail ratio) arrived at after considering both net markups and markdowns.
Option (C) is incorrect because the weighted average cost method calculates the value of the ending inventory based upon the weighted average price paid.
Option (D) is incorrect because the average cost retail method does not require estimates of price-level changes for specific inventories.
On December 28, Kerr Manufacturing Co. purchased goods costing $50,000. The terms were F.O.B. destination. Some of the costs incurred in connection with the sale and delivery of the goods were as follows: Packaging for shipment $1,000 Shipping $1,500 Special handling charges $2,000These goods were received on December 31. In Kerr's December 31 balance sheet, what amount of cost for these goods should be included in inventory? A $54,500 B $53,500 C $52,000 D $50,000
Answer is D
Explanation:
The term F.O.B. destination means free on board at destination; that is, the goods are shipped to their destination without charge to the buyer. Thus, the costs incurred in connection with the sale and delivery of the goods (i.e, packaging for shipment, shipping, and special handling charges) are borne by the seller. Thus, Kerr’s cost of the goods purchased is $50,000.
Fireworks, Inc. had an explosion in its plant that destroyed most of its inventory. Its records show that beginning inventory was $40,000. Fireworks made purchases of $480,000 and sales of $620,000 during the year. Its normal gross profit percentage is 25%. It can sell some of its damaged inventory for $5,000. The insurance company will reimburse Fireworks for 70% of its loss. What amount should Fireworks report as loss from the explosion?
A $50,000 B $35,000 C $18,000 D $15,000
ANswer is D
Explanation:
Beginning inventory + Purchases - COGS - Ending inventory - Available inventory = Inventory damaged in explosion.
Ref
Summary Amount
a
Beginning Inventory
$40,000
b
Purchase, during the year
$480,000
c
Total Inventory
$520,000
d
COGS for Inventory Sold ($620,000 x (1-0.25))
$465,000
e
Proceeds from damaged Inventory
$5,000
f
Inventory Lost in Explosion (c-d-e)
$50,000
g
Proceeds from Insurance Company (f x 70%)
$35,000
h
Loss from Explosion (f-g)
$15,000
Option (A) is incorrect because $50,000 is the loss determined.
Option (B) is incorrect because $35,000 is the proceeds from insurance claim.
Option (C) is incorrect as per the above explanation.
As of December 31, year 2, a company has an inventory item that was originally purchased for $80 in year 1. The inventory item was written down to its net realizable value of $60 as of December 31, year 1. As of December 31, year 2, the inventory item had a net realizable value of $75 and a replacement cost of $65. Normal profit margins for this company are 20%. Under IFRS, what is the carrying amount of the inventory item as of December 31, year 2?
A $60 B $65 C $75 D $80
Explanation:
The correct answer is (C)
The carrying amount of the inventory item as of December 31st, year 2 is $75.
Under IFRS, inventories are measured at the lower of cost or net realizable value (NRV). Inventories will be written down if needed, and a write-down can only be reversed up to the original cost. On December 31, year 1 the cost of the inventory was $80 and the net realizable value (NRV) was $60. The inventory would be valued at the lower of the two values and would be recorded at $60 in the financial statements at the end of year 1. As of December 31, year 2, the NRV of the inventory was $75. The original cost was $80. The inventory would continue to be stated at the NRV of $75 as this is lower than the original cost of $80.
Ames Company determined the following values for its inventory as of December 31: Historical Cost $200,000 Replacement Cost $160,000 Sales Value $190,000 Cost to Complete and Sell $10,000 Normal Profit Margin $8,000 Fair Value $194,000Under IFRS, what amount should Ames report for inventory at December 31? A $194,000 B $180,000 C $172,000 D $160,000
Answer is B
Explanation: Compute market (i.e., NRV) and compare to cost
NRV = Selling price - disposal cost = [$190,000 - $10,000] = $180,000
Cost = $200,000
Bach Co. adopted the dollar value LIFO inventory method as of January 1, year 1. A single inventory pool and an internally computed price index are used to compute Bach''s LIFO inventory layers. Information about Bach''s dollar value inventory follows: Inventory at Date Base year cost Current year cost 1/1, Year 1 $90,000 $90,000 Year 1 layer $20,000 $30,000 Year 2 layer $40,000 $80,000What was the price index used to compute Bach''s Year 2 dollar value LIFO inventory layer? A 1.09 B 1.25 C 1.33 D 2.00
Answer is C
Explanation:
The price index is computed by dividing the ending inventory at current year cost by its base year cost, $200,000 / $150,000 = 1.333.
Date Base year cost Current year cost
1/1, Year 1 $90,000 $90,000
Year 1 layer 20,000 30,000
12/31 year 1 $110,000 $120,000
Year 2 layer 40,000 80,000
12/31 year 2 $150,000 $200,000
Beck Co.’s inventory of trees is as follows:
Beginning Inventory 10 trees at $50 March 4 Purchased 6 trees at $55 March 12 Sold 8 trees at $100 March 20 Purchased 9 trees at $60 March 27 Sold 7 trees at $105 March 30 Purchased 4 trees at $6 What was Beck's cost of goods sold using the last in, first out (LIFO) perpetual method?
A $910 B $850 C $808 D $775
Explanation:
The correct answer is (B).
7 trees sold March 27th are assumed to come entirely from the March 20th purchase at $60/each.
6 of the 8 trees sold March 12th are assumed to come from the March 4th purchase at $55/each and balance, 2 trees will come from beginning inventory at $50/each.
Cost of Goods Sold (COGS) using LIFO is [(7 x $60) + (6 x $55) + (2 x $50)] = $850
Seafood Trading Co. commenced operations during the year as a large importer and exporter of seafood. The imports were all from one country overseas. The export sales were conducted as drop shipments and were merely transshipped at Seattle. Seafood Trading reported the following data: Purchases during the year $12.0 million Shipping costs from overseas 1.5 million Shipping costs to export customers 1.0 million Inventory at year end 3.0 millionWhat amount of shipping costs should be included in Seafood Trading's year-end inventory valuation? A $0 B $250,000 C $375,000 D $625,000
ANswer is C
Explanation:
Merchandise inventory should include freight-in, taxes, insurance while in transit, warehousing costs, and similar charges paid by the purchaser to bring the merchandise to its existing condition and location. Thus, the $1.5 million in shipping costs from overseas should be included in the inventory valuation. The shipping costs to export customers are a selling expense and should not be included in the cost of inventory. Seafood purchased $12 million in inventory during the year and has $3 million remaining in inventory at year end. The $12 million divided by $3 million means one-quarter of the inventory is still remaining. Thus, one-quarter of the $1.5 million, or $375,000, in shipping costs from overseas should be included in the year-end inventory valuation.
For a customer to have obtained control of a product in a bill-and-hold arrangement, a substantive reason for the bill-and-hold arrangement must exist. Other criteria to be met for a bill-and-hold arrangement would be:
Product must be identified separately as belonging to the customer.
Product currently must be ready for physical transfer to the customer.
Entity cannot use the product or direct it to another customer.
Substantive reason for the bill-and-hold arrangement (e.g., the customer has requested the arrangement).
A
i, ii, and iv.
B
i and ii.
C
i, iii, and iv.
D
i, ii, iii, and iv.
Explanation:
The correct answer is (D).
A bill-and-hold arrangement is a contract under which an entity bills a customer for a product but the entity retains (i.e., holds) physical possession of the product until it is transferred to the customer at a point in time in the future. Revenue should be recognized only when the customer obtains control of the product.
For a customer to have obtained control of a product in a bill-and-hold arrangement, all of the following criteria must be met:
Substantive reason for the bill-and-hold arrangement (e.g., the customer has requested the arrangement).
Product must be identified separately as belonging to the customer.
The product currently must be ready for physical transfer to the customer
The entity cannot use the product or direct it to another customer.
Herc Co.’s inventory at December 31 of the previous year was $1,500,000, based on a physical count priced at cost, and before any necessary adjustment for the following:
Merchandise costing $90,000, shipped FOB shipping point from a vendor on December 30 of the previous year was received and recorded on January 5 of the current year.
Goods in the shipping area were excluded from inventory although shipment was not made until January 4 of the current year. The goods, billed to the customer FOB shipping point on December 30 had a cost of $120,000.
What amount should Herc report as inventory in its December 31, previous year balance sheet?
A
$1,500,000
B
$1,590,000
C
$1,620,000
D
$1,710,000
Answer is D
Explanation:
Goods should be included in the purchaser’s inventory when legal title passes to the purchaser. Therefore, Herc should include the $90,000 cost of goods shipped to it FOB shipping point in inventory at 12/31 of the previous year because title to these goods passed to Herc when the goods were picked up by the common carrier on 12/30. Herc should also include the $120,000 cost of goods in its shipping area in inventory at 12/31 of the previous year. These goods should be included in inventory because shipment of these goods to the customer was not made until the current year.
During January, Metro Co., which maintains a perpetual inventory system, recorded the following information pertaining to its inventory: Units Unit Cost Total Cost Units on Hand Balance on 1/1 1,000 $1 $1,000 1,000 Purchased on 1/7 600 3 1,800 1,600 Sold on 1/20 900 700 Purchased on 1/25 400 5 2,000 1,100Under the LIFO method, what amount should Metro report as inventory at January 31? A $1,300 B $2,700 C $3,900 D $4,100
ANswer is B
Explanation:
Where the LIFO cost flow method is used in conjunction with a perpetual inventory system, the cost of the last goods purchased are matched against revenue every time a sale is made ($700 + $2,000 = $2,700).
Date Purchased Sold Balance 1/1 (1,000 @ $1) $1,000 1/7 (600 @ $3) $1,800 (1,000 @ $1) $1,000 (600 @ $3) $1,800 1/20 (600 @ $3) $1,800 (700 @ $1) $ 700 (300 @ $1) $ 300 1/25 (400 @ $5) $2,000 (700 @ $1) $ 700 (400 @ $5) $2,000
Loft Co. reviewed its inventory values for proper pricing at year end. The following summarizes two inventory items examined for the lower of cost or market:
Inventory Item #1 Inventory Item #2
Original cost $210,000 $400,000
Replacement cost 150,000 370,000
Net realizable value 240,000 410,000
Net realizable value less profit margin 208,000 405,000
What amount should Loft include in inventory at year end, if it uses the total of the inventory to apply the lower of cost or market?
A $520,000 B $610,000 C $613,000 D $650,000
Answer is B
Explanation:
In applying the lower of cost or market, one must first determine the market value. The market refers to the current replacement cost, yet it should not exceed a ceiling (the net realizable value) nor be less than a floor, the net realizable value minus normal profit. The market value is determined by comparing the ceiling, floor and replacement costs. The total inventory ceiling is $650,000 ($240,000 + $410,000), the total inventory floor is $613,000 ($208,000 + $405,000), and the total inventory replacement cost is $520,000 ($150,000 + $370,000). The middle number is selected as the market; here it is the $613,000 floor amount. The total inventory cost of $610,000 is the value of inventory at year end because it is lower than the $613,000 market value.
Option (A) is incorrect because it does not use relevant information.
Option (C) is incorrect because it uses NRV - Normal profit.
Option (D) is incorrect because it uses NRV.
On December 31 of the previous year, Jason Company adopted the dollar-value LIFO retail inventory method. Inventory data are as follows:
LIFO Cost Retail
Inventory, 12/31 previous year
$360,000
$500,000
Inventory, 12/31 current year
—
660,000
Increase in price level for current year
10%
Cost to retail ratio for current year
70%
Under the LIFO retail method, Jason’s inventory at December 31 of the current year should be
A $437,000 B $462,000 C $472,000 D $483,200
Answer is A
Explanation:
Inventory at retail, 12/31 current year adjusted ($660,000 / 1.1)
$ 600,000
Beginning inventory at retail, base year price
(500,000)
New layer added in current year
100,000
Times: Price level adjustment
× 1.1
Current year layer, at LIFO retail
110,000
Times: Cost to retail ratio
× 0.70
Current year layer, at LIFO cost
77,000
Add: Beg. inventory, at LIFO cost
360,000
Ending inventory, 12/31 current year
$ 437,000
On December 30 of the previous year, Astor Corp. sold merchandise for $75,000 to Day Co. The terms of the sale were net 30, FOB shipping point. The merchandise was shipped last December 31 and arrived at Day on January 5 of the current year. Due to a clerical error, the sale was not recorded until January and the merchandise, sold at a 25% markup, was included in Astor's inventory at December 31. As a result, Astor's cost of goods sold for the previous year ended December 31 was A Understated by $75,000. B Understated by $60,000. C Understated by $15,000. D Correctly stated.
Answer is B
Explanation:
Goods should be removed from the seller’s inventory when legal title passes to the purchaser. Therefore, Astor should exclude the cost of the goods shipped FOB shipping from inventory at 12/31 because title of these goods passed to Day when the goods were picked up by the common carrier on 12/31. The cost of the goods is calculated by dividing the selling price by 125% ($75,000 / 1.25 = $60,000). Since the goods were erroneously in Astor’s inventory at 12/31, Astor’s previous year ending inventory and cost of goods sold were overstated and understated, respectively, by the $60,000 cost of the goods.
Which of the following would not be included in the cost of the finished goods inventory of a manufacturer? A Direct materials B Direct labor C Fixed and variable manufacturing overhead D Freight-out expenses
Answer is D
Explanation:
Freight-out expense is a selling expense and should not be included in the cost of inventory.
Which inventory costing method would a company that wishes to maximize profits in a period of rising prices use?
A FIFO B Dollar-value LIFO C Weighted average D Moving average
Answer is A
Explanation:
The correct answer is (A).
The FIFO inventory costing method assumes that the goods first acquired are the first sold.
This would maximize profits in a period of rising prices due to COGS being comprised of goods purchased at lower prices, which leads to a higher profit.
The lower-of-cost-or-market rule for inventories may be applied to total inventory, to groups of similar items, or to each item. Which application generally results in the lowest inventory amount? A All applications result in the same amount B Total inventory C Groups of similar items D Separately to each item
Answer is D
Explanation:
The application of the lower of cost or market (LCM) rule directly to each inventory item generally results in the lowest inventory amount because unrealized losses on inventory items cannot be offset by unrealized gains on other inventory items. Generally, a different inventory amount would be reported when the LCM rule is applied to (1) total inventory, (2) groups of similar inventory items, or (3) each inventory item. The total inventory and groups of similar items methods of applying the LCM rule for inventories would allow unrealized losses on some inventory items to be offset by unrealized gains on others. This would result in a higher inventory amount than if the LCM rules were applied to each inventory item.
A company manufactured 1,000 units of product during the year and sold 800 units. Costs incurred during the current year are as follows:
Direct materials and direct labor $7,000
Indirect materials and indirect labor $2,000
Insurance on manufacturing equipment $3,000
Advertising $1,000
What amount should be reported as inventory in the company’s year-end balance sheet?
A $1,400 B $1,800 C $2,600 D $2,400
Answer is D
Explanation:
The correct answer is (D).
In order to compute the inventory cost, the following items are considered:
Direct material, labor, and overheads
Freight inwards, insurance and warehousing (up to the point of sale)
Handling expenses, normal spoilage, repacking
Discounts received, if any
In the given case, per unit inventory cost can be computed as under:
Description Amount
Direct materials and direct labor 7,000
Indirect materials and indirect labor 2,000
Insurance on manufacturing equipment 3,000
Total inventory costs 12,000
Total units manufactured = 1,000
Per unit cost = 12,000/1,000 12
Closing stock = Units produced – Units sold = 1,000 – 800 = 200 units
The value of the inventory in the year-end = 200 x 12 = $2,400
On January 1 of the current year, Card Corp. signed a three-year, noncancelable purchase contract, which allows Card to purchase up to 500,000 units of a computer part annually from Hart Supply Co. at $.10 per unit and guarantees a minimum annual purchase of 100,000 units. During the year, the parts unexpectedly became obsolete. Card had 250,000 units of this inventory at December 31 and believes these parts can be sold as scrap for $.02 per unit. What amount of probable loss from the purchase commitment should Card report in its year-end income statement? A $24,000 B $20,000 C $16,000 D $ 8,000
Answer is C
Explanation:
A loss on the purchase commitment should be calculated based only on the minimum unit purchase requirement for the remaining years on the contract. Therefore, Card should calculate its loss at 12/31 based on the two years remaining on the purchase contract.
Minimum annual unit purchase requirement 100,000
Years remaining on contract (3 - 1) x 2
Minimum unit purchase requirement for remaining duration on contract 200,000
Expected loss per unit purchased ($0.10 - $0.02) x $0.08
Probable loss on purchase commitment $ 16,000
Simm Co. has determined its December 31 inventory on a LIFO basis to be $400,000. Information pertaining to the inventory follows:
Estimated selling price $408,000 Estimated cost of disposal $20,000 Normal profit margin $60,000 Current replacement cost $390,000 At December 31, what should be the amount of Simm's inventory?
A $400,000 B $390,000 C $388,000 D $328,000
Explanation:
The correct answer is (C).
When an entity follows the LIFO method of inventory valuation, the inventory is valued at the lower of cost or market value. Market value is the middle of:
Ceiling = Net Realizable Value = Selling price – costs of disposal
Floor = Net Realizable Value – normal profit margin
Replacement Cost
In the given case, market value shall be the middle of:
Ceiling: 408,000 – 20,000 = $388,000
Floor: 388,000 – 60,000 = $328,000
Current Replacement Cost: $390,000
Market value = $388,000
The inventory should be valued at the lower of $390,000 and $388,000.
The final value of inventory is $388,000
At the end of the year, Ian Co. determined its inventory to be $258,000 on a FIFO (first in, first out) basis. The current replacement cost of this inventory was $230,000. Ian estimates that it could sell the inventory for $275,000 at a disposal cost of $14,000. If Ian’s normal profit margin for its inventory was $10,000, what would be its net carrying value?
A $244,000 B $251,000 C $258,000 D $261,000
Answer is C
Explanation:
The correct answer is (C).
LIFO or Retail inventories are valued at the Lower of Cost or Market.
FIFO, Weighted Avg, and inventories determined by methods other than LIFO or Retail use the Lower of Cost or Net Realizable Value.
Ian Co. determined its inventory on a FIFO basis and its Inventory should be valued at Lower of Cost or Net Realizable Value (NRV).
Cost = $258,000.
NRV = Net Selling Price - Cost to complete & dispose ($275,000 - $14,000 = $261,000).
As Cost is lower than the NRV, inventory is valued at cost.
Option (A) is incorrect because it reduces the selling cost from the original cost of inventory.
Option (B) is incorrect because it uses market value instead of taking the net realizable value for valuing inventory on a FIFO basis.
Option (D) is incorrect because the cost is lower than the net realizable value.
Information with respect to Bruno Co.'s cost of goods sold for the current year is as follows: Historical cost Units Inventory, 1/1 $1,060,000 20,000 Production during year 5,580,000 90,000 6,640,000 110,000 Inventory, 12/31 2,520,000 40,000 Costs of goods sold $4,120,000 70,000Bruno estimates that the current cost per unit of inventory was $58 at January 1, and $72 at December 31. In Bruno's supplementary information restated into average current cost, the cost of goods sold for the current year should be A $5,040,000 B $4,550,000 C $4,410,000 D $4,060,000
Answer is B
Explanation:
To calculate cost of goods sold on a current cost basis, multiply the number of units sold by the average current cost of the units during the period (sum of the current cost of the units at the beginning and the end of the period, divided by two).
Current cost per unit, 1/17 $ 58
Current cost per unit, 12/31 + 72
Total 130
Divide by two / 2
Average current cost per unit during the year 65
Times units sold in the year × 70,000
Cost of goods sold, average current cost $4,550,000
On December 31 of the previous year, Jason Company adopted the dollar-value LIFO retail inventory method. Inventory data are as follows:
LIFO Cost Retail
Inventory, 12/31 previous year $360,000 $500,000
Inventory, 12/31 current year — 660,000
Increase in price level for current year 10%
Cost to retail ratio for current year 70%
Under the LIFO retail method, Jason’s inventory at December 31 of the current year should be
A $437,000 B $462,000 C $472,000 D $483,200
Answer is A
Explanation:
Inventory at retail, 12/31 current year adjusted ($660,000 / 1.1) $ 600,000
Beginning inventory at retail, base year price (500,000)
New layer added in current year 100,000
Times: Price level adjustment × 1.1
Current year layer, at LIFO retail 110,000
Times: Cost to retail ratio × 0.70
Current year layer, at LIFO cost 77,000
Add: Beg. inventory, at LIFO cost 360,000
Ending inventory, 12/31 current year $ 437,000
Which of the following statements are correct when a company applying the lower-of-cost-or-market method reports its inventory at replacement cost?
The original cost is less than replacement cost. The net realizable value is greater than replacement cost. A I only B II only C Both I and II D Neither I nor II
Answer is B
Explanation:
The answer to this question assumes that the original cost, replacement cost, and net realizable value of the inventory differ in amount. Statement II is correct. Under lower-of-cost-or-market (LCM) procedures for inventory valuation, market value cannot exceed a ‘ceiling’ of net realizable value and cannot be below a ‘floor’ of net realizable value reduced by a normal profit margin. Since the inventory is reported at its replacement cost, the net realizable of the inventory exceeds its replacement cost. Statement I is incorrect. Under LCM procedures, inventory is reported at the lower of original cost or market value (which is replacement cost in this case). Since the inventory is reported at replacement cost, the original cost of the inventory is greater than its replacement cost.
Garcel, Inc. held unfinished inventory at a cost of $85,000 with a sales value of $125,000. The inventory will cost $10,500 to complete. The normal profit margin is 30% of sales. The replacement cost of the inventory was $75,000. What amount should Garcel report as inventory on balance sheet? A $114,500 B $85,000 C $77,000 D $75,000
Answer is C
Explanation:
Garcel should report the unfinished inventory at the lower of cost or market. Market means current replacement cost except that market should not exceed the net realizable value and market should not be less than the net realizable value minus normal profit. The net realizable value is the sales value less reasonably predictable costs of completion ($125,000 – $10,500 = $114,500) and this is the ceiling value. The normal profit is the estimated sales value times the normal profit margin: $125,000 × 30% = $37,500. The net realizable value minus normal profit ($114,500 – $37,500 = $77,000) is the floor value. The replacement cost is $75,000 but the market cannot be lower than the floor value of $77,000 so $77,000 would be the market value. The lower of cost ($85,000) or market ($77,000) is the market value of $77,000.
A company records inventory at the gross invoice price. Theoretically, how should the following affect the costs in inventory?
Warehousing costs Cash discounts available
A Increase Decrease
B No effect Decrease
C No effect No effect
D Increase No effect
Explanation:
Warehousing costs are usually treated as an expense in the period in which they are incurred, although conceptually they comprise part of the total cost of merchandise made ready for sale. Conceptually, the cost of inventory should be reduced for cash discounts available, because the acquisition cost of an asset should not exceed its cash equivalent price.
Rose Co. sells one product and uses the last-in, first-out method to determine inventory cost. Information for the month of January follows: Total Units Unit Cost Beginning inventory, 1/1 8,000 $8.20 Purchases, 1/5 12,000 7.90 Sales 10,000 Rose has determined that at January 31, the replacement cost of its inventory was $8 per unit and the net realizable value was $8.80 per unit. Rose's normal profit margin is $1 per unit. Rose applies the lower of cost or market rule to total inventory and records any resulting loss. At January 31, what should be the net carrying amount of Rose's inventory? A $79,000 B $79,800 C $80,000 D $81,400
Answer is C
Explanation:
The market maximum, or ceiling, should not exceed the net realizable value (NRV), which is the estimated selling price in the ordinary course of business less reasonably predictable costs of completion and disposal. Market minimum, or floor should not be less than the net realizable value minus normal profit.
Market:
Ceiling (NRV) $8.80
Replacement cost $8.00
Floor (NRV minus normal profit) $7.80LIFO charges Cost of Goods Sold with the latest acquisition costs, while ending inventories are reported at the older costs of the earliest units. The 10,000 units sold reduce the 1/5 purchases, leaving 8,000 units from the beginning inventory and 2,000 units purchased on 1/5.
Cost [(8,000 x $8.20) + (2,000 x $7.90)] $81,400
Market (10,000 x $8.00) $80,000$80,000 is the lowest of cost or market.
The following items were included in Opal Co.’s inventory account at December 31:
Merchandise out on consignment, at sales price, including 40% markup on selling price $40,000
Goods purchased, in transit, shipped F.O.B. shipping point $36,000
Goods held on consignment by Opal $27,000
By what amount should Opal’s inventory account at December 31 be reduced?
A $103,000 B $ 67,000 C $ 51,000 D $ 43,000
Explanation:
The correct answer is (D).
Opal’s inventory account at December 31, be reduced by $16,000 + $27,000 = $43,000. Merchandise that is out on consignment will be included in Opal’s inventory till the time it is sold by the consignor.
As it is included in inventory at the sales price, including the margin on the selling price, the same will have to be adjusted for margin to reflect the cost of the inventory on consignment.
The $40,000 will have be reduced by margin of 40% = $16,000 (i.e. 40% of $40,000).
Goods purchased, in transit, shipped f.o.b. shipping point, will not be adjusted as they are already included in the inventory.
Goods held on consignment by Opal will be included in the inventory of the consignor, so inventory will have to be reduced by $27,000.
The retail inventory method includes which of the following in the calculation of both cost and retail amounts of goods available for sale?
A Purchase returns B Sales returns C Net markups D Freight in
Answer is A
Explanation:
When the retail method is employed, purchase returns is included in the calculation of both cost and retail amounts of goods available for sale (AFS). Sales returns does not appear in the computation of the cost amount of goods AFS. Net markups appears in the retail amount of goods AFS (assuming the retail method is used to approximate a lower of average cost or market figure) but not in the cost amount of goods AFS. Freight in appears in the cost amount of goods available for sale but not in the retail amount of goods AFS.
The following information was obtained from Smith Co.:
Sales $275,000
Beginning inventory 30,000
Ending inventory 18,000
Smith’s gross margin is 20%. What amount represents Smith purchases?
A $202,000 B $208,000 C $220,000 D $232,000
Explanation:
The correct answer is (B).
Beginning inventory + Net purchases - Ending inventory = COGS
Net Purchases = Ending inventory + COGS - Beginning inventory
Net Purchases = ($18,000 + $220,000 - $30,000) = $208,000. [COGS = Sales x (1- Gross Margin)]
[$275,000 (1-0.2) = $220,000.]
Option (A) and (D) are incorrect as per the above explanation.
Option (C) is incorrect because $220,000 is the COGS, not the net purchases.
Solve for Goods Available for Sale:
(GAFS – COGS = EI)
GAFS – $220,000 = $18,000
GAFS = $238,000
Solve for Purchases:
(BI – Purch = GAFS)
$30,000 – Purch = $238,000
Purchases = $208,000.
Option (A) and (D) are incorrect as per the above explanation.
Option (C) is incorrect because $220,000 is the COGS, not the net purchases
At the end of year 1, a company reduced its inventory cost from $100 to its net realizable value of $80. As of the end of year 2, the inventory was still on hand and its net realizable value increased to $150. Under IFRS, what journal entry should the company record for year 2 to properly report the inventory value?
A
Debit inventory for $20 and credit expense for $20
B
Debit inventory for $70 and credit expense for $70
C
Debit inventory for $70, credit retained earnings for $50 and credit expense for $20
D
Debit inventory for $20, debit expense for $30 and credit retained earnings for $50
Answer is A
Explanation:
The correct answer is (A).
Inventory was written down to its NRV of $80 (from $100). Under IFRS, Net Realizable Value (NRV) is the best approximation of how much inventories are expected to realize moving forward. IFRS only allows inventory recovery up to the point that was written off initially - i.e., $100.Although the NRV has increased to well above the original inventory cost (book value), only $20 of recovery is allowed.
The B/S entry would be:
Inventory $20
Expense $20
Note: Recoveries are recorded as expenses in the I/S because they are a Reduction to COGS.
Under IFRS, which of the following is not an acceptable method of accounting for inventory?
A Gross profit method B Retail method C LIFO D Weighted average
Answer is C
Explanation:
Under IFRS, specific identification is required when the goods are not interchangeable; otherwise, you can use FIFO, the gross profit method (if a physical count is not possible), the retail method (in certain industries), and weighted average. LIFO is prohibited under IFRS.
During January, Metro Co., which maintains a perpetual inventory system, recorded the following information pertaining to its inventory: Units Unit Cost Total Cost Units on Hand Balance on 1/1 1,000 $1 $1,000 1,000 Purchased on 1/7 600 3 1,800 1,600 Sold on 1/20 900 700 Purchased on 1/25 400 5 2,000 1,100Under the moving-average method, what amount should Metro report as inventory at January 31? A $2,640 B $3,225 C $3,300 D $3,900
Answer is B
Explanation:
Under the moving-average method, a new average unit price is computed every time a purchase is made. The inventory is then priced on the basis of this “moving average.”
Units Unit cost Total cost Balance on 1/1 1,000 $1.00 $1,000 Purchased on 1/7 600 3.00 1,800 Balance after purchase 1,600 1.75 * 2,800 Sold on 1/20 900 1.75 1,575 Balance after sale 700 1.75 1,225 Purchased on 1/25 400 5.00 2,000 Balance after purchase 1,100 $3,225 * $2,800 / 1,600 units
During the current year, Kam Co. began offering its goods to selected retailers on a consignment basis. The following information was derived from Kam's current year accounting records: Beginning Inventory $122,000 Purchases 540,000 Freight in 10,000 Transportation to consignees 5,000 Freight out 35,000 Ending Inventory--held by Kam 145,000 Ending Inventory--held by consignees 20,000In its current year income statement, what amount should Kam report as cost of goods sold? A $507,000 B $512,000 C $527,000 D $547,000
Answer is B
Explanation:
Goods out on consignment remain the property of the consignor and must be included in the consignor’s inventory at purchase price or production cost, including freight and other costs incurred to process the goods up to the time of sale.
Beginning inventory $122,000
Add: Purchases $540,000
Freight in shipping 10,000
Transportation to consignees 5,000
Add: Total inventoriable costs 555,000
Goods available for sale 677,000
Less: Ending inventory ($145,000 + $20,000) (165,000)
Cost of goods sold $512,000
According to the FASB conceptual framework, which of the following attributes would not be used to measure inventory? A Historical cost B Replacement cost C Net realizable value D Present value of future cash flows
Answer is D
Explanation:
The present value of future cash flows (which would include profits not yet earned) pertains to the time value of money and is not appropriate for measuring inventory. The primary basis for accounting for inventories is cost. The cost of an inventory item is the cash price or fair value of other consideration given in exchange for it. Historical cost, replacement cost, and net realizable value are all appropriate methods for measuring inventory.
Generally, which inventory costing method approximates most closely the current cost for each of the following? Cost of goods sold Ending inventory A LIFO FIFO B LIFO LIFO C FIFO FIFO D FIFO LIFO
Answer is A
Explanation:
In using LIFO, the cost of the last goods in are used in pricing the cost of goods sold. Therefore, the LIFO method will result in having cost of goods sold most closely approximate current cost. In using FIFO, the cost of the last goods are used in pricing the ending inventory. Thus, the FIFO method will result in having ending inventory most closely approximate current cost.
A flash flood swept through Hat, Inc.’s warehouse on May 1. After the flood, Hat’s accounting records showed the following:
Inventory, January 1 $ 35,000
Purchases, January 1 through May 1 200,000
Sales, January 1 through May 1 250,000
Inventory not damaged by flood 30,000
Gross profit percentage on sales 40%
What amount of inventory was lost in the flood?
A $55,000 B $85,000 C $120,000 D $150,000
Answer is A
Explanation:
The amount of inventory lost in the flood is calculated by determining the difference between the estimated ending inventory using the gross margin method
and the actual physical inventory not damaged by the flood.
Beginning inventory, January 1 $35,000
Purchases, January 1 through May 1 200,000
Goods available for sale 235,000
Sales, January 1 through May 1 $250,000
Less: Gross margin (40% x $250,000) (100,000)
Less: Estimated CGS (150,000)
Estimated ending inventory 85,000
Less: Physical ending inventory (30,000)
Estimated flood loss $55,000
The UNO Company was formed on January 2, year 1, to sell a single product. Over a two-year period, UNO’s acquisition costs have increased steadily. Physical quantities held in inventory were equal to three months’ sales at December 31, year 1, and zero at December 31, year 2. Assuming the periodic inventory system, the inventory cost method which reports the highest amount for each of the following is:
Inventory December 31, Year 1 Cost of Sales Year 2 A LIFO FIFO B LIFO LIFO C FIFO FIFO D FIFO LIFO
Answer is C
Explanation:
Under the last-in, first-out (LIFO) method of inventory valuation, the units remaining in ending inventory are costed at the oldest unit costs available. Under the first-in, first-out (FIFO) method of inventory valuation, the units remaining in ending inventory are costed at the most recent unit costs available. Therefore, because inventory acquisition costs increased steadily during year 1, the FIFO method of inventory valuation would report a higher amount for ending inventory than the LIFO method. The question indicates that there were no goods in inventory at 12/31, year 2. So, cost of goods sold for year 2 is comprised of the cost of inventory purchases made in year 2 and the cost of ending inventory at 12/31, year 1. Because the cost of the ending inventory at 12/31, year 1, is higher under FIFO, cost of goods sold for year 2 would also be higher under FIFO.
Option (A), (B) and (D) are incorrect as per above explanation.
West Retailers purchased merchandise with a list price of $20,000, subject to trade discounts of 20% and 10%, with no cash discounts allowable. West should record the cost of this merchandise as A $14,000 B $14,400 C $15,600 D $20,000
Answer is B
Explanation:
The cost of the merchandise is computed by subtracting the amount of the trade discounts from the list price of the goods.
List price $20,000 Less: trade discount—20% 4,000 Balance 16,000 Less: trade discount—10% 1,600 Cost of the merchandise $14,400
A company decided to change its inventory valuation method from FIFO to LIFO in a period of rising prices. What was the result of the change on ending inventory and net income in the year of the change?
Ending inventory Net income A Increase Increase B Increase Decrease C Decrease Decrease D Decrease Increase
Answer is C
Explanation:
The change in inventory valuation from FIFO to LIFO is a change in accounting principle which should be accounted for retrospectively. However, because it is impracticable to determine the retrospective impact of such a change, the change will impact the current period only. As LIFO results in lower valuation of ending inventory in a period of rising prices it will decrease the net income for the period.
Options (A), (B) and (D) are incorrect based on the above explanation.
Garcel, Inc. held unfinished inventory at a cost of $85,000 with a sales value of $125,000. The inventory will cost $10,500 to complete. The normal profit margin is 30% of sales. The replacement cost of the inventory was $75,000. What amount should Garcel report as inventory as per retail inventory method on the balance sheet?
A $114,500 B $85,000 C $77,000 D $75,000
Answer is C
Explanation:
The correct answer is (C).
Under US GAAP, inventory is valued at the lower of cost or market (LCM) if LIFO or retail inventory method is used. FASB has issued a recent update replacing the lower of cost or market value criteria with lower of cost or net realizable value (LCNRV) for goods determined by other than LIFO or retail inventory methods.
Market = Middle of the following three numbers.
Ceiling = NRV.
Floor = NRV - normal profit margin.
Replacement cost = Cost to purchase or reproduce
Cost is $85,000.
Market price is $77,000.
Ceiling: NRV = ($125,000 - $10,500) = $114,500.
Floor: NRV- Normal profit = ($114,500 - $37,500) = $77,000. [Normal Profit: ($125,000 x 30%) = $37,500]
Replacement Cost = $75,000
So Inventory should be valued at market price of $77,000
(A) is incorrect because it uses net realizable value.
(B) is incorrect because it uses original cost which is higher than market.
(D) is incorrect because it uses replacement cost.
Union Corp. uses the first-in, first-out retail method of inventory valuation. The following information is available: Cost Retail Beginning Inventory $12,000 $ 30,000 Purchases 60,000 110,000 Net additional markups 10,000 Net markdowns 20,000 Sales revenue 90,000If the lower of cost or market rule is disregarded, what would be the estimated cost of the ending inventory? A $24,000 B $20,800 C $20,000 D $19,200
Answer is A
Explanation:
Under the first-in, first-out (FIFO) retail method of inventory valuation, the goods in beginning inventory are charged to cost of goods sold during the period; therefore, the cost/retail ratio is based only on the purchases for the period. If the lower of cost or market rule is disregarded, both net additional markups and net markdowns are included in the purchases cost-to-retail ratio.
Cost Retail
Purchases $60,000 $110,000
Net additional markups – 10,000
Net markdowns – (20,000)
Purchases cost-to-retail ratio amounts 60,000 100,000
Beginning inventory $12,000 $ 30,000
Goods available for sale $72,000 $130,000
Less: sales (90,000)
Estimated ending inventory at retail $ 40,000
Cost-to-retail ratio ($60,000 / $100,000) x 60%
Estimated ending inventory at cost $ 24,000
On January 1, year 1, Poe Company adopted the dollar-value LIFO inventory method. Poe's entire inventory constitutes a single pool. Inventory data for year 1 and year 2 are as follows: Date Inventory at current year cost Inventory at base year cost Relevant price index 1/1/, year 1 $150,000 $150,000 1.00 12/31, year 1 220,000 200,000 1.10 12/31, year 2 276,000 230,000 1.20Poe's LIFO inventory value at December 31, year 2 is A $230,000 B $236,000 C $241,000 D $246,000
Answer is C
Explanation: Date Layers at Base Year Cost Price Index Ending Inventory at LIFO Cost 01/01, year1 $150,000 1.00 $150,000 12/31, year1 50,000 [1] 1.10 55,000 12/31, year2 30,000 [2] 1.20 36,000 $230,000 $241,000 [1] $200,000 - $150,000 [2] $230,000 - $200,000
The following costs pertain to Den Co.’s purchase of inventory:
700 units of product A $3,750
Freight-in 175
Cost of materials and labor incurred to bring product A to saleable condition 900
Insurance cost during transit of purchased goods 100
Total $4,925
What amount should Den record as the cost of inventory as a result of this purchase?
A $3,925 B $4,650 C $4,825 D $4,925
Answer is D
Explanation:
The cost of merchandise inventory is net of any discounts but includes freight-in, taxes, insurance while in transit, warehousing costs, and similar charges paid to bring the article to its existing condition and loca¬tion. The cost of the inventory would be the $3,750 purchase price, plus the $175 freight-in charge, plus the $900 costs of materials and labor to bring it to saleable condition, plus the $100 insurance cost during transit for a total $4,925.All costs necessary to bring an item to its salable condition are capitalized to the inventory account. All four listed costs meet this requirement. Until the goods are ready for sale, the cost of all efforts to achieve that goal are treated as product costs rather than period costs
Option (A) is incorrect because it excludes costs incurred to sell the inventory and insurance charges.
Option (B) is incorrect because it excludes the insurance cost incurred and freight in charges.
Option (C) is incorrect because it excludes the insurance cost incurred.
In a period of rising general price levels, Pollard Corp. discloses income on a current cost basis in accordance with standards on financial reporting and changing prices. Compared to historical cost income from continuing operations, which of the following conditions increases Pollard’s current cost income from continuing operations?
A
Current cost of equipment is greater than historical cost.
B
Current cost of land is greater than historical cost.
C
Current cost of cost of goods sold is less than historical cost.
D
Ending net monetary assets are less than beginning net monetary assets.
Answer is C
Explanation:
When current cost of goods sold is less than historical cost, current cost income from continuing operations will be greater than historical cost income from continuing operations.
During periods of inflation, a perpetual inventory system would result in the same dollar amount of ending inventory as a periodic inventory system under which of the following inventory valuation methods? FIFO LIFO A Yes No B Yes Yes C No Yes D No No
Answer is A
Explanation:
Under the FIFO cost-flow method, a perpetual system would result in the same dollar amount of ending inventory as a periodic inventory system. Under the LIFO cost-flow method, however, a perpetual system would generally not result in the same dollar amount of ending inventory as a periodic inventory system.
Southgate Co. paid the in-transit insurance premium for consignment goods shipped to Hendon Co., the consignee. In addition, Southgate advanced part of the commissions that will be due when Hendon sells the goods. Should Southgate include the in-transit insurance premium and the advanced commissions in inventory costs? Insurance premium Advanced commissions A Yes Yes B No No C Yes No D No Yes
Answer is C
Explanation:
The in-transit insurance premium should be included in inventory costs because it is a cost necessary to get the goods in the place and condition for their intended sale. The advanced commissions represent a prepaid selling expense and thus should not be included in inventory cost.
An entity that uses the gross margin method of estimating inventory should do which of the following?
A
Assume that its gross margin percentage is relatively stable
B
Determine cost of goods sold by applying the gross margin ratio to sales and subtracting this amount from the sales figure
C
Must understand that the gross margin method is not considered as GAAP for annual financial reporting purposes
D
All of the above
Answer is D
Explanation:
The gross margin method rests on the assumption that the gross margin percentage is relatively stable, computes cost of goods sold by applying the gross margin ratio to sales and subtracting this amount from the sales figure, and is not generally accepted for annual financial reporting purposes. As the answers for A., B., and C. are all correct, the best answer for this question is D., all of the above.
Ashe Co. recorded the following data pertaining to raw material X during January:
Units
Date Received Cost Issued On Hand
1/01 Inventory $8.00 3,200
1/11 Issue 1,600 1,600
1/22 Purchase 4,800 9.60 6,400
The moving-average unit cost of X inventory at January 31 is
A $8.80 B $8.96 C $9.20 D $9.60
Answer is C
Explanation:
$9.2. Units beginning inventory remaining at year-end (3,200 - 1,600)$8 = $12,800 plus 1/22 purchase: 4,800($9.60) = 46,080. Ending inventory $58,880. Ending unit cost: $58,880/6,400 = $9.20
The moving average method costs issues at the unit cost of goods on hand at that point. Thus, the issue was costed at $8.00 per unit. The cost per unit changes with each purchase.
Option (A), (B) and (D) are incorrect as per above explanation.
The following information pertains to an inventory item: Cost $12.00 Estimated selling price 13.60 Estimated disposal cost 0.20 Normal gross margin 2.20 Replacement 10.90Under the lower-of-cost-or-market rule, this inventory item should be valued at A $10.70 B $10.90 C $11.20 D $12.00
Answer is C
Explanation:
Under the lower of cost or market rule, replacement cost cannot exceed a ‘ceiling’ of net realizable value (estimated selling price less estimated cost of disposal) and cannot be below a ‘floor’ of net realizable value reduced by a normal profit margin.
Replacement Cost Parameters Cost Replacement Cost Ceiling (NRV) Floor (NRV - NP) $12.00 $10.90 $13.40 [1] $11.20 [2] [1] $13.60 - $0.20. [2] $13.40 - $2.20.
The replacement cost is below the prescribed range; therefore, the ‘floor’ of $11.20 is the assigned market value. Because the assigned market value of $11.20 is below the historical cost of $12.00, the inventory item should be valued at $11.20.
Nomar Co. shipped inventory on consignment to Seabright Co. that cost $20,000. Seabright paid $500 for advertising that was reimbursable from Nomar. At the end of the year, 70% of the inventory was sold for $30,000. The agreement states that a commission of 20% will be provided to Seabright for all sales.
What amount of net inventory on consignment remains on the balance sheet for the first year for Nomar?
A $0 B $ 6,000 C $ 6,500 D $20,000
Answer is B
Explanation:
Nomar Co., is the consignor and the inventory is never owned by consignee. Title remains with consignor until goods are sold by the consignee. Thus, title passes directly to the third-party buyer at point of sale. Consignee acts as an agent with a commission on sales and reimbursable expenses. At the year end, Nomar co., would report the unsold inventory of $6,000 (i.e. 30% x $20,000).
Option (A) is incorrect because the ownership of the goods is with Nomar Co., and unsold inventory at year end should be reported.
Option (C) is incorrect because advertising expenses of $500 are included in the inventory cost [i.e. $6,500 = (30% x $20,000) + $500].
Option (D) is incorrect because total cost of inventory is reported at the year end which includes the inventory that is sold to third party.
Medus Industries is a manufacturer of pens. Many shops acquire these pens from Medus for their customers. Medus ships 100 pens to a shop on 1/1/Y1 with full payment due immediately. Legal title however, does not transfer and Medus retains the right to require shipment of any unsold pens from this shop to other shops that require these pens. If this right is exercised a full refund would be provided. Similarly, this shop may also return any unsold pens to Medus for a full refund. How should the transaction as of 1/1/Y1 be recognized?
A Sale. B Consignment arrangement. C Repurchase agreement put option. D Bill and hold arrangement.
Answer is B
Explanation:
Medus does not transfer legal title to the shop, continues to hold control as it can demand transfer of the pens to another shop, till the pens are sold to customers by the shop. Also the shop, even though owes full payment for the pens immediately upon transfer from Medus, it is considered more of a deposit, as the Medus would refund the full amount upon return of the pens back to Medus. All the above indicators are satisfied. This is a consignment arrangement.
Nest Co. recorded the following inventory information during the month of January:
Units Unit cost Total cost Units on hand
Balance on 1/1 2,000 $1 $2,000 2,000
Purchased on 1/8 1,200 3 3,600 3,200
Sold on 1/23 1,800 1,400
Purchased on 1/28 800 5 4,000 2,200
Nest uses the LIFO method to cost inventory. What amount should Nest report as inventory on January 31 under each of the following methods of recording inventory?
Perpetual Periodic A $2,600 $5,400 B $5,400 $2,600 C $2,600 $2,600 D $5,400 $5,400
Answer is B
Explanation:
Under perpetual method, inventory is quantified perpetually on an ongoing basis. Unders perpetual system, each time a sale is made, the LIFO method is used to calculate cost of goods sold. When the sale is made on 1/23, the LIFO is applied to associate COGS to 1,200 units at $3 and the remaining 600 units @ $1. The remaining would be 1,400 units at $1. To this add the purchase of 800 units @$5 will be added to calculate the ending inventory. Ending inventory under LIFO perpetual would be:
1,400 units at $1.00 $1,400
800 units at $5.00 $4,000
Total $5,400
Under the periodic inventory system, inventory quantity determined periodically by physical count usually at year / month end. Under the periodic system of LIFO, the COGS of the sale of 1,800 units is considered only at month end when LIFO is applied first to the last purchase made on 1/28. Of the 1,800 units sold, 800 would be associated to $5 per unit and the remaining 1,000 units would be associated to $3 per unit. Ending inventory would consist of the remaining 200 units @ $3 per unit and 2000 units @ $1 per unit. Ending inventory under LIFO periodic would be:
2,000 units at $1.00 $2,000
200 units at $3.00 $600
Total $2,600
Options (A), (C) and (D) are incorrect based on the above explanation.
A manufacturer whose finished goods inventories are items that are routinely manufactured or otherwise produced in large quantities, on a repetitive basis, should include ______________________ in its cost of finished goods inventory.
A Manufacturing overhead B Interest cost C Freight-out expense D Trade discounts
Answer is A
Explanation:
The finished goods inventory of a manufacturer must include the cost of both variable and fixed manufacturing overhead. Interest cost should not be capitalized for inventories that are routinely manufactured or otherwise produced in large quantities on a repetitive basis. Freight-out is a selling expenses and should not be included in the cost of inventory. A trade discount is a deduction from the list or catalog price of merchandise to arrive at the gross selling price. Trade discounts are not recorded in the seller or purchaser’s accounting records.
Options (B), (C) and (D) are incorrect as per above explanation.
A corporation entered into a purchase commitment to buy inventory. At the end of the accounting period, the current market value of the inventory was less than the fixed purchase price, by a material amount. Which of the following accounting treatments is most appropriate?
A
Describe the nature of the contract in a note to the financial statements, recognize a loss in the income statement, and recognize a liability for the accrued loss
B
Describe the nature of the contract and the estimated amount of the loss in a note to the financial statements, but do not recognize a loss in the income statement
C
Describe the nature of the contract in a note to the financial statements, recognize a loss in the income statement, and recognize a reduction in inventory equal to the amount of the loss by use of a valuation account
D
Neither describe the purchase obligation, nor recognize a loss on the income statement or balance sheet
Answer is A
Explanation:
The correct answer is (A). In a legal non-cancelable agreement for future purchase of Inventory, if Contract price > Market price and it is expected that loss will occur on purchase, recognize loss (Market Price - Contracted Price) at the time of decline in prices.
The journal entry would be:
Dr: Est. loss on purchase commitment.
Cr: Accrued loss on purchase commitment.
Details of the losses to be included in the footnote.
Option (B) is incorrect, because apart from describing the nature of the loss in footnotes, loss needs to be accrued and recognize loss in I/S.
Option (C) is incorrect because loss is accrued as a liability and not as a valuation account to reduce inventory.
Option (D) is incorrect because footnote disclosure, loss recognized in I/S and a liability for accrued loss.
In its financial statements, Hila Co. discloses supplemental information on the effects of changing prices in accordance with FASB Standards. Hila computed the increase in current cost of inventory as follows:
Increase in current cost (nominal dollars) $15,000
Increase in current cost (constant dollars) 12,000What amount should Hila disclose as the inflation component of the increase in current cost of inventories?
A
$ 3,000
B
$12,000
C
$15,000
D
$27,000
Answer is A
Explanation:
The “inflation component” of the increase in the current cost amount is defined as the difference between the nominal dollars and constant dollars measures. $15,000 - $12,000 = $3,000.
In January, Stitch, Inc. adopted the dollar-value LIFO method of inventory valuation. At adoption, inventory was valued at $50,000. During the year, inventory increased $30,000 using base-year prices, and prices increased 10%. The designated market value of Stitch’s inventory exceeded its cost at year end. What amount of inventory should Stitch report in its year-end balance sheet?
A $80,000 B $83,000 C $85,000 D $88,000
Answer is B
Explanation:
Inventory is reported at the lower of cost or market. As indicated in the problem, market exceeded cost, so inventory should be stated at cost, using dollar value LIFO. The inventory layer added in the current year is computed in terms of base year cost. It then must be converted to current year cost because the layer was added during the current year. The index (1.1) is computed by dividing the ending inventory at current year cost ($88,000) by the ending inventory at base year cost ($80,000). The cost of ending inventory ($83,000) is the $50,000 cost of the beginning amount and the converted Year 1 layer. ($30.000 × 1.1 = $33,000).
Option (A) is incorrect because the calculation does not adjust for inflation 10% on current year inventory.
Option (C) is incorrect because it adjusts the price index @ 10% on prior period inventory of $50,000.
Option (D) is incorrect because it adjusts the price index of 10% on prior period and current year’s inventory
Jones Wholesalers stocks a changing variety of products. Which inventory costing method will be most likely to give Jones the lowest ending inventory when its product lines are subject to specific price increases?
A Specific identification. B Weighted average. C Dollar-value LIFO. D FIFO periodic.
Answer is C
Explanation:
The dollar value LIFO will give Jones the lowest ending inventory when its product lines are subject to specific price increases. This is because LIFO methods operate under the assumption that the last item of inventory purchased is the first one to be sold. Thus, in the case of rising prices, the cost of the most recently acquired inventory will always be higher than the cost of earlier purchases, so the ending inventory balance will be valued at earlier costs which are lower, while the most recent costs appear in the cost of goods sold. As far as specific identification, weighted average or FIFO periodic methods, none of these would give a higher inventory value than dollar value LIFO method.
Options (A), (B) and (D) are incorrect based on the above explanation.
Delar Co. completed its year-end physical count of inventory. The inventory was valued at first-in, first-out (FIFO) costs and totaled $500,000.Delar subsequently noted the following two items:
1,000 units of inventory with a FIFO cost of $10 each were shipped and billed to a customer F.O.B.destination. These items were included in the physical count.
6,000 units at a FIFO cost of $5 each were held on consignment for one of its suppliers but were excluded from the physical count.
What amount should Delar report as inventory at year end?
A $530,000 B $520,000 C $500,000 D $490,000
Explanation:
The correct answer is (C).
Delar should report $500,000 as inventory at year-end.
FOB destination means title and risk of loss pass to the buyer when the seller makes a proper tender of delivery of the goods at the destination. This was shipped via FOB destination and should be included in inventory because the inventory had not been delivered to the customer by year-end.
Consigned goods are not sold but rather transferred to an agent for possible sale. Consigned goods are included in the inventory of the consignor (owner). Thus, the inventory that is held on consignment for one of the suppliers should be excluded from the inventory physical count as Delar is a consignee. Therefore, Delar should report $500,000 of inventory at year-end.
A company determined the following values for its inventory as of the end of its fiscal year: Historical cost $100,000 Current replacement cost 70,000 Net realizable value 90,000 Net realizable value less a normal profit margin 85,000 Fair value 95,000Under IFRS, what amount should the company report as inventory on its balance sheet? A $70,000 B $85,000 C $90,000 D $95,000
Answer is C
Explanation:
Under IFRS, inventory is carried at the lower of cost or net realizable value (best estimate of the net amounts inventories are expected to realize). This amount may or may not equal fair value. The net realizable value of $90,000 is lower than the historical cost of $100,000.
The following information pertains to each unit of merchandise purchased for resale by Vend Co.: March 1 December 31 Purchase price $ 8 Selling price 12 $ 15 Price level index 110 121 Replacement cost 10Under current cost accounting, what is the amount of Vend's holding gain on each unit of this merchandise? A $0 B $0.80 C $1.20 D $2.00
ANswer is D
Explanation:
Under current cost accounting, the amount of holding gain on a unit of inventory is the increase in current cost from holding the inventory from period to period. The inventory in question was purchased at $8 per unit on March 1 and has a replacement cost of $10 on December 31. Thus, under current cost accounting, Vend has a holding gain of $2 (i.e., $10 - $8) on each unit of the inventory.
Manhof Co. prepares supplementary reports on income from continuing operations on a current cost basis in accordance with FASB Standards. How should Manhof compute cost of goods sold on a current cost basis?
A
Number of units sold times average current cost of units during the year.
B
Number of units sold times current cost of units at year end.
C
Number of units sold times current cost of units at the beginning of the year.
D
Beginning inventory at current cost plus cost of goods purchased less ending inventory at current cost.
Answer is A
Explanation:
To compute cost of goods sold on a current cost basis, multiply the number of units sold by the average current cost of the units during the year. (Average current cost of the units during the year is the sum of the current cost of the units at the beginning and the end of the year, divided by two.).
A manufacturer has the following per-unit costs and values for its sole product:
Cost $10.00
Current replacement cost 5.50
Net realizable value 6.00
Net realizable value less normal profit margin 5.20
In accordance with IFRS, what is the per-unit carrying value of inventory in the manufacturer’s statement of financial position?
A $ 5.20 B $ 5.50 C $ 6.00 D $10.00
Answer is C
Explanation:
Under IFRS, inventory is carried at the lower of cost or net realizable value (best estimate of the net amounts inventories are expected to realize). This amount may or may not equal fair value. The net realizable value of $6.00 is lower than the historical cost of $10.00.
Bren Co.’s beginning inventory at January 1 was understated by $26,000, and its ending inventory was overstated by $52,000. As a result, Bren’s cost of goods sold for the year was
A Understated by $26,000. B Overstated by $26,000. C Understated by $78,000. D Overstated by $78,000.
Answer is C
Explanation:
Cost of goods sold = Beginning inventory + Purchases - Ending inventory
Beginning inventory was understated by $26,000, the beginning inventory would have been beginning inventory - $26,000.
Ending inventory was overstated by $52,000, the ending inventory would have been ending inventory + $52,000.
The incorrect cost of goods sold would have been Beginning inventory - 26,000 + Purchases - (Ending inventory + 52,000).
Thus cost of goods sold would have been Beginning inventory + Purchases - Ending inventory - $78,000 (i.e. - $26,000 - $52,000).
Cost of goods sold (COGS) would have been understated by $78,000. An alternative explanation: understatement of beginning inventory would lead to an understatement of COGS. Overstatement of ending inventory would lead to an understatement of COGS, leading to a net understatement of $78,000.
Options (A), (B) and (D) are incorrect based on the above explanation.
Based on a physical inventory taken on December 31, Chewy Co. determined its chocolate inventory on a FIFO basis at $26,000 with a replacement cost of $20,000. Chewy estimated that, after further processing costs of $12,000, the chocolate could be sold as finished candy bars for $40,000. Chewy’s normal profit margin is 10% of sales. Under the lower of cost or market rule, what amount should Chewy report as chocolate inventory in its December 31 balance sheet?
A $28,000 B $26,000 C $24,000 D $20,000
Explanation:
The correct answer is (B).
Inventory is valued at the Lower of Cost or Market (LCM) if LIFO or retail inventory is used. Exception: FASB has issued an update replacing the LCM valuation with lower of cost or net realizable value (LCNRV) valuation for other than LIFO or retail inventory methods.
Chewy Co. determined its chocolate inventory on a FIFO basis. Inventory should be valued at LCNRV.
Cost = $26,000.
NRV = Net selling price - Costs to complete and dispose = $40,000 - $12,000 = $28,000.
Lower of the two: inventory is $26,000.
Which of the following statements regarding inventory accounting systems is true?
A
A disadvantage of the perpetual inventory system is that the inventory dollar amounts used for interim reporting purposes are estimated amounts.
B
A disadvantage of the periodic inventory system is that the cost of goods sold amount used for financial reporting purposes includes both the cost of inventory sold and inventory shortages.
C
An advantage of the perpetual inventory system is that the record keeping required to maintain the system is relatively simple.
D
An advantage of the periodic inventory system is that it provides a continuous record of the inventory balance.
Explanation:
The correct answer is (B).
A disadvantage of the periodic inventory system is that the cost of goods sold amount used for financial reporting purposes includes both the cost of inventory sold and inventory shortages.
The perpetual inventory system is that the inventory dollar amounts used for interim reporting purposes are actual amounts.
The disadvantage of the perpetual inventory system is that the record keeping required to maintain the system is relatively complex and time-consuming.
The disadvantage of the periodic inventory system is that it provides only a period-end record of the inventory balance
The replacement cost of an inventory item is below the net realizable value and above the net realizable value less the normal profit margin. The original cost of the inventory item is above the replacement cost and below the net realizable value. As a result, under the lower-of-cost-or-market method, the inventory item should be valued at the A Original cost B Replacement cost C Net realizable value D Net realizable value less the normal profit margin
Answer is B
Explanation:
Valuation of inventory items is required at the lower of cost or replacement cost (commonly referred to as market). Market cannot exceed the net realizable value (ceiling) of the good (i.e., selling price less expected costs to sell), and market should not be less than this net realizable value reduced by an allowance for a normal profit margin (floor). In this problem, the replacement cost is between the ceiling and floor amounts, so it is used as the market value. Since the original cost is greater than replacement (i.e., market) cost, the item will be carried at the lower market/replacement cost.
At December 31 of the current year, the following information was available from Huff Co.’s accounting records:
Cost Retail
Inventory, 1/1 $147,000 $ 203,000
Purchases 833,000 1,155,000
Additional markups – 42,000
Available for sale $980,000 $1,400,000
Sales for the year totaled $1,106,000. Markdowns amounted to $14,000. Under the approximate lower of average cost or market retail method, Huff’s inventory at December 31 was
A $308,000 B $280,000 C $215,600 D $196,000
Answer is D
Explanation:
Under the approximate lower of average cost or market retail method, the estimated inventory is computed as follows:
Cost Retail
Inventory, 1/1 $147,000 $203,000
Purchases 833,000 1,155,000
Additional markups – 42,000
Cost-to-retail ratio amounts $980,000 $1,400,000
Less: Sales – (1,106,000)
Markdowns – (14,000)
Estimated ending inventory at retail – $280,000
Times cost-to-retail ratio ($980,000 / $1,400,000)= 0.7 i.e 70% – x 70%
Estimated ending inventory at cost – $196,000
An entity purchased new machinery from a supplier before the entity’s year end. The entity paid freight charges for the purchased machinery. The entity took out a loan from a bank to finance the purchase. Under IFRS, what is the proper accounting treatment for the freight and interest costs related to the machinery purchase?
A
The freight and interest costs should be immediately expensed.
B
The freight and interest costs should be capitalized as part of property, plant and equipment.
C
The interest cost should be capitalized as part of property, plant and equipment, and the freight cost should be immediately expensed.
D
The freight cost should be capitalized as part of property, plant and equipment, and the interest cost should be immediately expensed.
Answer is D
Explanation:
Under both IFRS and US GAAP, freight cost is capitalized as part of property, plant and equipment, and interest is only capitalized on qualifying assets. Assets qualifying for interest capitalization include assets constructed or produced for self-use on a repetitive basis, assets acquired for self-use through arrangements requiring down payments or progress payments, and assets constructed or produced as discrete projects for sale or lease (e.g., ships or real estate developments). Assets not qualifying for interest capitalization include inventories that are routinely manufactured on a repetitive basis, assets in use or ready for use, and assets not in use and not being prepared for use. Without any indication otherwise, it would be determined that this machinery is ready for use and as such the interest costs would not be capitalized but instead would be immediately expensed.
Walt Co. adopted the dollar-value LIFO inventory method as of January 1 of the current year, when its inventory was valued at $500,000. Walt's entire inventory constitutes a single pool. Using a relevant price index of 1.10, Walt determined that its December 31 inventory was $577,500 at current year cost, and $525,000 at base year cost. What was Walt's dollar-value LIFO inventory at December 31 of the current year? A $525,000 B $527,500 C $552,500 D $577,500
Answer is B
Explanation:
The current year layer is determined by the difference of the ending inventory at base year cost and the beginning inventory (which is the base year in this case) at base year cost and applying the relevant price index to the difference.
Ending inventory at base year cost $ 525,000
Less: Beginning inventory at base year cost (500,000)
Current year layer at base year cost $ 25,000
Relevant price index 1.10
Current year layer at current year prices $ 27,500
Beginning inventory (1/1) at DV LIFO 500,000
Dollar-value LIFO inventory, 12/31 $ 527,500
Garson Co. recorded goods in transit purchased F.O.B. shipping point at year end as purchases. The goods were excluded from ending inventory. What effect does the omission have on Garson’s assets and retained earnings at year end?
Assets Retained earnings A No effect Overstated B No effect Understated C Understated No effect D Understated Understated
Answer is D
Explanation:
Goods purchased f.o.b. shipping point should be included in inventory when the goods are shipped. It states the goods are in transit, so they have been shipped. The goods were erroneously excluded from ending inventory, when they should have been included, meaning that inventory is understated. Inventory is an asset, so assets are understated. An understated inventory also would lead to an overstated cost of goods sold (goods available for sale – ending inventory = cost of goods sold). An overstated cost of goods sold would lead to an understated gross profit (revenue – cost of goods sold = gross profit) which in turn results in an understated retained earnings.
Options (A), (B) and (C) are incorrect as per above explanation.
The following information relates to a company’s year-end inventory:
Inventory cost $910
Selling price of inventory $1,000
Normal profit margin 10% of selling price
Current replacement cost $740
Cost of completion and disposal $100
Under IFRS, what is the company’s year-end inventory balance?
A $740 B $800 C $900 D $910
Answer is C
Explanation:
The correct answer is (C).
Under IFRS, inventories are valued at lower of cost or net realizable value.
Net realizable value = Fair value - Estimated sales cost = $1,000 - $100 = $900.
As the NRV is lower at $900 when compared to the cost of $910, inventory will be valued at $900.
A firm’s ending inventory balance was overstated by $1,000. Which of the following statements is correct according to a periodic inventory system?
A
The retained earnings were overstated by $1,000.
B
The cost of goods sold was overstated by $1,000.
C
The cost of goods available for sale was overstated by $1,000.
D
The gross margin was understated by $1,000.
Explanation:
The correct answer is (A).
Cost of goods sold (COGS) is calculated as beginning inventory plus purchases during the period minus ending inventory.
Therefore, a $1,000 overstatement of the ending inventory balance will result in a $1,000 understatement of cost of goods sold.
The $1,000 understatement of COGS results in a $1,000 overstatement of gross profit as well, which carries over to a $1,000 overstatement of retained earnings.
The dollar-value LIFO inventory cost flow method involves computations based on
Inventory pools of similar items A specific price index for each year
A No Yes
B No No
C Yes No
D Yes Yes
Answer is D
Explanation:
The dollar-value LIFO method is based on the aggregation of similar inventory items into pools. Acquisitions and issuances of similar materials are recorded in the same pool, even if the substitute items are not exactly the same as the replaced items. As a general rule, dollar-value LIFO uses a “double-extension method” to determine: (1) the value of the ending inventory in terms of base year prices, and (2) the value of the ending inventory at current prices. The ratio of (2) over (1), above, provides the specific price index for valuing any layers of inventory added in the period.
When the double extension approach to the dollar value LIFO inventory method is used, the inventory layer added in the current year is multiplied by an index number. Which of the following correctly states how components are used in the calculation of this index number?
A
In the numerator, the average of the ending inventory at base year cost and at current year cost.
B
In the numerator, the ending inventory at current year cost, and, in the denominator, the ending inventory at base year cost.
C
In the numerator, the ending inventory at base year cost, and, in the denominator, the ending inventory at current year cost.
D
In the denominator, the average of the ending inventory at base year cost and at current year cost.
Answer is B
Explanation:
The inventory layer added in the current year is computed in terms of base year cost. It then must be converted to current year cost because the layer was added during the current year. Because we are converting to current year cost from base year cost, the index is computed by dividing the ending inventory at current year cost by the ending inventory at base year cost
The replacement cost of an inventory item is below the net realizable value and above the net realizable value less a normal profit margin. The inventory item’s original cost is above the net realizable value. Under the lower of cost or market method, the inventory item should be valued at
A Original cost B Replacement cost C Net realizable value D Net realizable value less normal profit margin
Answer is B
Explanation:
Under U.S. GAAP, inventory is valued at the Lower of Cost or Market (LCM) if LIFO or retail inventory method is used, where Cost = Original cost of inventory and Market = middle for the following three numbers:- Net realizable Value (NRV). NRV - normal profit margin. Replacement cost. Exception: FASB has issued an update replacing the LCM valuation with lower of cost of net realizable value (LCNRV) valuation for other than LIFO or retail inventory methods. As per LCM, in the given case replacement cost is below the net realizable value and above the net realizable value less a normal profit margin, replacement cost is in middle and is the market. Cost is given to be higher than NRV, and would be higher than replacement cost or the market. Thus, the inventory will be valued at lower of cost or market, in this case, market or replacement cost.
Options (A), (C) and (D) are incorrect based on the above explanation.
Carver Co., a retailer, uses the perpetual inventory method. Carver uses the moving average method to determine the value of its inventory. The following information relates to inventory transactions that took place during the month of March:
3/1 Beginning inventory 30,000 units at $10
3/5 Purchase 10,000 units at $12
3/10 Sales at $20 per unit 20,000 units
3/20 Purchase 20,000 units at $13
What amount should Carver report as cost of goods sold on its income statement at the end of March?
A $200,000 B $210,000 C $240,000 D $260,000
Answer is B
Explanation:
The correct answer is (B)
Under the perpetual inventory method, COGS is updated for every inventory sale according to the valuation method followed by an entity. In the given case, Carver Co. follows a perpetual inventory method and uses the moving average method to determine the inventory’s value. The cost of goods sold on the 10th of March can be calculated as below:
Date Particulars Units and unit cost Inventory (A) Inventory Units (B) Moving Avg. Cost (A/B)
3/1 Beginning 30000@ $10 = $300,000 300,000 30,000 10
3/5 Purchase 10,000@ $12 = $120,000 420,000 40,000 10.5
3/10 Sales 20,000@ $10.5 = $210,000 210,000 20,000 10.5
The 20,000 units sold on 3/10 cost $10.5 per unit and therefore, the cost of goods sold is $210,000. Note: Purchases made on 3/20 is irrelevant to compute the COGS under the perpetual inventory method since it is after the sales date. However, if the entity was using the periodic inventory method, the purchase would have been taken into account.
In a \_\_\_\_\_\_\_\_\_\_\_\_\_\_\_\_, a customer agrees to purchase the goods, but the seller retains physical possession until the customer requests shipment to designated locations. A Side agreement B Bill and hold transaction C Related-party transaction D Trade loading transaction
Answer is B
Explanation:
In a “bill and hold” transaction, a customer agrees to purchase the goods, but the seller retains physical possession until the customer requests shipment to designated locations.
The original cost of an inventory item is below the net realizable value and above the net realizable value less a normal profit margin. The inventory item's replacement cost is below the net realizable value less a normal profit margin. Under the lower of cost or market method, the inventory item should be valued at A Original cost. B Replacement cost. C Net realizable value. D Net realizable value less normal profit margin.
Answer is D
Explanation:
Under the lower-of-cost-or-market rules, market means current replacement cost with the exception that market value should not exceed the net realizable value (ceiling) and should not be less than the net realizable value minus normal profit (floor). Because in this question replacement cost is below the floor, this would be the minimum amount at which market could be valued. This market value is then compared to the original cost, and the lower amount is used for the value of the inventory item. The question states that the original cost is above the net realizable value less a normal profit margin, thus, the market value is the lower amount.
Dart Company's accounting records indicated the following information: Inventory, 1/1 $ 500,000 Purchases during the year 2,500,000 Sales during the year 3,200,000A physical inventory taken on December 31 resulted in an ending inventory of $575,000. Dart's gross profit on sales has remained constant at 25% in recent years. Dart suspects some inventory may have been taken by a new employee. At December 31, what is the estimated cost of missing inventory? A $ 25,000 B $100,000 C $175,000 D $225,000
Answer is A
Explanation:
The missing inventory is estimated by determining the difference between the estimated ending inventory using the gross margin method and the actual physical inventory on hand at year-end.
Beginning Inventory $ 500,000 Purchases 2,500,000 Goods available for sale 3,000,000 Less: estimated COGS Sales $3,200,000 Less: gross margin (25% x $3,200,000) 800,000 (2,400,000) Estimated ending inventory 600,000 Less: actual physical inventory (575,000) Estimated cost of missing inventory $ 25,000
Drew Co. uses the average cost inventory method for internal reporting purposes and LIFO for financial statement and income tax reporting. At December 31, the inventory was $375,000 using average cost and $320,000 using LIFO. The unadjusted credit balance in the LIFO Reserve account on December 31 was $35,000. What adjusting entry should Drew record to adjust from average cost to LIFO at December 31?
Debit Credit
A Cost of Goods Sold 55,000 Inventory 55,000
B Cost of Goods Sold 55,000 LIFO Reserve 55,000
C Cost of Goods Sold 20,000 Inventory 20,000
D Cost of Goods Sold 20,000 LIFO Reserve 20,000
Answer is D
Explanation:
Some companies use LIFO for tax and external reporting purposes, but they maintain a FIFO, average cost, or standard cost system for internal reporting purposes. The difference between the inventory method used for internal reporting purposes and LIFO is often referred to as the LIFO reserve.The LIFO reserve is a contra-inventory account that must be adjusted to its required balance at the financial statement date. At 12/31 Drew’s inventory was $375,000 using average cost and $320,000 using LIFO. The required balance in the LIFO reserve at 12/31 is $55,000 (i.e., $375,000 - $320,000). Since the unadjusted LIFO reserve balance was $35,000, the reserve must be increased by $20,000 (i.e., $55,000 - $35,000).
The original cost of an inventory item is above the replacement cost. The inventory item’s replacement cost is above the net realizable value. Under the lower of cost or market method, the inventory item should be valued at
A Original cost B Replacement cost C Net realizable value D Net realizable value less normal profit margin
Answer is C
Explanation:
Valuation of inventory items is required at the lower of cost or replacement cost (commonly referred to as market). Market cannot exceed the net realizable value (ceiling) of the good (i.e., selling price less expected costs to sell), and market should not be less than this net realizable value reduced by an allowance for a normal profit margin (floor). In this problem, the replacement cost exceeds net realizable value, so market is defined as NRV. Since the original cost is greater than defined market, the item will be carried at the lower of market/NRV amount.
Option (A) is incorrect because original cost is higher than market.
Option (B) is incorrect because the market price used should be net realizable value when replacement cost is higher.
Option (D) is incorrect because the relevant information is missing related to net realizable value - normal profit. However, given the fact that the replacement clost is higher than net realizable value the middle of the three will be the net realizable value.
Young Corp. purchased equipment by making a down payment of $4,000 and issuing a note payable for $18,000. A payment of $6,000 is to be made at the end of each year for three years. The applicable rate of interest is 8%. The present value of an ordinary annuity factor for three years at 8% is 2.58, and the present value for the future amount of a single sum of one dollar for three years at 8% is .735. Shipping charges for the equipment were $2,000, and installation charges were $3,500. What is the capitalized cost of the equipment? A $19,480 B $21,480 C $24,980 D $27,500
Answer is C
Explanation:
Assets are to be recorded at their acquisition cost. Acquisition cost is defined as the cash price, or its equivalent, plus all the costs reasonably necessary to bring it to the location and to make it ready for its intended use. The cash price, or its equivalent, would be the $4,000 down payment plus the $15,480 present value of the $6,000 payments, an ordinary annuity, (2.58 × $6,000). Add to that price of $19,480 the $2,000 in shipping charges to get the equipment to its location and the $3,500 of installation charges to get it ready for its intended use. The total capitalized cost of the equipment is $4,000 + $15,480 + $2,000 + $3,500 = $24,980.
Weir Co. uses straight-line depreciation for its property, plant, and equipment, which, stated at cost, consisted of the following: 12/31,Yr2 12/31, Yr1 Land $ 25,000 $ 25,000 Buildings 195,000 195,000 Machinery and equipment 695,000 650,000 915,000 870,000 Less: Accumulated depreciation (400,000) (370,000) $ 515,000 $ 500,000Weir's depreciation expense for year 2 and year 1 was $55,000 and $50,000, respectively. What amount was debited to accumulated depreciation during year 2 because of property, plant, and equipment retirements? A $40,000 B $25,000 C $20,000 D $10,000
Answer is B
Explanation:
Weir’s Accumulated Depreciation account increased by $30,000 ($400,000 - $370,000) during year 2, despite the fact that $55,000 of depreciation expense was recorded during year 2. Therefore, the debit to accumulated depreciation during year 2 because of plant and equipment retirements was $25,000 (i.e., $55,000 - $30,000).
Carr, Inc. purchased equipment for $100,000 on January 1, year 2. The equipment had an estimated 10-year useful life and a $15,000 salvage value. Carr uses the 200% declining balance depreciation method. In its year 3 income statement, what amount should Carr report as depreciation expense for the equipment? A $13,600 B $16,000 C $17,000 D $20,000
ANswer is B
Explanation:
The straight-line (SL) depreciation method is a fixed charge method where an equal amount of depreciable cost is allocated to each period. The SL formula is (historical cost - salvage value) / estimated useful life. The 200% declining balance depreciation method, also known as double-declining balance (DDB), uses a rate of depreciation twice the SL rate applied to the book value (i.e., declining balance) of the asset to obtain the depreciation expense for the period. The salvage value is not used in the calculation except as a lower bound for the asset’s book value. The DDB formula is (2 / estimated useful life) × (historical cost - accumulated depreciation).
year 2: (2/10) × ($100,000 - 0) = $20,000
year 3: (2/10) × ($100,000 - $20,000) = $16,000.
On January 2, Lem Corp. bought machinery under a contract that required a down payment of $10,000, plus 24 monthly payments of $5,000 each, for total cash payments of $130,000. The cash equivalent price of the machinery was $110,000. The machinery has an estimated useful life of ten years and estimated salvage value of $5,000. Lem uses straight-line depreciation. In its year-end income statement, what amount should Lem report as depreciation for this machinery? A $10,500 B $11,000 C $12,500 D $13,000
Answer is A
Explanation:
Assets are recorded at their acquisition cost. Acquisition cost is the cash price, or its equivalent.
[($110,000-$5,000) / 10 years = $10,500 / year.].
Samm Corp. purchased a plot of land for $100,000. The cost to raze a building on the property amounted to $50,000 and Samm received $10,000 from the sale of scrap materials. Samm built a new plant on the site at a total cost of $800,000 including excavation costs of $30,000. What amount should Samm capitalize in its land account? A $150,000 B $140,000 C $130,000 D $100,000
Answer is B
Explanation:
Land must be recorded at its acquisition cost. Generally, acquisition cost is defined as the cash price, or its equivalent, plus all other costs reasonably necessary to make it ready for its intended use. Such additional costs include demolition of an old building, less any scrap proceeds received.
Purchase price of land $100,000
Add: Cost to raze old building 50,000
Less: Proceeds from sale of scrap materials (10,000)
Capitalized value of land $140,000
Ichor Co. reported equipment with an original cost of $379,000 and $344,000, and accumulated depreciation of $153,000 and $128,000, respectively, in its comparative financial statements for the years ended December 31, year 5, and year 4. During year 5, Ichor purchased equipment costing $50,000, and sold equipment with a carrying value of $9,000. What amount should Ichor report as depreciation expense for year 5. A $19,000 B $25,000 C $31,000 D $34,000
Answer is C
Explanation:
Equipment cost, 12/31, year 4 $344,000
Cost of year 5 purchase 50,000
Equipment cost without sale $394,000
Equipment cost, 12/31, year 5 (379,000)
Cost of equipment sold $ 15,000
Carrying value of equipment sold (9,000)
Accumulated depreciation of equipment sold $ 6,000
Accumulated depreciation, 12/31, year 4 $128,000
Accumulated depreciation of equipment sold (6,000)
Balance without year 5 depreciation $122,000
Accumulated depreciation, 12/31, year 5 (153,000)
Depreciation expense year 5 $ 31,000
\_\_\_\_\_\_\_\_\_\_\_\_\_\_\_\_ are costs that extend the service life of the asset, increase its output rate, or lower production costs. A Betterments or improvements B Transfers C Depreciation D Ordinary repairs
Explanation:
A
Costs that extend the service life of the asset, increase its output rate, or lower production costs are betterments or improvements and are capitalized.
A manufacturing firm purchased used equipment for $135,000. The original owners estimated that the residual value of the equipment was $10,000. The carrying amount of the equipment was $120,000 when ownership transferred. The new owners estimate that the expected remaining useful life of the equipment was 10 years, with a salvage value of $15,000. What amount represents the depreciable base used by the new owners? A $105,000 B $110,000 C $120,000 D $125,000
Explanation:
C
The asset would be recorded at $135,000, its acquisition cost to the new owners. The salvage value would be the $15,000 estimated by the new owners. The carrying amount and residual value of the previous owners does not matter to the new owners. The depreciable base would be $120,000; the $135,000 acquisition cost less the $15,000 estimated salvage value.
Gold Co. purchased equipment from Marshall Co. on July 1. Gold paid Marshall $10,000 cash and signed a $100,000 noninterest-bearing note payable, due in three years. Gold recorded a $24,868 discount on notes payable related to this transaction. What is the acquired cost of the equipment on July 1? A $75,132 B $85,132 C $100,000 D $110,000
Explanation:
Equipment purchased by the issuance of securities is recorded at its fair value or the fair value of the securities issued, whichever is more clearly determinable. The fair value of the note would be $75,132 ($100,000 less the $24,868 discount). Gold also paid $10,000 cash that would need to be included in the fair value of the equipment. $75,132 + $10,000 = $85,132.
On December 1, of the current year, Clark Co. leased office space for five years at a monthly rental of $60,000. On the same date, Clark paid the lessor the following amounts: First months' rent $ 60,000 Last months' rent 60,000 Security deposit (refundable at lease expiration) 80,000 Installation of new walls and offices 360,000What should be Clark's current year expense relating to utilization of the office space? A $60,000 B $66,000 C $120,000 D $140,000
Explanation:
The installation of new walls and offices are leasehold improvements since they are not separable from the leased property and revert to the lessor at the end of the lease term. They are amortized over the lease term. The prepayment of the last month’s rent was made to secure the lease and should be reported as a leasehold within intangible assets. Current year expense is $66,000 ($60,000 rent for December and $6,000 amortization of leasehold improvements [$360,000/60]).
Net income is understated if, in the first year, estimated salvage value is excluded from the depreciation computation when using the
Straight-line method Production or use method
A Yes No
B Yes Yes
C No No
D No Yes
Answer is B
Explanation:
An estimated salvage value is directly incorporated into the formulas for computing annual depreciation expense under the straight-line and productive output methods.
Straight-line depreciation expense = Historical cost - Estimated salvage value
Estimated useful life in time
Productive output depreciation expense = Historical cost - Estimated salvage value x current output
Estimated productive outputTherefore, if estimated salvage value is excluded from either depreciation computation, the depreciable base (i.e., historical cost minus estimated salvage value) would be overstated. Thus, in the first year of the life of the asset, depreciation expense would be overstated and net income would be understated under both methods.
Which of the following conditions must exist in order for an impairment loss to be recognized?
The carrying amount of the long-lived asset is less than its fair value. The carrying amount of the long-lived asset is not recoverable. A I only B II only C Both I and II D Neither I nor II
ANswer is B
Explanation:
Recognition of an impairment loss is required only if a long-term asset’s, or asset group’s, carrying amount is not recoverable and exceeds its fair value.
Restorations of carrying value for long-lived assets are permitted if an asset's fair value increases subsequent to recording an impairment loss for which of the following? Held for use Held for disposal A Yes Yes B Yes No C No Yes D No No
Answer is C
Explanation:
Impaired assets are divided into three categories: held for use, held for disposal by sale, and held for disposal other than by sale. For assets held for use, any subsequent reversal of a previously recognized impairment loss is prohibited. For assets held for disposal by sale, the asset is measured at the lower of its book or fair value less cost to sell and its depreciation (or amortization) discontinues. Increases in the fair value, up to but not exceeding book value, would be recognized.
Last year, Katt Co. reduced the carrying amount of its long-lived assets used in operations from $120,000 to $100,000, in connection with its annual impairment review. During the current year, Katt determined that the fair value of the same assets had increased to $130,000. What amount should Katt record as restoration of a previously recognized impairment loss in the current year’s financial statements?
A $0 B $10,000 C $20,000 D $30,000
Explanation:
The correct answer is (A).
The subsequent reversal or restoration of a previously recognized impairment loss is prohibited under GAAP.
A company issued a purchase order on December 15, year 1, for a piece of capital equipment that costs $100,000. The capital equipment was shipped from the vendor on December 31, year 1, and received by the company on January 5, year 2. The equipment was installed and placed in service on February 1, year 2. On what date should the depreciation expense begin?
A December 15, year 1 B December 31, year 1 C January 5, year 2 D February 1, year 2
Explanation:
The correct answer is (D).
Depreciation allocates the total cost of an asset over the periods that derive benefit from the asset. This is in line with the matching principle whereby firms recognize revenues and their related expenses in the same accounting period. The equipment was installed and placed in service on February 1, year 2. It is only from this date that the benefit from the asset can be derived and depreciation will also begin from February 1, year 2.
Purchase Order, Shipping and Receiving are irrelevant to depreciation for this piece of equipment.
On January 1, year 1, a company purchased equipment for $100 million. The equipment consists of four major components, of which two components comprise 80% of the total cost and each has a 20-year useful life. The remaining two components have costs of $10 million each; one of them has a useful life of four years, and the other has a useful life of five years. The company applies the cost model to the equipment and uses the straight-line method of depreciation. Under IFRS, what is the depreciation expense for the year ended December 31, year 1?
A $4,000,000 B $5,000,000 C $8,000,000 D $8,500,000
Explanation:
The correct answer is (D)
Under IFRS, if a tangible asset has significant identifiable components with different values and estimated lives, they should record and depreciated separately.
Hence, depreciation can be calculated as under:
Components Value (in $ million) Useful life (years) Depreciation
Two components comprising 80% of total value 80 20 4
Component 3 10 4 2.5
Component 4 10 5 2
Total Depreciation 8.5
An entity, upon initial recognition of an asset retirement obligation, should not take which of the following actions?
A
Allocate asset retirement cost to expense over the useful life of the related asset
B
Measure the asset retirement cost at fair value
C
Capitalize the asset retirement cost by increasing the carrying amount of the related asset
D
Capitalize the asset retirement cost at its undiscounted cash flow value
Explanation:
The correct answer is (D).
An Asset Retirement Obligation (ARO) involves the retirement of a tangible, long-lived asset that depends on a future event beyond the control of an obligated party. An ARO is an essential part of producing fair and accurate financial statements. So the entity should not capitalize the asset retirement cost at its undiscounted cash flow value. Initial recognition of liability is at fair value (FV) or present value (PV) of expected future restoration cost using adjusted risk-free rate.
(A), (B) and (C) are incorrect because these are the legal obligations associated with the retirement of long-lived assets.
Which of the following is not a true statement regarding the accounting and reporting requirements related to property, plant, and equipment under both IFRS and US GAAP?
A
Both IFRS and US GAAP require a provision for asset retirement costs to be recorded when there is a legal obligation.
B
Changes in depreciation method, residual value, and useful economic life are treated as a change in accounting estimate requiring prospective treatment under US GAAP and retrospective application under IFRS.
C
Under US GAAP eligible borrowing costs do not include exchange rate differences and under IFRS eligible borrowing costs include exchange rate differences from foreign currency borrowings.
D
Under US GAAP investment property is not separately defined and, therefore, is accounted for as held for use or held for sale whereas under IFRS investment property is separately defined.
Answer is B
Explanation:
Changes in depreciation method, residual value, and useful economic life are treated as a change in accounting estimate requiring prospective treatment under both US GAAP and IFRS. Both IFRS and US GAAP require a provision for asset retirement costs to be recorded when there is a legal obligation. Under US GAAP, eligible borrowing costs do not include exchange rate differences and under IFRS eligible borrowing costs include exchange rate differences from foreign currency borrowings. Under US GAAP, investment property is not separately defined and, therefore, is accounted for as held for use or held for sale whereas under IFRS investment property is separately defined.
During the year, Bay Co. constructed machinery for its own use and for sale to customers. Bank loans financed these assets both during construction and after construction was complete. How much of the interest incurred should be reported as interest expense in the year-end income statement?
Interest incurred for machinery for own use Interest incurred for machinery held for sale
A All interest incurred All interest incurred
B All interest incurred Interest incurred after completion
C Interest incurred after completion Interest incurred after completion
D Interest incurred after completion All interest incurred
Answer is D
Explanation:
Interest cost during construction is to be capitalized on assets constructed for an enterprise’s own use (including assets constructed by others for which progress payments have been made), or assets intended for sale or lease that are constructed as discrete projects. However, interest cost shall not be capitalized for inventories that are routinely manufactured or otherwise produced in large quantities on a repetitive basis. Once the constructed assets are completed, all interest should be expensed as incurred. In the question at hand, it is not clear, but it is assumed that the machinery constructed for sale is routinely manufactured in large quantities on a repetitive basis. Therefore, the interest during construction should be capitalized only for the equipment constructed for Bay’s own use. The interest incurred during construction of the inventory items should be expensed. The interest incurred after completion should be expensed on both groups of equipment.
The graph below depicts three depreciation expense patterns over time.
Which depreciation expense pattern corresponds to the sum-of-the-years’-digits method and which corresponds to the double-declining-balance method?
Sum-of-the-years'-digits Double-declining-balance A III II B II I C I III D II III
Answer is D
Explanation:
The depreciation pattern corresponding to the sum-of-the-years’-digits (SYD) method is the straight sloping line (II) and the pattern corresponding to the double-declining-balance (DDB) method is the curved line (III). The straight line parallel with the horizontal axis (I) describes the pattern of the straight-line (SL) method of depreciation because the SL method results in a constant amount of depreciation each period. The line for the pattern of the SYD method is a straight sloping line because the method results in a decreasing depreciation charge based on a decreasing fraction of depreciable cost where each fraction has the same denominator (sum of the years). The line for the pattern of the DDB method will be a curved sloping line because that type of method employs a constant percentage multiplied by a decreasing balance to obtain a decreasing charge for depreciation.
company with a June 30 fiscal year-end entered into a $3,000,000 construction project on April 1 to be completed on September 30. The cumulative construction-in-progress balances at April 30, May 31, and June 30 were $500,000, $800,000, and $1,500,000, respectively. The interest rate on company debt used to finance the construction project was 5% from April 1 through June 30 and 6% from July 1 through September 30. Assuming that the asset is placed into service on October 1 and an additional amount spent at the beginning of the month, what amount of interest should be capitalized to the project on June 30?
A $11,667 B $18,750 C $75,000 D $90,000
Explanation:
The correct answer is (A).
The interest cost incurred during the construction period needs to be capitalized to the asset. The interest to be capitalized is lower of the following:
Weighted Average Accumulated Expenditure * Interest rate.
The actual interest cost incurred.
Interest: ($500,000 x 3/12 (April - June) x 5%) + ($300,000 x 2/12 (May - June) x 5% ) + ($700,000 x 1/12 (June) x 5%) = $11,667.
The company has a June 30th year-end and the question only asks for that, therefore, any information beyond June 30th is irrelevant.
Interest until June 30th is 5%. As of June 30, the interest amount to be capitalized is $11,667.
Which of the following uses the straight-line depreciation method? Group depreciation Composite depreciation
Group depreciation Composite depreciation
A No No
B Yes No
C Yes Yes
D No Yes
Answer is C
Explanation:
The composite depreciation method refers to the depreciation of a collection of assets that are dissimilar. The group depreciation method refers to the depreciation of a collection of assets that are similar in nature. From an accounting standpoint, there is no distinction between the two methods. The same procedures are followed for both, and both can utilize the straight-line depreciation method.
At the beginning of the year, the carrying value of an asset was $1,000,000 with 20 years of remaining life. The fair value of the liability for the asset retirement obligation was $100,000. At year end, the carrying value of the asset was $950,000. The risk-free interest rate was 5%. The credit-adjusted riskfree interest rate was 10%. What was the amount of accretion expense for the year related to the asset retirement obligation? A $ 10,000 B $ 50,000 C $ 95,000 D $ 100,000
Answer is A
Explanation:
An asset retirement obligations (ARO) must be recorded at fair value in the accounting period in which it occurs and in which its amount can be reasonably measured. AROs incur depreciation and accretion expenses each year. Accretion expense is offset with an increase to the liability account, and, at the end of the asset’s life, the liability account will have a balance equal to the amount needed to settle the retirement obligation. Accretion expense is calculated by multiplying the balance of the recorded liability by the company’s credit-adjusted discount rate each year, so the amount of accretion expense for the year is $10,000 ($100,000 x 10%).
A transportation company purchased a passenger bus for $100,000 on January 1, year 1. The company expects the bus to be used for 20 years if it follows a maintenance schedule of replacing the engine after 10 years and replacing the seats every eight years. It estimates that the current cost to replace the engine is $25,000 and the current cost to replace the seats is $10,000. The company uses straight-line depreciation and the bus has no residual value. The company considers any component equal to or greater than 10% of the overall cost to be significant. Under IFRS, how much depreciation expense should the company recognize for the bus for the year ended December 31, Year 1?
A $5,000 B $7,000 C $7,250 D $8,500
Explanation:
The correct answer is (B).
The question is asking to calculate depreciation expense for the bus for the year ended December 31, year 1. Calculate the depreciation separately for each Item. Depreciation for each item:
Bus: $100,000 - $25,000 (engine) - $10,000 (seat) = $65,000 / 20 years = $3,250 depreciation for the bus.
Engine: $25,000 / 10 = $2,500. Engine is replaced every 10 years.
Seats: $10,000 / 8 = $1,250. Seats are replaced every 8 years.
Total Depreciation = $3,250 + $2,500 + $1,250 = $7,000.
A fixed asset with a five-year estimated useful life and no residual value is sold at the end of the second year of its useful life. How would using the sum-ofthe- years'-digits method of depreciation instead of the double-declining-balance method of depreciation affect a gain or loss on the sale of the fixed asset? Gain Loss A Decrease Decrease B Decrease Increase C Increase Decrease D Increase Increase
Answer is B
Explanation:
This question requires the determination of how using the sum-of-the-years’ digits method of depreciation instead of the double declining balance method of depreciation affects the gain or loss on the sale of a fixed asset with a five-year estimated useful life and no residual value that is sold at the end of the second year of its useful life. A gain or loss on the sale of a fixed asset is computed by comparing the proceeds received from its sale to its carrying amount at the date of sale. To determine the carrying amount of the fixed asset at the sale date, two years of depreciation must be subtracted from its cost. Because the cost of the fixed asset is not given, there are two options: (1) determine depreciation as a percent or fraction of the unknown cost of the fixed asset, ‘x,’ or (2) plug in any value for the cost of the machine, and simply compute depreciation under both methods based on this amount. We chose the latter approach, assigning to the machine a cost of $300. Now we can compare the carrying amount of the fixed asset at the sale date under both methods. The fixed asset has a greater carrying amount at the date of sale under the sum-of-the-years’-digits method. Therefore, using the sum-of-the-years’-digits method of depreciation instead of the double-declining-balance method of depreciation decreases any gain and increases any loss recognized on the sale of the fixed asset. :
Cost of fixed asset $300 Depreciation to date of sale: Yr. 1: 5/15 x ($300 - $0) (100) Yr. 2: 4/15 x ($300 - $0) (80) Carrying amount, date of sale, SYD 120 Cost of fixed asset $300 Depreciation to date of sale: Yr. 1: 2/5 x ($300 - $0) ($120) Yr. 2: 2/5 x ($300 - $120) (72) Carrying amount, date of sale, DDB $108
When equipment is retired, accumulated depreciation is debited for the original cost less any residual recovery under which of the following depreciation methods?
Composite depreciation Group depreciation
A No No
B No Yes
C Yes No
D Yes Yes
Answer is D
Explanation:
Under both the group depreciation and the composite depreciation methods, assets are depreciated on the basis of their average lives. New assets are recorded at cost, and retirements are accounted for by crediting Assets for the original cost of the equipment and debiting Accumulated Depreciation for the original cost less proceeds received; thus, no gain or loss is recognized on disposal. The theoretical justification for this is that some assets will be retired early, while some will be used longer than originally estimated. The same approach is used under both group and composite depreciation–the only difference is that “group depreciation” refers to pools of assets that are similar in nature, whereas “composite” refers to essentially dissimilar assets.
On January 2, Year 1, Ames Corp. signed an eight-year lease for office space. Ames has the option to renew the lease for an additional four-year period on or
before January 2, Year 8. During January of year 1, Ames incurred the following costs:
$120,000 for general improvements to the leased premises with an estimated useful life of ten years.
$50,000 for office furniture and equipment with an estimated useful life of ten years.
At December 31, Year 1, Ames’ intentions as to exercise of the renewal option are uncertain. A full year’s amortization of leasehold improvements is taken for
calendar year 1. In Ames’ December 31, Year 1 balance sheet, accumulated amortization should be
A $10,000 B $15,000 C $17,000 D $21,250
Explanation:
The correct answer is (B).
Leasehold improvements are improvements made by the lessee to the leased property. The improvements are not separable from the leased property and revert to the lessor at the end of the lease term.
Of the items listed, only the general improvements made to the leased premises represent a leasehold improvement.
As such, it should be amortized over the lesser of the lease term (i.e., 8 years) or the estimated useful life of the leasehold improvement (i.e., 10 years).
The likelihood of lease renewal is too uncertain to warrant apportioning the cost over the sum of the remaining term of the lease and the period covered by the renewal option (i.e., 12 years).
Therefore, the accumulated amortization to be reported in the 12/31, year 1 balance sheet with respect to the leasehold improvements is $15,000 ($120,000 / 8 years).
Office furniture and equipment are fixed assets and depreciated (not amortized) and the depreciation of $5,000 ($50,000/10) is not included in accumulated amortization.
Sea Manufacturing Corp. is constructing a new factory building. During the current calendar year, Sea made the following payments to the construction company:
January 2 $1,000,000
December 31 $1,000,000
Sea has an 8%, three-year construction loan of $3,000,000. What is the amount of interest costs that Sea may capitalize during the current year?
A $0 B $80,000 C $160,000 D $240,000
Explanation:
The correct answer is (B).
Interest cost incurred during the construction period needs to be capitalized to the asset.
The interest to be capitalized is lower of the following:
Weighted average accumulated expenditure x Interest rate
The actual interest cost incurred.
The amount of capitalized interest is based on the weighted average amount of accumulated expenditures.
Weighted average amount of accumulated expenditures is:
$1,000,000 (12/12) + $1,000,000 (0/12) = $1,000,000
$1,000,000 * 8% = $80,000
The full $80,000 should be capitalized because the total interest on the loan is $240,000 ($3,000,000 * 8%).
(A) is incorrect because Interest is capitalized at $80,000, not $0.
(C) is incorrect because it calculated on $2,000,000 x 8% = $160,000.
(D) is incorrect because it is the actual interest cost incurred
Lano Corp.’s forest land was condemned for use as a national park. Compensation for the condemnation exceeded the forest land’s carrying amount. Lano purchased similar, but larger, replacement forest land for an amount greater than the condemnation award. As a result of the condemnation and replacement, what is the net effect on the carrying amount of forest land reported in Lano’s balance sheet?
A
The amount is increased by the excess of the replacement forest land’s cost over the condemned forest land’s carrying amount.
B
The amount is increased by the excess of the replacement forest land’s cost over the condemnation award.
C
The amount is increased by the excess of the condemnation award over the condemned forest land’s carrying amount.
D
No effect, because the condemned forest land’s carrying amount is used as the replacement forest land’s carrying amount.
Answer is A
Explanation:
The condemnation of the forest land represents an example of an involuntary conversion of a nonmonetary asset to a monetary asset. Any gain or loss realized on the property converted be recognized in income even though the enterprise reinvests or is obligated to reinvest the monetary assets in replacement nonmonetary assets. Since any gain or loss realized on the property converted is recognized in income, the replacement nonmonetary asset is recorded at cost. Because the cost of the replacement forest land exceeds the condemnation award which exceeds the condemned forest land’s carrying amount, the debit to Forest Land to record the cost of the replacement property exceeds the credit to Forest Land to remove the carrying amount of the condemned property, and thus the carrying amount of Forest Land in the balance sheet increases by the amount of the excess.
Land was purchased to be used as the site for the construction of a plant. A building on the property was sold and removed by the buyer so that construction on the plant could begin. The proceeds from the sale of the building should be
A
Netted against the costs to clear the land and expensed as incurred.
B
Netted against the costs to clear the land and amortized over the life of the plant.
C
Deducted from the cost of the land.
D
Classified as other income.c
c
Explanation:
Sometimes a structure is constructed on a newly acquired site with an existing building on it. If the existing building is sold and the buyer assumes the costs for its removal, the entire amount of the proceeds from the sale of the building should be deducted from the cost of the land.
Sun Co. was constructing fixed assets that qualified for interest capitalization. Sun had the following outstanding debt issuances during the entire year of construction: $6,000,000 face value, 8% interest $8,000,000 face value, 9% interest. None of the borrowings were specified for the construction of the qualified fixed asset. Average expenditures for the year were $1,000,000. What interest rate should Sun use to calculate capitalized interest on the construction?
A 8.00% B 8.50% C 8.57% D 9.00%
Explanation:
The correct answer is (C).
The cost of assets constructed for the use of the business should include all directly-related costs: cost of direct materials, cost of direct labor, additional overhead incurred, and interest costs incurred during the construction period. If the average accumulated expenditures of an asset exceed the amount of any specific borrowings associated with the asset, the excess should be capitalized at the weighted average of interest rates applicable to other borrowings of the business. There was no specific borrowing mentioned, so the interest rate would simply be the weighted average.
(($6,000,000 × 0.08) + ($8,000,000 × 0.09)) / $14,000,000 = 0.0857.
Which of the following is not an example of wasting assets? A Minerals B Gas deposits C Land D Oil deposits
C
Explanation:
Wasting assets are natural resources, such as minerals and gas and oil deposits that are subject to exhaustion through extraction. Land is deemed to have an indefinite life and is not depreciated.
Derby Co. incurred costs to modify its building and to rearrange its production line. As a result, an overall reduction in production costs is expected. However, the modifications did not increase the building’s market value, and the rearrangement did not extend the production line’s life. Should the building modification costs be capitalized?
Building modification costs Production line rearrangement costs
A Yes No
B Yes Yes
C No No
D No Yes
b
Explanation:
Both the building modification costs and the production line rearrangement costs should be capitalized because they have resulted in an overall reduction of production costs, the benefits of which extend beyond the current period.
Question #1487, Blueprint Area: Property, Plant and Equipment
On January 1, Pax Company acquired for $18,000 a new piece of equipment with an estimated useful life of 10 years. The equipment required the addition of a custom-made component, costing $3,000 that must be replaced in 4 years. Pax uses the straight-line method of depreciation. Under IFRS, what is the depreciation expense for the year ended December 31?
A $0 B $1,800 C $2,100 D $2,550
d
Explanation:
Under IFRS, components with a different estimated useful life must be depreciated separately. The equipment would have $1,800 depreciation ($18,000/10 years) and the component would have $750 depreciation ($3,000/4 years), for total depreciation expense of $2,550 annually.
A company has a long-lived asset with a carrying value of $120,000, expected future cash flows of $130,000, present value of expected future cash flows of $100,000, and a market value of $105,000. What amount of impairment loss should be reported? A $0 B $5,000 C $15,000 D $20,000
a
Explanation:
An impairment loss shall be recognized only if the carrying amount of a long-lived asset, or asset group, is not recoverable and exceeds its fair value. The carrying amount (book value) is not recoverable if it exceeds the sum of the undiscounted cash flows expected to result from the use and eventual disposition of the asset. The amount of an impairment loss is the difference between an asset’s book and fair value. The $120,000 carrying value of the company’s longlived does not exceed the $130,000 undiscounted future cash flows expected to result from the asset so there is no impairment loss.
An asset is being constructed for an enterprise’s own use. The asset has been financed with a specific new borrowing. The interest cost incurred during the construction period as a result of expenditures for the asset is
A
Interest expense in the construction period
B
A prepaid asset to be written off over the estimated useful life of the asset
C
A part of the historical cost of acquiring the asset to be written off over the estimated useful life of the asset
D
A part of the historical cost of acquiring the asset to be written off over the term of the borrowing used to finance the construction of the asset
c
Explanation:
The capitalization of interest cost incurred during the construction of most assets having an extended construction period is required. Capitalized interest is included in the cost of the asset in the same manner as any other construction cost; it is then written off over the life of the asset as part of the periodic depreciation charges.
A depreciable asset has an estimated 15% salvage value. At the end of its estimated useful life, the accumulated depreciation would equal the original cost of the asset under which of the following depreciation methods?
Productive output Sum-of-the-years’-digits
A Yes No
B No No
C No Yes
D Yes Yes
b
Explanation:
A fixed asset should never be depreciated below its salvage value under any method of depreciation. The depreciable base of a plant asset is its acquisition cost less any estimated salvage value. The depreciable base is allocated to expense over the life of the asset in a systematic and rational manner. At the end of its estimated useful life, the accumulated depreciation would equal the depreciable base of the asset.
An expenditure to install an improved electrical system is a Capital expenditure Revenue expenditure A No Yes B No No C Yes No D Yes Yes
c
Explanation:
The expenditure to install an improved electrical system represents a betterment (or improvement); that is, the substitution of a better asset for an existing asset. The expenditure increases the service potential of the electrical system. Because this increased service potential is a benefit that will be enjoyed throughout the life of the electrical system, the expenditure should be capitalized and depreciated over the life of the system. Revenue expenditures, on the other hand, are recurring expenditures that do not add to the service potential of a plant asset; they serve merely to maintain a given level of services. Revenue expenditures should be expensed when incurred.
In an exchange with commercial substance, Vey Co. traded equipment with an original cost of $100,000 and accumulated depreciation of $40,000 for similar productive equipment with a fair value of $120,000. In addition, Vey received $30,000 cash in connection with this exchange. What should be Vey's carrying amount for the equipment received on the day of exchange? A $ 90,000 B $ 60,000 C $ 48,000 D $120,000
d
Explanation:
In an exchange with commercial substance, the transaction is recorded at the fair value of the asset received or the asset given up, whichever is more clearly evident, and a gain or loss is recognized on the exchange. The fair value of the equipment received is given in the scenario, therefore, it is more clearly evident than the fair value of the equipment given up.
The journal entry to be recorded in Vey’s books is:
Cash 30,000 New Equipment 120,000 Accumulated Depreciation 40,000 Old Equipment 100,000 Gain on Exchange 90,000
On January 1, year 1, Crater, Inc. purchased equipment having an estimated salvage value equal to 20% of its original cost at the end of a 10-year life. The equipment was sold December 31, year 5, for 50% of its original cost. If the equipment's disposition resulted in a reported loss, which of the following depreciation methods did Crater use? A Double-declining balance. B Sum-of-the-years'-digits. C Straight-line. D Composite.
c
Explanation:
The carrying amount of the equipment at the date of sale is computed by subtracting the accumulated depreciation on the equipment from the cost of the equipment. Under the straight-line method, the accumulated deprecation on the equipment at the date of sale is equal to 40% of the original cost of the equipment [i.e., (100% - 20%) x 1/10 x 5]. Thus, under the straight-line method, the carrying amount of the equipment at the date of sale is 60% (i.e., 100% - 40%) of the original cost of the equipment. Since the equipment was sold for only 50% of its original cost, use of the straight-line method would have resulted in a loss being recognized on the sale equal to 10% (i.e., 60% - 50%) of the original cost of the equipment.
Cole Co. began constructing a building for its own use in January of the current year. During the year, Cole incurred interest of $50,000 on specific construction debt, and $20,000 on other borrowings. Interest computed on the weighted-average amount of accumulated expenditures for the building during the year was $40,000. What amount of interest cost should Cole capitalize? A $20,000 B $40,000 C $50,000 D $70,000
b
Explanation:
The amount of interest that may be capitalized is based on the weighted-average amount of accumulated expenditures. The weighted-average amount of accumulated expenditures applies the avoidable interest concept. This concept limits the amount of interest to be capitalized to the lower of the actual interest cost incurred during the period or avoidable interest. Avoidable interest is the amount of interest cost incurred during the period that theoretically could have been avoided if expenditures for the asset had not been made.
Which of the following statements concerning the acquisition of assets is false?
A
Donated assets should be recorded at book value along with any incidental costs incurred.
B
When an asset is received from a governmental entity, no income is recognized, and the offsetting credit is to an owners’ equity account, “Additional Paid-In Capital: Donated Assets.”
C
Assets donated by entities other than governmental units are included in revenue in the period of receipt.
D
If several dissimilar assets are purchased for a lump sum, the total amount paid should be allocated to each individual asset on the basis of its relative fair value.
Explanation:
The correct answer is (A).
The question is asking for which statement is false. Donated assets should be recorded at fair value along with any incidental costs incurred and not at Book Value.
A company recently moved to a new building. The old building is being actively marketed for sale, and the company expects to complete the sale in four months. Each of the following statements is correct regarding the old building, except:
A
It will be reclassified as an asset held for sale.
B
It will be classified as a current asset.
C
It will no longer be depreciated.
D
It will be valued at historical cost.
D
Explanation:
Criteria to determine when long-lived assets are held for sale include requirements that: (a) the asset is available for prompt sale as is, subject only to customary and usual sales terms for such assets; and (b) the asset sale is probable and generally to be completed within 12 months. The asset is reclassified to a current asset, and is measured at the lower of its book or fair value less cost to sell and its depreciation (or amortization) discontinues.
On January 1, year 2, Victor Company purchased for $85,000 a machine having a useful life of ten years and an estimated salvage value of $5,000. The machine was depreciated by the straight-line method. On July 1, year 7, the machine was sold for $45,000. For the year ended December 31, year 7, how much gain should Victor record on the sale? A $0 B $1,000 C $4,000 D $6,750
Explanation:
The machine had a depreciable basis of $80,000 ($85,000 cost - $5,000 salvage value) and was depreciated at a rate of $8,000 per year ($80,000 / 10-year useful life). At the time of sale, Victor depreciated the machine for 5.5 years and its carrying amount was equal to $41,000, i.e., [$85,000 - (5.5 × $8,000)]. Thus, a $4,000 gain ($45,000 - $41,000) should be recognized.
Cart Co. purchased an office building and the land on which it is located for $750,000 cash and an existing $250,000 mortgage. For realty tax purposes, the property is assessed at $960,000, 60% of which is allocated to the building. At what amount should Cart record the building? A $500,000 B $576,000 C $600,000 D $960,000
Explanation:
Assets are to be recorded at their acquisition cost, which is defined as the cash price or its equivalent. The acquisition cost of the office building and the land together is $1,000,000; the total of the $750,000 cash and $250,000 mortgage. The property is assessed with 60% allocated to the building. Cart would record the building at $600,000 ($1,000,000 x 60%) and the land at the remaining $400,000.
Rye Co. purchased a machine with a four-year estimated useful life and an estimated 10% salvage value for $80,000 on January 1 of the current year. In its income statement for the third year, what would Rye report as the depreciation expense using the double-declining-balance method? A $ 9,000 B $10,000 C $18,000 D $20,000
Explanation:
Under the double-declining-balance method of depreciation, the annual depreciation charge is computed by multiplying the carrying amount of the plant asset (i.e., cost minus accumulated depreciation) by a rate equal to 200% of the straight-line rate (e.g., 200% x 25% = 50%). Salvage value is not used in the depreciation formula, but the plant asset cannot be depreciated below its salvage value.
First year expense [50% x ($80,000 - $0)] = $40,000
Second year expense [50% x ($80,000 - $40,000)] = $20,000
Third year expense [50% x ($80,000 - $60,000)] = $10,000
Talton Co. installed new assembly line production equipment at a cost of $185,000. Talton had to rearrange the assembly line and remove a wall to install the equipment. The rearrangement cost $12,000 and the wall removal cost $3,000. The rearrangement did not increase the life of the assembly line but it did make it more efficient. What amount of these costs should be capitalized by Talton? A $185,000 B $188,000 C $197,000 D $200,000
D
Explanation:
Assets are to be recorded at their acquisition cost. Acquisition cost is defined as the cash price, or equivalent, plus all costs reasonably necessary to bring it to the location and to make it ready for its intended use. Both the $12,000 rearrangement cost and $3,000 removal cost were necessary to bring the asset to its location and make it ready for its intended use. Talton would capitalized $200,000 ($185,000 + $12,000 + $4,000 = $200,000).
Ott Co. purchased a machine at an original cost of $90,000 on January 2, year 1. The estimated useful life of the machine is 10 years, and the machine has no salvage value. Ott uses the straight-line method to calculate depreciation. On July 1, year 10, Ott sold the machine for $5,000. What is the amount of gain or loss on the disposal of the machine?
A $500 loss B $500 gain C $4,500 loss D $4,500 gain
Explanation:
The correct answer is (B).
Calculate Depreciation:
Depreciation under straight-line method each year = Cost of the asset / Useful life = $90,000 / 10 years = $9,000 per year.
Calculate Accumulated Depreciation& Carrying Value
So the total accumulated depreciation for 9 ½ years (January 2, year 1 to June 30, year 10) @ $9,000 each year = $9,000 x 9.5 years = $85,500.
The carrying value of the machine on July 1, year 10 = $4,500 [i.e. $90,000 (historical cost) - $85,500 (accumulated depreciation)].
Calculate Gain/Loss on Sale:
The amount of gain on disposal of a machine is equal to the selling price of machine less the carrying value of the machine on the date of sale.
$4,500 Carrying value - Selling price of $5,000 = $500 Gain.
A company completes construction of a $400 million offshore oil platform and places it into service on January 1. State law requires that the platform bed is mantled and removed at the end of its useful life, which is estimated to be 10 years. The company estimates that the cost of dismantling the platform will be $20 million. The discounted value of the liability is $9 million using the company’s credit-adjusted, risk-free rate. The company has already capitalized the $400 million construction cost of the platform. What amounts should the company record as liability and expense when the asset is placed into service?
A Liability, $0; expense, $0 B Liability, $9,000,000; expense, $0 C Liability, $9,000,000; expense, $9,000,000 D Liability, $20,000,000; expense, $20,000,000
Explanation:
The correct answer is (B).
An Asset Retirement Obligation (ARO) involves the retirement of a tangible, long-lived asset that depends on a future event beyond the control of an obligated party. An ARO is an essential part of producing fair and accurate financial statements so those viewing them can have a better idea of a company’s obligations, as well as its overall value. An ARO liability must be recorded when the asset is placed into service. An accretion expense, which is a periodic expense recognized as the present value of a balance sheet liability increases, will be recognized at the end of each period, gradually increasing the ARO to its full $20,000,000 million amount when due in 10 years. The discounted value of the liability is $9,000,000 million using the company’s credit-adjusted, risk-free rate. The expense will be $0.
Which of the following would not be an example of an event or change in circumstances indicating that the carrying amount of the asset or asset group may not be recoverable?
A
A significant increase in the market price of a long-lived asset (asset group)
B
A significant adverse change in the extent or manner in which a long-lived asset (asset group) is being used or in the physical condition of the asset
C
An accumulation of costs significantly in excess of the amount originally expected for the acquisition or construction of a long-lived asset (asset group)
D
A current expectation that, more likely than not, a long-lived asset (asset group) will be sold or otherwise disposed of significantly before the end of its previously estimated useful life
A
Explanation:
A significant decrease (not increase) in the market price of a long-lived asset (asset group) would be an example of an event or change in circumstances indicating that the carrying amount of the asset or asset group may not be recoverable.
A company recorded a decommissioning liability and recognized the amount recorded as part of the cost of the related property. After the property was fully depreciated, the decommissioning liability was reviewed and adjusted. How should this change in the decommissioning liability be recognized under IFRS?
A
The change in the liability is recognized in other comprehensive income.
B
The change in the liability is recognized in profit or loss.
C
The change in the liability is recognized as a change in the carrying amount of the property if the liability increases but is otherwise recognized in profit or loss.
D
The change in the decommissioning liability is not recognized until it is settle
Explanation:
The correct answer is (B)
The change in the liability is recognized in profit or loss.
A decommissioning liability is the estimated cost to put the property into a usable or saleable condition when it is no longer going to be used for its original purpose. The liability is reviewed and adjusted to reflect changes in the estimated cost with increases or decreases recognized as adjustments to the carrying value of the property. When the property is fully depreciated, any adjustment to the decommissioning liability is recognized as a profit or loss since it is too late to recognize it as an adjustment to depreciation or amortization.
(A), (C) and (D) are incorrect as per the above explanation.
A company performing its long-lived asset impairment testing is reviewing the fair value of equipment. Each of the following valuation techniques may be appropriate for measuring the fair value of the equipment, except the
A Market approach. B Income approach. C Cost approach. D Net realizable value approach.
Explanation:
The correct answer is (D)
US GAAP permits only the following techniques for measuring the fair value of an asset or liability:
Market approach
Income approach
Cost approach
However, net realizable value approach is not considered to be a valid technique for fair value measurement and is not appropriate.
On December 31 of the current year a building owned by Carr, Inc., was destroyed by fire. Carr paid $12,000 for removal and clean-up costs. The building had a book value of $250,000 and a fair value of $280,000 on December 31. What amount should Carr use to determine the gain or loss on this involuntary conversion? A $250,000 B $262,000 C $280,000 D $292,000
B
Explanation:
Building carrying amount, 12/31 $250,000
Removal and clean-up costs 12,000
Amount to determine gain or loss on involuntary conversion $262,000
A building suffered uninsured water and related damage. The damaged portion of the building was refurbished with upgraded materials. The cost and related accumulated depreciation of the damaged portion are identifiable. To account for these events, the owner should
A
Capitalize the cost of refurbishing and record a loss in the current period equal to the carrying amount of the damaged portion of the building.
B
Capitalize the cost of refurbishing by adding the cost to the carrying amount of the building.
C
Record a loss in the current period equal to the cost of refurbishing and continue to depreciate the original cost of the building.
D
Record a loss in the current period equal to the sum of the cost of refurbishing and the carrying amount of the damaged portion of the building.
A
Explanation:
The damaged portion of the building was refurbished with upgraded materials which indicates that a ‘betterment’ is involved. There are benefits to future periods as a result of the refurbishing expenditures; thus, they should be capitalized. The cost and related accumulated depreciation of the damaged portion of the building are identifiable so they should be removed from the books (with a resulting debit to ‘loss’ for the difference) because this portion of the building has been
replaced to a certain extent and upgraded.
Quick Co. acquired the following assets from a liquidating competitor for a $200,000 lump-sum purchase price: Competitor's carrying amount Fair Value Inventory $ 70,000 $50,000 Land 40,000 50,000 Building 110,000 150,000 $220,000 $250,000What amount should Quick report as the cost of the building? A $100,000 B $120,000 C $150,000 D $200,000
Explanation:
If several dissimilar assets are purchased for a lump sum, the total amount paid should be allocated to each individual asset on the basis of its relative fair value. The allocation formula: Asset Y = Total cost of assets × FV of Y / Total FV. In this case: $200,000 × $150,000 ÷ $250,000 = $120,000.
During the year, Burr Co. had the following transactions pertaining to its new office building: Purchase price of land $ 60,000 Legal fees for contracts to purchase land 2,000 Architects' fees 8,000 Demolition of old building on site 5,000 Sale of scrap from old building 3,000 Construction cost of new building (fully completed) 350,000In Burr's December 31 balance sheet, what amounts should be reported as the cost of land and cost of building? Land Building A $60,000 $360,000 B $62,000 $360,000 C $64,000 $358,000 D $65,000 $362,000
Explanation:
The legal fees for contracts to purchase land are part of the cost of the land. The cost of demolishing the old building (net of any scrap proceeds) is a reasonable and necessary cost to get the land ready for its intended use.
Purchase price of land $ 60,000
Legal fees for contracts to purchase land 2,000
Demolition of old building on site 5,000
Sale of scrap from old building (3,000)
Cost of land $ 64,000
Architect’s fees $ 8,000
Construction cost of new building 350,000
Cost of building $358,000
A company obtained a $300,000 loan with a 10% interest rate on January 1, year 1, to finance the construction of an office building for its own use. Building construction began on January 1, year 1, and the project was not completed as of December 31, year 1. The following payments were made in year 1 related to the construction project: January 1 Purchased land for $120,000 September 1 Progress payment to contractor for $150,000What amount of interest should be capitalized for the year ended December 31, year 1? A $13,500 B $15,000 C $17,000 D $30,000
Explanation:
The amount of interest cost to be capitalized is the interest cost incurred during the acquisition period that could have been avoided if expenditures for the asset had not been made. If a specific interest rate is associated with the asset, that rate should be used to the extent that the average accumulated expenditures on the asset do not exceed the amount borrowed at the specific rate.
Average expenditures during year $120,000 × 12/12 $120,000 150,000 × 4/12 50,000 Average expenditures 170,000 Specific borrowing rate 10% Capitalized interest expense $ 17,000
Fountain Co. is constructing an office building for its own use. Fountain started the two-year construction project on April 1, year 1, at which point the interest capitalization period began. Fountain made the following payments in year 1 related to the construction of the building:
April 1 Payment to architect for building plans $ 30,000
July 1 Progress payment to contractor 60,000
October 1 Progress payment to contractor 150,000
For the purpose of capitalizing interest, what is Fountain’s weighted-average accumulated expenditures for the year ended December 31, year 1?
A $80,000 B $105,000 C $120,000 D $240,000
Explanation:
The correct answer is (C).
Interest cost incurred during the construction period needs to be capitalized to the asset. The interest to be capitalized is lower of the following:
- Weighted-average accumulated expenditure x Interest rate.
- The actual interest cost incurred.
The weighted-average accumulated expenditures for the purpose of capitalization of borrowing costs can be calculated as under:
Date Particulars Amount Capitalization Period Weighted-average accumulated expenditures
April 1 Payment to architect for building plans $30,000 9/9 $30,000
July 1 Progress payment to contractor $60,000 6/9 $40,000
October 1 Progress payment to contractor $150,000 3/9 $50,000
For the purpose of capitalizing interest, what is Fountain’s weighted-average accumulated expenditures for the year ended December 31, year 1?
Hudson Corp. operates several factories that manufacture medical equipment. The factories have a historical cost of $200 million. Near the end of the company's fiscal year, a change in business climate related to a competitor's innovative products indicated to Hudson's management that the $170 million carrying amount of the assets of one of Hudson's factories may not be recoverable. Management identified cash flows from this factory and estimated that the undiscounted future cash flows over the remaining useful life of the factory would be $150 million. The fair value of the factory's assets is reliably estimated to be $135 million. The change in business climate requires investigation of possible impairment. Which of the following amounts is the impairment loss? A $15 million B $20 million C $35 million D $65 million
Explanation:
A long-lived asset shall be tested for recoverability whenever events or changes in circumstances indicate that its carrying amount may not be recoverable. Impairment is the condition that exists when the carrying amount of a long-lived asset, or asset group, exceeds its fair value. An impairment loss shall be recognized only if the carrying amount of a long-lived asset is not recoverable and exceeds its fair value. The carrying amount (book value) is not recoverable if it exceeds the sum of the undiscounted cash flows expected to result from the use and eventual disposition of the asset. The amount of an impairment loss is the difference between an asset’s book and fair value. The new book value is used as a basis for depreciation. Since the carrying amount of $170,000 exceeds the undiscounted cash flows of $150,000 an impairment loss must be recognized. The $35,000 impairment loss is the difference between the book value of $170,000 and the fair value of $135,000.
On January 3, Quarry Co. purchased a manufacturing machine for $864,000. The machine had an estimated eight-year useful life and a $72,000 estimated salvage value. Quarry expects to manufacture 1,800,000 units over the life of the machine. During the year, Quarry manufactured 300,000 units. Quarry uses the units-of-production depreciation method. In its December 31 balance sheet, what amount of accumulated depreciation should Quarry report for the machine? A $ 99,000 B $108,000 C $132,000 D $144,000
Explanation:
Units-of-output depreciation takes into account salvage value and the number of units produced by the asset.
(Cost - salvage) / expected output x current output:
$864,000 - $72,000 = $792,000
$792,000 / 1,800,000 = $0.44
$0.44 x 300,000 = $132,000
If an outlay will provide a service benefit beyond the current period, it is considered which of the following? A An expense B A capital expenditure C A revenue expenditure D None of the above
B
Explanation:
If an outlay will provide a service benefit beyond the current period, it is a capital expenditure and is recorded as an asset.
Quick Co. acquired the following assets from a liquidating competitor for a $200,000 lump-sum purchase price:
Competitor’s carrying amount ($) Fair Value ($)
Inventory 70,000 50,000
Land 40,000 50,000
Building 110,000 150,000
Total 220,000 250,000
What amount should Quick report as the cost of the building?
A $100,000 B $120,000 C $150,000 D $200,000
Explanation:
The correct answer is (B).
In the case of a basket purchase, the purchase price is allocated based on the appraised fair value of individual items. The cost of building reported will be calculated as a proportion of total fair value at 60% ($150,000/$250,000).
Proportion of the purchase price allocated to building: $120,000 = 60% x $200,000.
Summary Fair Value ($) Proportion of the FV paid Cost of Acquisition ($)
Inventory 50,000 20% 40,000
Land 50,000 20% 40,000
Building 150,000 60% 120,000
Total 250,000 100% 200,000
(A) is incorrect because it is calculated based on the proportionate share of the carrying value instead of using fair value to determine the proportionate share, $100,000 (50% x 200,000). Proportionate share using carrying value: ($110,000/$220,000 = 50%).
(C) is incorrect because it is the actual fair value of the building. Quick should report at proportionate share derived from the fair values of the assets purchased.
(D) is incorrect because it is the actual cost paid by the quick to acquire inventory, land, and building.
Isle Co. owned a copy machine that cost $5,000 and had accumulated depreciation of $2,000. Isle exchanged the copy machine for a computer that cost $4,000. Isle’s future cash flows are not expected to change significantly as a result of the exchange. What amount of gain or loss should Isle report and at what amount should it record the asset?
A
No gain or loss in the income statement; $3,000 asset in the balance sheet.
B
No gain or loss in the income statement; $4,000 asset in the balance sheet.
C
$1,000 gain in the income statement; $3,000 asset in the balance sheet.
D
$1,000 gain in the income statement; $4,000 asset in the balance sheet.
Explanation:
The correct answer is (A).
An exchange of non-monetary assets that is not expected to change Isle Co’s cash flows significantly lacks commercial substance. Therefore, accounting for a non-monetary exchange is based on the carrying amount of the assets given up. Also, there is no boot, so no gain is recognized. The computer received by Isle Co is recorded at the carrying amount of the copy machine of $3,000 ($5,000 - $2,000) and no gain or loss is recognized in the income statement.
Murray Co. maintains its records under IFRS. During the year, Murray sold a machine that had been accounted for using the revaluation method. Details are presented below:
Sales Price $150,000
Machine Book Value $ 80,000
Revaluation Surplus $ 9,000
Which of the following is correct regarding the sale?
A
The gain of $70,000 should be credited to Other Comprehensive Income (OCI).
B
The gain of $79,000 should be recorded in profit and loss on the Income Statement.
C
The gain of $70,000 should be recorded in profit and loss, and the $9,000 revaluation surplus should be transferred to retained earnings.
D
No gain or loss should be recorded.
Explanation:
Two journal entries are required when an asset accounted for under the revaluation model is sold. First, transfer the related revaluation surplus of $9,000 to retained earnings. Second, record the sale and gain/ loss as usual. Since the machine’s net book value is $80,000, and cash of $150,000 was received, the gain on sale would be $70,000.
On October 1 of the current year, Shaw Corp. purchased a machine for $126,000 that was placed in service on November 30. Shaw incurred additional costs for this machine as follows: Shipping $3,000 Installation 4,000 Testing 5,000In Shaw's December 31 balance sheet, the machine's cost should be reported as A $126,000 B $129,000 C $133,000 D $138,000
Explanation:
The acquisition cost of the machine includes all of the costs incurred to get it ready for its intended use. Thus, the costs incurred in shipping, installing and testing the machine are capitalized and added to its purchase price, resulting in an acquisition cost for the machine of $138,000 ($126,000 + $3,000 + $4,000 + $5,000).
During the previous year, Yvo Corp. installed a production assembly line to manufacture furniture. In the current year, Yvo purchased a new machine and rearranged the assembly line to install this machine. The rearrangement did not increase the estimated useful life of the assembly line, but it did result in significantly more efficient production. The following expenditures were incurred in connection with this project: Machine $75,000 Labor to install machine 14,000 Parts added in rearranging the assembly line to provide future benefits 40,000 Labor and overhead to rearrange the assembly line 18,000What amount of the above expenditures should be capitalized in the current year? A $147,000 B $107,000 C $ 89,000 D $ 75,000
Explanation:
All of the expenditures related to the purchase of the new machine ($75,000 + $14,000) and the rearrangement of the assembly line to install this machine ($40,000 + $18,000) should be capitalized. The costs associated with the rearrangement of the assembly line are capitalized because the rearrangement has resulted in significantly more efficient production, the benefits of which extend beyond the current period.
Dell Printing Co. incurred the following costs for one of its printing presses:
Purchase of collating and stapling attachment $84,000
Installation of attachment 36,000
Replacement parts for overhaul of press 26,000
Labor and overhead in connection with overhaul 14,000The overhaul resulted in a significant increase in production. Neither the attachment nor the overhaul increased the estimated useful life of the press. What amount of the above costs should be capitalized?
A
$0
B
$84,000
C
$120,000
D
$160,000
Explanation:
The collating and stapling attachment is an addition to the printing press and so its cost (including installation cost) should be capitalized. The overhaul resulted in a significant increase in the productivity of the printing press; therefore, its cost (replacement parts, labor and overhead) should also be capitalized. The total cost capitalized is $160,000 (i.e., $84,000 + $36,000 + $26,000 + $14,000).
In January, Vorst Co. purchased a mineral mine for $2,640,000 with removable ore estimated at 1,200,000 tons. After it has extracted all the ore, Vorst will be required by law to restore the land to its original condition at an estimated cost of $180,000. Vorst believes it will be able to sell the property afterwards for $300,000. During the year, Vorst incurred $360,000 of development costs preparing the mine for production and removed and sold 60,000 tons of ore. In its year-end income statement, what amount should Vorst report as depletion? A $135,000 B $144,000 C $150,000 D $159,000
Explanation: Purchase price of mine $2,640,000 Development costs to prepare mine for production 360,000 Estimated restoration costs 180,000 Estimated residual value (300,000) Depletion base of mine $2,880,000 Estimated tons of removable ore / 1,200,000 Depletion charge per ton $ 2.40 Tons sold in the year x 60,000 Depletion expense for the year $ 144,000
A depreciable asset has an estimated 15% salvage value. Under which of the following methods, properly applied, would the accumulated depreciation equal the original cost at the end of the asset's estimated useful life? Straight-line Double-declining balance Straight-line Double-declining balance A Yes Yes B Yes No C No Yes D No No
Explanation:
Under neither the straight-line method or the double-declining balance method of depreciation would the accumulated depreciation equal the original cost at the end of the asset’s estimated useful life. The straight-line method depreciates the cost less salvage value evenly over the estimated useful life of the asset. The double-declining balance method uses a rate of depreciation twice that of the straight-line rate, but the asset cannot be depreciated below the salvage value.
The following information pertains to the transfer of real estate pursuant to a troubled debt restructuring by Knob Co. to Mene Corp. in full liquidation of Knob’s liability to Mene.
Carrying amount of liability liquidated $150,000
Carrying amount of real estate transferred 100,000
Fair value of real estate transferred 90,000At what amount should Mene record the real estate transferred?
A
$60,000
B
$90,000
C
$100,000
D
$150,000
Explanation:
In a troubled debt restructuring, the creditor, Mene, records the assets and/or equity securities (real estate transferred) at fair value.
Sun Ltd. enters into a contract with a customer for equipment with unique specifications. Sun Ltd. and the customer develop the specifications for the equipment, which Sun communicates to Ralph industries, a supplier that the Sun contracts with to manufacture the equipment. Sun Ltd. also arranges to have Ralph industries deliver the equipment directly to the customer. Upon delivery of the equipment to the customer, the terms of the contract require Sun Ltd. to pay Ralph the price agreed to by Sun and Ralph for manufacturing the equipment. Sun and the customer negotiate the selling price, and the entity invoices the customer for the agreed-upon price with 30-day payment terms. Sun’s profit is based on the difference between the sales price negotiated with the customer and the price charged by Ralph. The contract between Sun and the customer requires the customer to seek remedies for defects in the equipment from the Ralph under the Ralph’s warranty. However, Sun is responsible for any corrections to the equipment required resulting from errors in specifications. In this transaction is Sun:
A A principal B An agent C A consignor D A dealer
A
Explanation:
To determine whether Sun’s performance obligation is to provide the specified goods or services itself (i.e. the entity is a principal) or to arrange for another party to provide those goods or services (i.e. the entity is an agent), the following is considered. Sun has promised to provide the customer with specialized equipment; however it has subcontracted the manufacturing of the equipment to a supplier. In determining whether Sun obtains control of the equipment before control transfers to the customer and whether Sun is a principal, the entity considers the below:
Sun is primarily responsible for fulfilling the contract. Although Sun subcontracted the manufacturing, it is ultimately responsible for ensuring that the equipment meets the specifications for which the customer has contracted.
Sun has inventory risk because of its responsibility for corrections to the equipment resulting from errors in specifications, even though Ralph has inventory risk during production and before shipment.
Sun has discretion in establishing the selling price with the customer, and the profit earned by Sun is an amount that is equal to the difference between the selling price negotiated with the customer and the amount to be paid to Ralph.
Sun’s consideration is not in the form of a commission.
Sun would conclude that its promise is to provide the equipment to the customer. On the basis of the above indicators Sun controls the equipment before it is transferred to the customer. Thus, Sun is a principal in the transaction and recognizes revenue in the gross amount of consideration to which it is entitled from the customer in exchange for the equipment
On July 1, Casa Development Co. purchased a tract of land for $1,200,000. Casa incurred additional costs of $300,000 during the remainder of the year in preparing the land for sale. The tract was subdivided into residential lots as follows: Lot Class Number of lots Sales price per lot A 100 $24,000 B 100 $16,000 C 200 $10,000Using the relative sales value method, what amount of costs should be allocated to the Class A lots? A $300,000 B $375,000 C $600,000 D $720,000
Explanation:
Under the relative sales value method, the total cost of the individual units purchased at a single lump-sum price should be allocated to the various units on the basis of their relative sales value. The total cost includes the purchase price of $1,200,000 plus the additional costs of $300,000 to prepare the land for sale.
Lot Class Number of lots Sales price per lot Total sales for class
A 100 $24,000 $2,400,000
B 100 16,000 1,600,000
C 200 10,000 2,000,000
Total $6,000,000Class A lots ($2,400,000) / Total sales value ($6,000,000) = relative Class A sales value (40%). The total cost ($1,500,000) x the Class A lots relative value percentage (40%) = costs allocated to Class A lots ($600,000).
A state government condemned Cory Co.’s parcel of real estate. Cory will receive $750,000 for this property, which has a carrying amount of $575,000. Cory incurred the following costs as a result of the condemnation:
Appraisal fees to support a $750,000 value $2,500
Attorney fees for the closing with the state 3,500
Attorney fees to review contract to acquire replacement property 3,000
Title insurance on replacement property 4,000What amount of cost should Cory use to determine the gain on the condemnation?
A
$581,000
B
$582,000
C
$584,000
D
$588,000
Explanation:
The amount of cost that should be used to determine the gain on condemnation is $581,000 (i.e., the property’’s carrying amount of $575,000 plus the $2,500 appraisal fee plus the $3,500 attorney fees for the closing with the state). The attorney fees to review the contract and the title insurance are costs of acquiring the replacement property.
Bayberry Co. has an asset with a cost of $200,000 and accumulated depreciation of $120,000. Driftwood Co. has an asset with a cost of $250,000 and accumulated depreciation of $160,000. Both assets have a fair value of $100,000. Bayberry and Driftwood find it mutually advantageous to exchange assets, and the exchange results in improved future cash flow for both companies. What amount, if any, is Bayberry’s gain on the exchange?
A $0 B $10,000 C $20,000 D $50,000
Explanation:
The correct answer is (C).
Non-monetary exchanges with commercial substance, use fair value to measure the transaction and treat as if two unrelated events have occurred. The realized gains / losses are recognized immediately. Bayberry Co. should recognize gain on the exchange at $20,000.
Steps
Exchange with Commercial Substance
FV is determinable
Calculations
Amount
1
Calculate realized gain or loss
FV of asset given up - CV of asset given up.
= $100,000 - $80,000
$20,000
2
Recognize the realized gain or loss
Recognize all the realized gain / loss (Irrespective of whether no boot involved, boot paid / received)
$20,000
3
Record the new asset
FV of the asset given up + cash paid (if any) - cash received (if any)
= $100,000
$100,000
The Journal entry is as follows:
Equipment (Driftwood) $100,000
Accumulated Depreciation $120,000
Equipment (Bayberry) $200,000
Gain on Exchange $20,000
A company has experienced operating losses from its appliances division for the past five years. The division is the lowest level of identifiable cash flows. Having determined the division is the lowest level of identifiable cash flows, the company’s next step in performing its impairment test is to
A
Perform a recoverability test on the carrying amount of the division’s assets
B
Reduce the carrying amount of the division’s assets to the amount of expected divisional cash flows
C
Adjust the carrying amount of the division’s assets to fair value
D
Adjust the carrying amount of the division’s assets to replacement value
Explanation:
The correct answer is (A).
Testing for impairment occurs when events or changes in circumstances indicate that the carrying amount of a long-lived asset or asset groups may not be recoverable. The impairment test is a one-step process:
An indefinite-lived intangible asset is impaired when the fair value is less than its carrying amount.
Goodwill is tested for impairment at least annually, using a one-step process, and the goodwill impairment test may be performed any time during the fiscal year, provided the test is performed at the same time every year.
To identify potential impairment, we compare the reporting unit’s fair value with its carrying amount, including goodwill i.e. performing a recoverability test on the carrying amount of the division’s assets
If the fair value exceeds its carrying amount, the reporting unit’s goodwill is considered not impaired.
If the carrying amount exceeds its fair value, then the Impairment Loss of the reporting unit recognized is calculated as Carrying Value – Fair Value.
In year 1, a company purchased equipment that cost $70,000. The equipment has a useful life of seven years and no salvage value. The company used the straight-line method to depreciate the equipment and reported $10,000 of depreciation expense in years 1 and 2. At the beginning of year 3, the company determines that the equipment will last for only three more years (five years total) and changes the depreciable life of the asset accordingly. What amount of depreciation expense should the company report in year 3?
A $10,000 B $14,000 C $16,667 D $22,000
Explanation:
The correct answer is (C)
Under the straight-line method of depreciation, the asset is supposed to be depreciated equally over its useful life. The formula for calculating deprecation is as under:
Depreciation every year = (Cost of asset – salvage value)/Estimated useful life
When the useful life of the equipment was decided to be 7 years, the depreciation every year was = (70,000-0)/7 = $10,000 per year.
However, any revision in the useful life of the asset is to be accounted for prospectively. Hence, the depreciation expense is to be recalculated on the revision of useful life.
Depreciation expense = (Cost – salvage value – accumulated depreciation) / Revised remaining useful life. Hence, revised depreciation expense = (70,000 – 0 – 20,000) / (5-2) = $16,667 per year.
Particulars $
Cost of equipment $70,000
Less: Depreciation per year (when useful life was 7 years)
First-year depreciation ($10,000)
Second-year depreciation ($10,000)
Written Down Value of asset at the beginning of 3rd year $50,000
Less: Depreciation per year (when useful life was revised to 5 years)
Third-year depreciation ($16,667)
Fourth-year depreciation ($16,666)
Fifth-year depreciation ($16,666)
Written Down Value of asset at the beginning of 5th year $0
Four years ago on January 2, Randall Co. purchased a long-lived asset. The purchase price of the asset was $250,000, with no salvage value. The estimated useful life of the asset was 10 years. Randall used the straight-line method to calculate depreciation expense. An impairment loss on the asset of $30,000 was recognized on December 31 of the current year. The estimated useful life of the asset at December 31 of the current year did not change. What amount should Randall report as depreciation expense in its income statement for the next year? A $20,000 B $22,000 C $25,000 D $30,000
Explanation:
Impairment is the condition that exists when the carrying amount of a long-lived asset exceeds its fair value. The amount of an impairment loss is the difference between an asset’s book and fair value. The new book value is depreciated over the remaining useful life. Subsequent reversal of a previously recognized impairment loss is prohibited. This is classified as a change in accounting estimate and is reported in the period of change. There are no pro forma reports for prior periods, and amounts reported in financial statements of prior periods are not restated. Randall should report $20,000 ($250,000 cost - $100,000 accumulated depreciation - $30,000 impairment loss = $120,000 remaining basis depreciated over 6 years of remaining life) as depreciation expense in its income statement for the next year.
\_\_\_\_\_\_\_\_\_\_\_\_\_\_\_\_ is(are) allocated to inventory by the application of depletion charges because these resources are subject to exhaustion through extraction. A Plant and equipment B Land C Timber D Copyright
Explanation:
Natural resources, such as mineral, gas, and oil deposits, and standing timber, are subject to exhaustion through extraction and their costs must be allocated to inventory by the application of depletion charges.
Cantor Co. purchased a coal mine for $2,000,000. It cost $500,000 to prepare the coal mine for extraction of the coal. It was estimated that 750,000 tons of coal would be extracted from the mine during its useful life. Cantor planned to sell the property for $100,000 at the end of its useful life. During the current year, 15,000 tons of coal were extracted and sold. What would be Cantor's depletion amount per ton for the current year? A $2.50 B $2.60 C $3.20 D $3.30
Explanation:
Assets are to be recorded at their acquisition cost. Acquisition cost is the cash price, or its equivalent, plus all costs reasonably necessary to bring it to the location and to make it ready for its intended use. Depletion refers to periodic allocation of acquisition costs of natural resources. A per-unit depletion rate is computed by dividing the acquisition cost of the natural resource (i.e., purchase price, and other development costs), less any estimated residual value, by the estimated number of units of the resource available for extraction.
Purchase price of coal mine $2,000,000
Plus: Mine preparation costs 500,000
Mine total acquisition cost $2,500,000
Less: Value at end of useful life (100,000)
Net depletable amount $2,400,000
Divided by: Estimated number of units available for extraction 750,000
Depletion amount per ton $ 3.20
Which of the following is an example of an asset that qualifies for interest cost capitalization?
A
Asset constructed or produced as a discrete project for sale or lease, such as a ship
B
Item of inventory routinely manufactured on a repetitive basis
C
Asset in use or ready for use
D
Asset not in use and not being prepared for use
Explanation:
Assets qualifying for interest capitalization include assets constructed or produced as discrete projects for sale or lease; for example, ships or real estate developments.
A company is constructing an asset for its own use. Construction began in the previous year. The asset is being financed entirely with a specific new borrowing. Construction expenditures were made in last year and this year at the end of each quarter. The total amount of interest cost capitalized in the current year should be determined by applying the interest rate on the specific new borrowing to the
A
Total accumulated expenditures for the asset in both years.
B
Average accumulated expenditures for the asset in both years.
C
Average expenditures for the asset in the current year.
D
Total expenditures for the asset in the current year.
B
Explanation:
The amount of interest cost to be capitalized is that portion of interest cost incurred during the asset’s acquisition periods that theoretically could have been avoided if expenditures for the asset had not been made. In this question, the amount of interest cost to be capitalized in the current year is determined by applying the interest rate on the specific new borrowing to the average accumulated expenditures for the asset in the previous year and current year.
Town Company purchased for $540,000 a warehouse building and the land on which it is located. The following data were available concerning the property: Current appraised value Seller's original cost Land $200,000 $140,000 Warehouse building 300,000 280,000 $500,000 $420,000 Town should record the land at A $140,000 B $180,000 C $200,000 D $216,000
Explanation:
When several dissimilar assets are purchased for a lump sum, the amount paid should be allocated to each asset on the basis of its relative fair value. Land cost = (Total cost of assets) × (FMV of land) / Total FMV. Land = $540,000 × $200,000 / $500,000 = $216,000.
A corporation issued debt to purchase 10 acres of land for development purposes. Expenditures related to this purchase are as follows:
Description Amount Purchase Price $1,000,000 Real estate taxes in arrears $15,000 Debt issuance costs $2,000 Attorney fee title search on land $5,000 The company should record its acquisition of the land in its financial statements at a value of
A $1,000,000 B $1,015,000 C $1,020,000 D $1,022,000
Explanation:
The correct answer is (C).
The company should record its acquisition of the land in its financial statements at a value of $1,020,000.
All costs necessary to acquire an asset and prepare it for its intended use may be included in the total cost of the asset. The Purchase price of $1,000,000 + Real estate taxes in arrears or $15,000 + Attorney fees of $5,000 were all necessary expenditures to buy the land.
Finance costs, such as debt issuance costs, are generally not capitalizable into the purchase price of an asset
For an involuntary conversion of a nonmonetary asset, in which event would an entity not be in compliance with the provisions of GAAP revenue recognition criteria?
A
If it recognizes a gain or loss on the involuntary conversion of a nonmonetary asset, even if the insurance or condemnation award proceeds are reinvested in a replacement nonmonetary asset
B
If it takes removal and clean-up costs into account when computing the gain or loss recognized on the involuntary conversion
C
If it takes into account other incidental costs incurred in the acquisition of replacement property when computing the gain or loss recognized on the involuntary conversion
D
None of the above
C
Explanation:
Incidental costs incurred in the acquisition of replacement property are capitalized as costs of acquiring the replacement property. The incidental costs do not affect the gain or loss recognized due to the involuntary conversion.
Finch Co. reported a total asset retirement obligation of $257,000 in last year's financial statements. This year, Finch acquired assets subject to unconditional retirement obligations measured at undiscounted cash flow estimates of $110,000 and discounted cash flow estimates of $68,000. Finch paid $87,000 toward the settlement of previously recorded asset retirement obligations and recorded an accretion expense of $26,000. What amount should Finch report for the asset retirement obligation in this year's balance sheet? A $238,000 B $264,000 C $280,000 D $306,000
Explanation:
An asset retirement obligations (ARO) must be recorded at fair value in the accounting period in which it occurs and in which its amount can be reasonably measured. If an active market for the ARO doesn’t exist to provide fair value, the expected present value technique using discounted cash flows is the best way to determine the fair value of the asset. AROs incur depreciation and accretion expenses each year. Accretion expense is offset with an increase to the liability account, and, at the end of the asset’s life, the liability account will have a balance equal to the amount needed to settle the retirement obligation. So the amount reported for the asset retirement obligation in this year’s balance sheet would be the starting amount of $257,000 from last year’s financial statements less $87,000 paid towards settlement of previously recorded AROs plus the $68,000 cash flow estimate for newly acquired AROs plus the $26,000 accretion expense. $257,000 - $87,000 + $68,000 + $ 26,000 = $264,000 .
Which of the following is true regarding depreciation?
A
Depreciation accounting recognizes only the physical decline of an asset.
B
Property, plant, and equipment are reduced by depreciation, and then, written up to reflect appraisal, market, or current values above cost.
C
The depreciation process matches the depreciable cost of the asset with revenues generated from its use.
D
None of the above.
C
Explanation:
Depreciation is the process of allocating the depreciable cost of fixed assets over their estimated useful lives in a systematic and rational manner. This process matches the depreciable cost of the asset with revenues generated from its use.
When should a long-lived asset be tested for recoverability?
A
When external financial statements are being prepared.
B
When events or changes in circumstances indicate that its carrying amount may not be recoverable.
C
When the asset’s carrying amount is less than its fair value.
D
When the asset’s fair value has decreased, and the decrease is judged to be permanent.
B
Explanation:
Recoverability is associated with an impairment loss. An impairment loss is recognized only if the carrying amount of a long-lived asset is not recoverable and exceeds its fair value. The carrying amount (book value) is not recoverable if it exceeds the sum of the undiscounted cash flows expected to result from the use and eventual disposition of the asset. That assessment shall be based on the carrying amount of the asset at the date it is tested for recoverability. A long-lived asset (asset group) shall be tested for recoverability whenever events or changes in circumstances indicate that its carrying amount may not be recoverable.
On July 1, one of Rudd Co.'s delivery vans was destroyed in an accident. On that date, the van's carrying amount was $2,500. On July 15, Rudd received and recorded a $700 invoice for a new engine installed in the van in May, and another $500 invoice for various repairs. In August, Rudd received $3,500 under its insurance policy on the van, which it plans to use to replace the van. What amount should Rudd report as gain (loss) on disposal of the van in its year-end income statement? A $1,000 B $ 300 C $0 D $ (200)
B
Explanation:
A gain or loss on the involuntary conversion (e.g., casualty, condemnation, theft) of a nonmonetary asset should be recognized in income even if the proceeds received as a result of the involuntary conversion are reinvested in a replacement nonmonetary asset. The normal maintenance performed on the van (i.e., the various repairs) should not be capitalized. Normal maintenance does not enhance the service potential of the van, it serves only to maintain a given level of services from the van. On the other hand, the cost of the new engine should be capitalized because the expenditure for the new engine is nonrecurring in nature, enhances the service potential of the van, and is expected to yield benefits over a number of accounting periods.
Insurance proceeds $3,500
Carrying amount of van at date of conversion:
Carrying amount, 7/1 $2,500
Cost of new engine installed prior to involuntary conversion 700 3,200
Gain recognized on involuntary conversion $ 300
Sea Manufacturing Corp. is constructing a new factory building. During the current calendar year, Sea made the following payments to the construction company: January 2 $1,000,000 December 31 1,000,000Sea has an 8%, three-year construction loan of $3,000,000. What is the amount of interest costs that Sea may capitalize during the current year? A $0 B $80,000 C $160,000 D $240,000
Explanation:
The cost of assets constructed for the use of the business should include all directly related costs, such as direct materials, direct labor, and additional overhead incurred. Interest costs incurred during the construction period must be capitalized. The amount of interest cost to be capitalized is the interest cost incurred during the acquisition period that could have been avoided if expenditures for the asset had not been made. If a specific interest rate is associated with the asset, that rate should be used to the extent that the average accumulated expenditures on the asset do not exceed the amount borrowed at the specific rate. The interest rate is applied to the average amount of accumulated expenditures for the asset during the period. The average accumulated expenditure is computed as [($0 + $2,000,000) /2] = $1,000,000. Then $1,000,000 × 8% interest = $80,000 capitalized interest cost for the current year. The total interest cost capitalized in a period may not exceed the total interest cost incurred during that period. With an 8% three-year construction loan of $3,000,000 Sea incurs $80,000 of interest costs each year.
What factor must be present to use the units-of-production (activity) method of depreciation?
A
Total units to be produced can be estimated.
B
Production is constant over the life of the asset.
C
Repair costs increase with use.
D
Obsolescence is expected.
A
Explanation:
The units-of-production depreciation method allocates the cost of plant assets on the basis of units produced (i.e., activity). Depreciation for a period is computed by multiplying the number of units produced during the period by the amount of depreciation per unit. The amount of depreciation per unit is determined by dividing the depreciable base of the plant asset (i.e., cost minus estimated salvage value) by the estimated number of total units to be produced. Thus, an estimate of total units to be produced must be available to use this method.
On January 2 of the current year, Cruises, Inc. borrowed $3 million at a rate of 10% for three years and began construction of a cruise ship. The note states that annual payments of principal and interest in the amount of $1.3 million are due every December 31. Cruises used all proceeds as a down payment for construction of a new cruise ship that is to be delivered two years after start of construction. What should Cruise report as interest expense related to the note in its income statement for the second year? A $0 B $300,000 C $600,000 D $900,000
A
Explanation:
None of the interest is expensed; it is all capitalized. Assets qualifying for interest capitalization include assets constructed or produced for self-use on a repetitive basis, assets acquired for self-use through arrangements requiring down payments or progress payments, and assets constructed or produced as discrete projects for sale or lease (e.g., ships or real estate developments).
On January 1 ten years ago, Andrew Co. created a subsidiary for the purpose of buying an oil tanker depot at a cost of $1,500,000. Andrew expected to operate the depot for 10 years, at which time it is legally required to dismantle the depot and remove underground storage tanks. It was estimated that it would cost $150,000 to dismantle the depot and remove the tanks at the end of the depot’s useful life. However, the actual cost to demolish and dismantle the depot and remove the tanks in the tenth year is $155,000. What amount of expense should Andrew recognize in its financial statements in year 10?
A None, recognized in prior years B $5,000 expense C $150,000 expense D $155,000 expense
B
Explanation:
Obligations for dismantlement, restoration, and abandonment costs are accounted for as asset retirement obligations. Upon initial recognition of a liability for an asset retirement obligation, an entity capitalizes an asset retirement cost by increasing the carrying amount of the related long-lived asset by the same amount as the liability. An entity subsequently allocates that asset retirement cost to expense using a systematic and rational method over its useful life. Application of a systematic and rational allocation does not preclude an entity from capitalizing an amount of asset retirement cost and allocating an equal amount to expense in the same accounting period. The company increased the carrying amount of the depot the $150,000 cost to dismantle and amortized it over the 10 years. In this case, only the extra $5,000 ($155,000 actual cost - $150,000 estimate) would be expensed in year 10.
Editor’s Note: The AICPA provided item (b) as the unofficial solution. The editors believe the company could have, and perhaps should have, chosen to expense the $150,000 evenly over 10 years in what is termed an accretion expense. If it had done so, the expense in year 10 would have been $20,000 (the $15,000 original allocation plus the additional $5,000 in actual cost). Given that $20,000 was not an answer option, one must assume that the company did not do so. Candidates need to prepare to answer questions with the best solution choice possible.
Turtle Co. purchased equipment on January 2, year 1, for $50,000. The equipment had an estimated five-year service life. Turtle's policy for five-year assets is to use the 200% double-declining depreciation method for the first two years of the asset's life, and then switch to the straight-line depreciation method. In its December 31, year 3, balance sheet, what amount should Turtle report as accumulated depreciation for equipment? A $30,000 B $38,000 C $39,200 D $42,000
B
Explanation:
On December 31, year 3 Turtle reports $32,000 + $6,000 = $38,000 as accumulated depreciation. Double-declining-balance is computed using twice the straight-line rate, which in this case is 40% (1/5 = 0.20; 0.20 x 2 = .40). In year 3, depreciation is computed by dividing the remaining life of 3 years = $6,000.
Book value beginning of year Rate Depreciation Expense Accumulated Depreciation Book Value End of Year
Year 1 $50,000 40% $20,000 $20,000 $30,000
Year 2 30,000 40% 12,000 32,000 18,000
Oak Co., a newly formed corporation, incurred the following expenditures related to land and building:
County assessment for sewer lines $ 2,500
Title search fees 625
Cash paid for land with a building to be demolished 135,000
Excavation for construction of basement 21,000
Removal of old building $21,000 less salvage of $5,000 16,000At what amount should Oak record the land?
A
$138,125
B
$153,500
C
$154,125
D
$175,625
C
Explanation:
Assets are to be recorded at their acquisition cost. Acquisition cost is defined as the cash price, or equivalent, plus all other costs reasonably necessary to make it ready for it’s intended use. The land is recorded at the $135,000 cash paid for the land plus the additional costs: the $2,500 for county assessment for sewer lines, the $625 for title search fees, and the $16,000 for removal of old building less salvage value ($21,000 - $5,000) = $154,125. The $21,000 excavation cost for construction of a basement is part of the cost of the new building, not the land.
Newt Co. sold a warehouse and used the proceeds to acquire a new warehouse. The excess of the proceeds over the carrying amount of the warehouse sold should be reported as a(an):
A
Reduction of the cost of the new warehouse.
B
Gain from discontinued operations, net of income taxes.
C
Part of continuing operations.
D
Gains from discontinued operations, gross of income taxes.
Explanation:
The correct answer is (C).
The sale of the warehouse by the company is in the ordinary course of business and its proceeds are used to acquire a new warehouse.
Therefore, the gain from the proceeds over the carrying value of the warehouse will be reported in the income from continuing operations under non-operating Items.
Option (A) is incorrect because the gain from the sale of the warehouse cannot be used to reduce the cost basis of the new warehouse.
Option (B) and (D) is incorrect because the sale was part of continuing operation, not from discontinued operations.
On January 1, Year 1, Bay Co. acquired a land lease for a 21-year period with no option to renew. The lease required Bay to construct a building in lieu of rent. The building, completed on January 1, Year 2, at a cost of $840,000, will be depreciated using the straight-line method. At the end of the lease, the building's estimated market value will be $420,000. What is the building's carrying amount in Bay's December 31, Year 2, balance sheet? A $798,000 B $800,000 C $819,000 D $820,000
A
Explanation:
The building constructed on the leased property in lieu of rent represents a leasehold improvement because the building will revert to the lessor at the end of the lease term. The $840,000 cost of the building is allocated equally over the remaining 20-year period of the lease, resulting in annual amortization of $42,000. Thus, the building’s carrying amount at the end of the building’s first year is $798,000 ($840,000 cost minus $42,000 amortization to date).
A company using the composite depreciation method for its fleet of trucks, cars and campers, retired one of its trucks and received cash from a salvage company. The net carrying amount of these composite asset accounts would be decreased by the
A
Cash proceeds received and original cost of the truck.
B
Cash proceeds received.
C
Original cost of the truck less the cash proceeds.
D
Original cost of the truck.
B
Explanation:
Under the composite or group method of depreciation, no gain or loss is recognized upon the retirement of a plant asset. This practice is justified because some assets will be retired before the average service life and others after the average service life. Accumulated depreciation is debited for the difference between original cost and the cash received; no gain or loss is recorded on the disposition. The net carrying amount of these composite asset accounts is decreased by the cash proceeds received of $3,000 (cost removed of $18,000 minus accumulated depreciation removed of $15,000).
Cash 3,000
Accumulated Depreciation 15,000
Truck 18,000To record sale of truck
Spiro Corp. uses the sum-of-the-years'-digits method to depreciate equipment purchased in January of the current year for $20,000. The estimated salvage value of the equipment is $2,000 and the estimated useful life is four years. What should Spiro report as the asset's carrying amount as of December 31 in the third year? A $1,800 B $2,000 C $3,800 D $4,500
C
Explanation: Asset Cost $20,000 Year 1 [(4/10)($20,000 - 2,000)] $7,200 Year 2 [(3/10)($20,000 - 2,000)] 5,400 Year 3 [(2/10)($20,000 - 2,000)] 3,600 (16,200) Carrying amount 12/31, year 3 $ 3,800
Under IFRS, when an entity chooses the revaluation model as its accounting policy for measuring property, plant and equipment, which of the following is correct?
A
When an asset is revalued, the entire class of property, plant and equipment to which the asset belongs must be revalued.
B
When an asset is revalued, individual assets within a class of property, plant and equipment to which that asset belongs can be revalued.
C
Revaluations of property, plant and equipment must be made at least every three years.
D
Increases in an asset’s carrying value as a result of the first revaluation must be recognized as a component of profit or loss.
A
Explanation:
Under IFRS, revaluation is a permitted accounting policy if fair value can be reliably measured (i.e., the revaluation model).
At the beginning of the year, Cann Co. started construction on a new $2 million addition to its plant. Total construction expenditures made during the year were $200,000 on January 2, $600,000 on May 1, and $300,000 on December 1. On January 2, the company borrowed $500,000 for the construction at 12%. The only other outstanding debt the company had was a 10% interest rate, long-term mortgage of $800,000, which had been outstanding the entire year. What amount of interest should Cann capitalize as part of the cost of the plant addition? A $140,000 B $132,000 C $ 72,500 D $ 60,000
C
Explanation:
The cost of assets constructed for the use of the business should include all directly related costs; cost of direct materials, cost of direct labor, additional overhead incurred, and interest costs incurred during the construction period. If the average accumulated expenditures of an asset exceed the amount of any specific borrowings associated with the asset, the excess should be capitalized at the weighted average of interest rates applicable to other borrowings of the business.
Average expenditure during year $200,000 x 12/12 $ 200,000 600,000 x 8/12 400,000 300,000 x 1/12 25,000 Average expenditures $ 625,000 Specific borrowings 500,000 x 12% $ 60,000 Excess expenditures 125,000 x 10% 12,500 Total capitalized interest $ 72,500
Birk Co. purchased 30% of Sled Co.'s outstanding common stock on December 31 of the current year for $200,000. On that date, Sled's stockholders' equity was $500,000, and the fair value of its identifiable net assets was $600,000. On December 31, what amount of goodwill should Birk attribute to this acquisition? A $0 B $20,000 C $30,000 D $50,000
B
Explanation:
Goodwill is recognized and recorded at an amount equal to the excess of the cost of the enterprise acquired over the fair value of the identifiable net assets (INA).
Purchase price of 30% of Sled Co’’s O/S common stock $200,000
Fair value of Sled’s INA $600,000
Times: Percentage acquired by Birk x 30%
Less: Sled’s INA acquired by Birk (180,000)
Goodwill $ 20,000
Lee Corp. reported the following marketable debt security on its December 31 previous year balance sheet, classified as an available-for-sale debt security:
Neu Corp. Bond, at cost $100,000
Market adjustment to reflect decline in fair value (20,000)
Balance $ 80,000
On December 31 of the current year, the fair value of Lee’s investment in the Neu Corp. stock was $85,000. Assuming that no credit losses are expected, as a result of the current year increase in the debt security’s fair value, Lee’s current year income statement should report
A An unrealized gain of $5,000 B A realized gain of $5,000 C An unrealized loss of $15,000 D No gain or loss
Explanation:
The correct answer is (D).
On 12/31 of the previous year, the required balance of the Market Adjustment–AFS account was a $20,000 credit ($100,000 - $80,000). On 12/31 of the current year, the required balance of the Market Adjustment–AFS account is a $15,000 credit ($100,000 - $85,000). Therefore, in the current year, the required balance of the Market Adjustment–AFS account decreased by $5,000. Changes in the Market Adjustment account related to the available-for-sale category are included in other comprehensive income. Lee does not recognize any gain or loss from a change in the Market Adjustment–AFS account on its income statement.
On January 1, Year 1, Fring Chicken Co. purchased a bond with a face value of $1000, maturing after 7 years, and classified as an available-for-sale debt security on its balance sheet. The coupon rate on the bond was stated to be 8%, payable semi-annually on June 30 and December 31. On January 1, Year 5, the fair value of the said bond was estimated to be $650. What is the impact of the loss on the financial statements of Fring? Assume a discount rate of 10%.
Given:
Present Value of $1 at 5% and n=14: 0.505
Ordinary Annuity of $1 at 5% and n=14: 9.8986
Present Value of $1 at 5% and n=6: 0.746
Ordinary Annuity of $1 at 5% and n=6: 5.0757
Income Statement Other Comprehensive Income
A 0 350
B 350 0
C 96 203.14
D 203.14 96
Explanation:
The correct answer is (D).
Impairment for Available-for-Sale Securities is calculated as the difference between Amortized Cost (i.e. Carrying Value) and the Fair Value of the security. However, Credit Losses on the Income Statement are limited to Amortized Cost – Present Value (calculated the same way as a Held-to-Maturity investment) because if the unrealized loss at any given time is more than the expected credit loss till maturity, the Investor can minimize the loss by holding the security.
Excess losses are charged to OCI.
Carrying Value of the Bond on January 1, Year 1:
⇒ $1,000 x Present Value of $1 at 5% for 14 periods + $40 x Present Value of Ordinary Annuity $1 at 5% for 14 periods
⇒ $1,000 x 0.505 + $40 x 9.8986
⇒ $505 + $395.94
⇒ $900.94
Carrying Value of the Bond on January 1, Year 5 (after 8 Periods):
Period Interest Income (5%) Interest Received (4%) Interest Received (4%) Carrying Value 0 $900.94 1 $45.05 40 $5.05 $905.99 2 $45.30 40 $5.30 $911.29 3 $45.56 40 $5.56 $916.86 4 $45.84 40 $5.84 $922.70 5 $46.13 40 $6.13 $928.83 6 $46.44 40 $6.44 $935.27 7 $46.76 40 $6.76 $942.04 8 $47.10 40 $7.10 $949.14 In the given case, Present value of the bond on January 1, Year 5 (after 8 Periods): ⇒ $1,000 x Present Value of $1 at 5% for 6 years ⇒ 1,000 x 0.746 ⇒ $746
Expected Credit Loss = Carrying Value – Fair Value = $949.14 - $650 = $299.14
Expected Credit loss = $949.14 - $746 = $203.14.
Loss Charged to Income Statement = $299.14 - $203.14 = $203.14
Loss Charged to OCI = $96
Pal Corp.’s current year dividend income included only part of the dividend received from its Ima Corp. investment. The balance of the dividend reduced Pal’s carrying amount for its Ima investment. This reflects that Pal accounts for its Ima investment by the
A
Fair method, and only a portion of Ima’s current year dividends represent earnings after Pal’s acqui-sition
B
Fair value method, and its carrying amount exceeded the proportionate share of Ima’s market value
C
Equity method, and its carrying amount exceeded the proportionate share of Ima’s market value
D
Equity method, and Ima incurred a loss in the current year
A
Explanation:
Dividend income is not recognized under the equity method of accounting for investments in common stock. Under this method, the investor’s share of dividends declared by the investee reduce the carrying amount of the investment. Pal recognized dividend income from the investment in the current year. Hence it must account for the investment by the fair method. Under this method, investment income reported for the year is usually the investor’s share of dividends declared by the investee during the year. The exception is where the investor’s share of dividends declared by the investee exceeds the investor’s share of investee earnings subsequent to the date of the investment. In this case, the excess amount represents a liquidating dividend that is recorded as a reduction of the carrying amount of the investment and not as dividend income. Since only part of the dividends that Pal received in the current year was recorded as dividend income, only a portion of the current year dividends represent earnings subsequent to the date of Pal’s investment (i.e., the balance of the current year dividends represent liquidating dividends).
Options (B), (C) and (D) are incorrect as per above explanation.
A marketable debt security is transferred from the trading portfolio to the available-for-sale portfolio. At the transfer date, the security’s cost exceeds its market value. What amount is used at the transfer date to record the security in the available-for-sale debt portfolio?
A
Market value, regardless of whether the decline in market value below cost is considered permanent or temporary.
B
Market value, only if the decline in market value below cost is considered permanent.
C
Cost, if the decline in market value below cost is considered temporary.
D
Cost, regardless of whether the decline in market value below cost is considered permanent or temporary.
A
Explanation:
If there is a change in the classification of a marketable debt security between trading and available-for-sale, the security is transferred between the corresponding portfolios at the fair value at the date of transfer. If fair value is less than cost, the fair value becomes the new basis, and the difference is accounted for as if it were a realized loss and included in the determination of net income.
Moss Corp. owns 20% of Dubro Corp.’s preferred stock and 40% of its common stock. Dubro’s stock outstanding on December 31, 20X1, is as follows:
10% cumulative preferred stock $100,000
Common stock $700,000
Dubro reported net income of $60,000 for the year ending December 31, 20X1. What amount should Moss record as equity in earnings of Dubro for the year ending December 31, 20X1?
A $22,000 B $24,000 C $24,200 D $25,000
A
Explanation:
When calculating an investor’s income share, deduct the cumulative preferred dividends from net income first..
Moss’ share of preferred dividends
= 20% x 10% x $100,000
= $2,000
Earnings available to common shareholders
= $60,000 - 10% x $100,000
= $50,000
Moss’ share of common earnings
= 40% x $50,000
= $20,000
Moss’ total equity in Dubro earnings
= $2,000 + $20,000
= $22,000
FASB ASC 323-10-35-16 states: “If an investee has outstanding cumulative preferred stock, an investor shall compute its share of earnings (losses) after deducting the investee’s preferred dividends, whether or not such dividends are declared.”
Option (B), (C) and (D) are incorrect as per above explanation.
Which of the following factors would not be an indicator of an investor’s ability to exercise significant influence over the operating and financial policies of an investee?
A
Investor recommendation for the investee to hire a specific executive
B
Interchange of managerial personnel between investor and investee
C
Investor representation on the investee board of directors
D
Dependence by the investee on the investor’s proprietary technology
Explanation:
The correct answer is (A).
Investee has a significant technological or transactional dependency on the investor.
Investor has representation on the board of directors of the investee company.
Investor is a major customer or supplier of the investee.
Material inter-company transactions between the investor and investee company.
Investor company’s participation in the operating or financial policy-making decisions of the investee company.
Interchange of managerial personnel between the investor and investee.
Investor recommendation for the investee to hire a specific executive does not imply exercising significant influence.
At year-end, Rim Co. held several investments with the intent of selling them in the near term. The investments consisted of $100,000, 8%, five-year bonds, purchased for $92,000, and equity securities purchased for $35,000. At year-end, the bonds were selling on the open market for $105,000 and the equity securities had a market value of $50,000. What amount should Rim report as trading securities in its year-end balance sheet?
A $ 50,000 B $105,000 C $92,000 D $155,000
Explanation:
The correct answer is (B).
Trading securities are the debt securities bought and held principally for the purpose of selling them in the near term. Trading debt securities are carried at market value. Rim Co. investments in bonds will be classified as trading debt securities and reported on the balance sheet at $105,000.
(A) is incorrect because investment in equity securities should be classified as “Fair value through net income” if the market value of securities is readily determinable. There is no longer a trading or available for sale classification for equity securities.
(C) is incorrect because it uses cost instead of market value to report the debt securities purchased.
(D) is incorrect because it includes the market value of both debt and equity securities.
Cobb Co. purchased 10,000 shares (2% ownership) of Roe Co. on February 12 of the current year. Cobb received a stock dividend of 2,000 shares on March 31, when the carrying amount per share on Roe's books was $35 and the market value per share was $40. Roe paid a cash dividend of $1.50 per share on September 15. In Cobb's income statement for the year ended October 31, what amount should Cobb report as dividend income? A $98,000 B $88,000 C $18,000 D $15,000
Explanation:
Cobb owns only a 2% interest in Roe. Thus, Cobb does not have the ability to exercise significant influence over Roe by virtue of the investment, and the investment should be accounted for under the cost method. Therefore, Cobb should report the cash dividend received from Roe as dividend income. No income is recognized from the receipt of the stock dividend from Roe, since Cobb’s proportionate interest in Roe has not changed and Roe’s underlying assets and liabilities have also not changed.
Shares of Roe purchased 2/12 10,000
Add: Shares of Roe received from stock dividend, 3/31 2,000
Shares of Roe held, 9/15 12,000
Times: Cash dividend per share x 1.50
Dividend income, year ended 10/31 $18,000
At the beginning of the fiscal year, End Corp. purchased 25% of Turf Co. for $550,000. At the end of the fiscal year, Turf reported net income of $65,000 and declared and paid cash dividends of $30,000. End uses the equity method of accounting. At year end, what amount should End report in its balance sheet for the investment in Turf?
A $550,000 B $558,750 C $566,250 D $573,750
B
Explanation:
End Corp. has a holding of 25% in Turf Co. and uses the equity method to report its investment in Turf Co.
Initial purchase amount $ 550,000
Share of income ($65,000 × 25%) 16,250
Share of dividends ($30,000 × 25%) __(7,500)
Escrow liability end of year $ 558,750
Option (A) is incorrect because it is the cost of the initial investment, which should be adjusted for proportionate share in net income which would increase the investment account and dividends received at proportionate share of 25% which would reduce the investment account.
Option (C) is incorrect because it should also adjust for dividends received at 25% proportionate share.
Option (D) is incorrect because dividends received is added to the investment; instead of deducting it.
Puff Co. acquired 40% of Straw, Inc.'s voting common stock on January 2 of the current year for $400,000. The carrying amount of Straw's net assets at the purchase date totaled $900,000. Fair values equaled carrying amounts for all items except equipment, for which fair values exceeded carrying amounts by $100,000. The equipment has a five year life. During the year, Straw reported net income of $150,000. What amount of income from this investment should Puff report in its year-end income statement? A $40,000 B $52,000 C $56,000 D $60,000
xplanation:
If fair value exceeds the net assets’ carrying value, the excess is allocated among most undervalued assets. Then consideration is given to the existence of goodwill. As the related undervalued asset is depreciated or sold, the investment account is adjusted. Straw’s fair value is $900,000 + $100,000 = $1,000,000. Puff’s investment in Straw is 40% x $1,000,000 = $400,000, the same as the purchase price. Puff offsets its share of Straw’s income with the depreciation of its share of the difference in the carrying and fair value of equipment. [$150,000 - ($100,000 / 5 years)] x 40% = $52,000.
On January 1 of the current year, Lean Co. made an investment of $10,000. The following is the present value of $1.00 discounted at a 10% interest rate: Periods Present value of $1.00 Discounted at 10% 1 0.909 2 0.826 3 0.751What amount of cash will Lean accumulate in two years? A $12,000 B $12,107 C $16,250 D $27,002
B
Explanation:
The future value factor is equal to 1 divided by the present value factor. An investment in two years would accumulate to the principal multiplied by the future value factor. In this case the $10,000 x 1/0.826 = $12,107.
Editors Note: This question appears exactly as it was received from the AICPA. The AICPA provided $12,107 (derived as explained) as the correct answer. The question stated that Lean Co. made an investment on $10,000 on Jan 1 of the current year and provided no further details. Also provided is a three year table for the present value of $1.00 discounted at a 10% interest rate. In lieu of other information, you have to assume the prevailing rate of interest is 10% and you are to use the table provided.
Recording a credit loss on a debt security will result in which of the following?
A Lower retained earnings B Increase in retained earnings C Higher working capital D Increased current assets
Explanation:
The correct answer is (A).
Recording expected credit losses cause a decrease in the earnings of an entity since the credit loss expense is reflected in the Income Statement.
Consequently, the indirect effect is lower retained earnings due to a lower net income.
When the equity method is used to account for investments in common stock, which of the following affect(s) the investor's reported investment income? A change in market value of investee's common stock Cash dividends from investee A Yes Yes B Yes No C No Yes D No No
D
Explanation:
Under the equity method of accounting for investments in common stock, the investment is recorded at cost. Changes in the market value of the investee’s common stock do not affect the Investment account or the Investment Income account. The investor recognizes as income its share of the investee’s earnings or losses in the periods in which they are reported by the investee. Dividends declared by the investee represent a distribution of earnings previously recognized and, thus, do not affect the Investment Income account.
On March 1, Year 1, Max Inc. bought $200,000 face value 12% US treasury bonds for $200,000 which includes accrued interest. The interest on these is payable semi-annually on June 30 and December 31. These bonds are held to maturity, which is June 30, Year 2. What will be the carrying value of the bond as on December 31, year 1 if the straight-line method of amortization is used?
A $198,500 B $196,000 C $193,500 D $200,000
Explanation:
The correct answer is (A).
The bonds purchased included accrued interest.
The interest component in the purchase price of the bond will be the interest for two months (January and February, year 1), which will be $4,000 (i.e. $200,000 x 12% x 2 /12).
This means that the purchase price of the bond is $196,000 (i.e. $200,000 - $4,000).
The discount of $4,000 will be amortized for 16 months (from March 1, year 1 to June 30, year 2).
The discount amortized for 10 months, which will be added to the carrying value of the bond for year 1, will be $2,500 (i.e. $4,000 x 10 /16).
The carrying value of the bond at the end of year 1 will be $198,500 (i.e. $196,000 + $2,500).
(b) is incorrect because this will be the result when the discount is not amortized and the bond is reported at the purchase price, which is $196,000.
(c) is incorrect because this will be the result when discount amortized is reduced from the purchase price of the bond, which will be $193,500 (i.e. $196,000 - $2,500).
(d) is incorrect because this will be the result when accrued interest is completely ignored and the discount is not considered.
Janson traded bonds in Flax Co. held as trading debt securities during year 1 as follows:
Number of Bonds purchased (sold) Price per Bond
February 3, year 1
1,100
$11
April 15, year 1
2,500
9
May 28, year 1
(750)
13
July 5, year 1
1,400
12
September 30, year 1
(4,000)
15
No other transactions took place for Flax during the remainder of the year. At December 31, year 1, Flax is trading at $10 per bond. Janson trades debt securities on a last-in, first-out basis. What amount is the net value of the investment in Flax at year-end?
A $250 B $2,500 C $2,750 D $3,750
Explanation:
The correct answer is (B).
Trading securities are the debt securities bought and held principally for the purpose of selling them in the near term. The trading securities are carried at market value. Flax Co. investments which are held as trading securities and reported on the balance sheet at $2,500 (i.e. 250 per bond x $10).
(A) is incorrect because 250 bonds are valued at $1 instead of valuing at a market price of $10.
(C) is incorrect because 250 bonds are valued at $11 instead of valuing at market price of $10 (i.e. $2,750 = 250 x $11).
(D) is incorrect because 250 bonds are valued at $15 instead of valuing at market price of $10 (i.e. $3,750 = 250 x $15).
On January 2 of the current year, Emme Co. sold equipment with a carrying amount of $480,000 in exchange for a $600,000 noninterest bearing note due on January 2 in three years. There was no established exchange price for the equipment. The prevailing rate of interest for a note of this type at January 2 of the current year was 10%. The present value of 1 at 10% for three periods is 0.75.
In Emme’s current year income statement, what amount should be reported as gain (loss) on sale of machinery?
A ($ 30,000) loss. B $ 30,000 gain. C $120,000 gain. D $270,000 gain.
A
Explanation:
A noninterest-bearing note exchanged for property, goods, or services should not be recorded at its face amount. Since there is not an established exchange price for the equipment and the question does not indicate the fair value of the note, the note should be recorded at its present value, which is computed by discounting all future payments of the note at the prevailing rate of interest for a note of this type.
Present value of note ($600,000 x .75) $ 450,000
Carrying amount of machinery (480,000)
Loss on sale of machinery $ (30,000)
On July 1 of the current year, Denver Corp. purchased 3,000 shares of Eagle Co.'s 10,000 outstanding shares of common stock for $20 per share. On December 15, Eagle paid $40,000 in dividends to its common stockholders. Eagle's net income for the year ended December 31 was $120,000, earned evenly throughout the year. In its year-end income statement, what amount of income from this investment should Denver report? A $36,000 B $18,000 C $12,000 D $ 6,000
Explanation:
This investment should be accounted for under the equity method because Denver’s purchase of 30% (i.e., 3,000 / 10,000) of Eagle’s common stock on 7/1 gives Denver the ability to exercise significant influence over the operating and financial policies of Eagle by virtue of the investment. Denver should recognize investment income only for its share of Eagle’s net income subsequent to the date of the investment. While dividends declared by Eagle reduce the carrying amount of the investment, they do not affect the amount of investment income that Denver recognizes. Therefore, in the year, Denver should report income from the equity method investment of $18,000 (i.e., $120,000 x 6/12 x 30%).
In its financial statements, Pare, Inc. uses the cost method of accounting for its 15% ownership of Sabe Co. At December 31, Pare has a receivable from Sabe. How should the receivable be reported in Pare’s December 31 balance sheet?
A
The total receivable should be reported separately.
B
The total receivable should be included as part of the investment in Sabe, without separate disclosure.
C
Eighty-five percent of the receivable should be reported separately, with the balance offset against Sabe’s payable to Pare.
D
The total receivable should be offset against Sabe’s payable to Pare, without separate disclosure.
A
Explanation:
The total receivable from Sabe should be separately reported in Pare’s financial statements since Sabe is not a subsidiary of Pare (i.e., Pare has only a 15% interest in Sabe). Pare would have to own a majority voting interest (i.e., > 50%) in Sabe in order for Sabe to be considered a subsidiary.
On January 1 of the current year, Point, Inc. purchased 10% of Iona Co.’s common stock. Point purchased additional shares bringing its ownership up to 40%
of Iona’s common stock outstanding on August 1. During October, Iona declared and paid a cash dividend on all of its outstanding common stock. How much
income from the Iona investment should Point’s year-end income statement report?
A
10% of Iona’s income for January 1 to July 31, plus 40% of Iona’s income for August 1 to December 31
B
40% of Iona’s income for August 1 to December 31 only
C
40% of Iona’s total year income
D
Amount equal to dividends received from Iona
A
Explanation:
On 1/1, Point purchased 10% of Iona’s common stock. On 8/1, when Point increased its investment in Iona’s common stock from 10 percent to 40 percent,
Point gained the ability to exercise significant influence over the financial and operating policies of Iona and accordingly should report its investment using
the equity method. The change from the cost method of reporting the investment to the equity method should be made by prospectively.
In year 1, Flynn estimated its credit loss on an available-for-sale debt security to be $900. In the subsequent year, the estimated loss was found to be only $600. How should this change be accounted for?
A Increasing the value of the debt security by $300 B Decreasing the loss allowance by $300 C Writing off the original allowance and creating a new allowance of $600 D Accounting it as a prior period event
Explanation:
The correct answer is (B).
Any decrease in expected loss shall call for a decrease in the allowance, thereby causing a reversal of the credit loss expense on the income statement.
The decrease in the loss of $300 should be accounted for by decreasing the pre-existing loss allowance.
Band Co. uses the equity method to account for its investment in Guard, Inc. common stock. How should Band record a 2% stock dividend received from Guard?
A
As dividend revenue at Guard’s carrying value of the stock.
B
As dividend revenue at the market value of the stock.
C
As a reduction in the total cost of Guard stock owned.
D
As a memorandum entry reducing the unit cost of all Guard stock owned.
Explanation:
Stock dividends are recorded as memorandum entries only, reducing the unit cost of the stock owned. No dividend revenue is recorded and the total cost of the stock owned remains the same.
A company has a 22% investment in another company that it accounts for using the equity method. Which of the following disclosures should be included in the company’s annual financial statements?
A
The names and ownership percentages of the other stockholders in the investee company.
B
The reason for the company’s decision to invest in the investee company.
C
The company’s accounting policy for the investment.
D
Whether the investee company is involved in any litigation.
Explanation:
When an entity accounts for an investment in common stock under the equity method, it is required to disclose the names of each of its investee with its ownership percentage in the investee companies, the company’s accounting policies for investment in common stock, and the difference between the carrying value of the investment and the entity’s underlying equity in the investee’s net assets and the accounting treatment of the difference. Entity is not required to disclose the reason for having made the investment, the names and ownership percentage of the other stockholders in the investee company, or whether the investee company is involved in any litigation.
Options (A), (B) and (D) are incorrect based on the above explanation.
On January 2 of the current year, Well Co. purchased 10% of Rea, Inc.'s outstanding common shares for $400,000. Well is the largest single shareholder in Rea, and Well's officers are a majority on Rea's board of directors. Rea reported net income of $500,000 for the year and paid dividends of $150,000. In its December 31 balance sheet, what amount should Well report as investment in Rea? A $450,000 B $435,000 C $400,000 D $385,000
Explanation:
Well should use the equity method to account for its investment in Rea because it can exercise significant influence over the operating and financial policies of Rea.
Purchase price, 1/1 $400,000
Add: Share of Rea’’s income ($500,000 x 10%) 50,000
Less: Share of dividends paid ($150,000 x 10%) (15,000)
Investment carrying amount, 12/31 $435,000
Leaf Co. purchased from Oak Co. a $20,000, 8%, 5-year note that required five equal annual year-end payments of $5,009. The note was discounted to yield a 9% rate to Leaf. At the date of purchase, Leaf recorded the note at its present value of $19,485. What should be the total interest revenue earned by Leaf over the life of this note? A $5,045 B $5,560 C $8,000 D $9,000
Explanation:
Total interest revenue earned over the life of the note is determined as the excess of the summation of the required annual year-end payments over the present value of the note.
Summation of required annual year-end payments ($5,009 x 5) $25,045
Less: Present value of note (19,485)
Interest revenue earned over life of note $ 5,560 q
Park Co. uses the equity method to account for its January 1 current year purchase of Tun, Inc.'s common stock. On this date, the fair values of Tun's FIFO inventory and land exceeded their carrying amounts. How do these excesses of fair values over carrying amounts affect Park's reported equity in Tun's current year earnings? Inventory excess Land excess A Decrease Decrease B Decrease No effect C Increase Increase D Increase No effect
B
Explanation:
The excess of the fair value of Tun’s FIFO inventory over its carrying amount would decrease Park’’s reported equity in Tun’s earnings and the excess of the fair value of Tun’’s land over its carrying amount would have no effect on Park’s reported equity in Tun’s earnings.
At the beginning of year 2, a company invested $40,000 in a marketable equity security. At that time the security was appropriately classified security with readily determinable market value. At the end of year 2, the security had a fair value of $28,500. The change in fair value is deemed temporary. How should this change in fair value be reported in the financial statements?
A
As a realized loss of $11,500 as part of net income.
B
As a realized loss of $11,500 as part of other comprehensive income.
C
As an unrealized loss of $11,500 as part of net income.
D
As an unrealized loss of $11,500 as part of other comprehensive income.
Explanation:
The correct answer is (C)
As per FASB issued accounting standards update 2016-01, investment in marketable equity securities should be measured at fair value through net income (FVTNI).
All type of changes in fair value of equity securities whether temporary or permanent needs to be routed through net income only. Also, there is no longer classification of equity investments as trading or Available For Sale (AFS), and there is no longer a requirement to recognize unrealized holding gains and losses on equity securities in other comprehensive income as previously required.
(A) is incorrect because loss is not realized and hence should not be reported as realized loss as part of net income.
(B) & (D) are incorrect because unrealized and realized are both types of losses that should be reported as part of net income, not as a part of other comprehensive income.
At the end of year 1, Lane Co. held trading debt securities that cost $86,000 and which had a year-end market value of $92,000. During year 2, all of these securities were sold for $104,500. At the end of year 2, Lane had acquired additional trading debt securities that cost $73,000 and which had a year-end market value of $71,000. What is the impact of these stock activities on Lane’s year 2 income statement?
A Loss of $2,000 B Gain of $10,500 C Gain of $16,500 D Gain of $18,500
Explanation:
The correct answer is (B).
Trading debt securities that are bought and held principally for the purpose of selling them in the near term.
They are reported at fair value (market value) and any unrealized holding gains and losses are included in current earnings.
At the end of year 1, the trading debt securities would have been valued at $92,000 and a gain of $6,000 ($92,000 year-end market value - $86,000 cost) would have been reported on the income statement.
In year 2, there would be a gain of $12,500 ($104,500 selling price - $92,000 beginning value) from the initial set of securities and a $2,000 loss ($71,000 year-end market value - $73,000 cost) from the newly acquired trading debt securities.
The net impact of these stock activities would be a $10,500 gain on the income statement in year 2.
On January 2 of the current year, Otto Co. purchased 40% of Penn Co.’s outstanding common stock. The carrying amount of Penn’s depreciable assets was $1,000,000 on January 2. Penn’s depreciable assets had an original useful life of 10 years, and a remaining useful life of five years. Otto recognized $8,000 amortization for the current year ending December 31 related to its investment in Penn due to the excess of fair value over book value on these assets. What was the fair value of Penn’s depreciable assets on January 2 of the current year?
A $100,000 B $900,000 C $1,000,000 D $1,100,000
Explanation:
Otto recognized $8,000 amortization for the current year due to the excess of fair value over book value of these assets. The depreciable assets had a remaining useful live of 5 years, which indicates Otto would recognize a total of $40,000 ($8,000 × 5) amortization over the 5 years. Otto was only responsible for 40% of the total of the excess in fair value over book value, so the total fair value over book value must have been $100,000 ($40,000 ÷ 40%). The total fair value would have been $1,100,000 ($1,000,000 book value + $100,000 excess) on January 2 of the current year.
Bang Inc. acquired 40% stake in Boom Inc. for $120,000 in the beginning of year 1. None of the other investors have more than 20% stake in Boom Inc. The book value and fair value of Boom Inc. is $200,000 and $250,000 respectively. The difference in the book value and fair value is attributable to higher fair value of equipment by $30,000 and land by $20,000. The equipment is depreciated over next 10 years using straight line method. Goodwill is also impaired by 10% during the year and the land is also sold by Boom Inc. If Boom Inc. declares a dividend of $10,000 out of the total earnings of $80,000, what would be the investment income (or charge) recorded in the statement of income of Bang Inc. for the year concerning the investment in Boom Inc?
A $28,800 B $32,800 C ($11,200) D $20,800
D
Explanation:
Bang Inc. holds more than 40% stake in Boom Inc. and also further no other group of shareholders has any majority ownership and exercise total control, the investment will be accounted using equity method in the books of Bang Inc. Under equity method following journal entries will be passed:
At the time of purchase of investment (Recording investment at cost):
Investment in Boom Inc. $120,000
Cash $120,000
Recording dividend income:
Cash (i.e. 40% of $10,000) $4,000
Investment in Boom Inc. $4,000
Recording percentage of earnings:
Investment in Boom Inc. $32,000
Equity in earnings $32,000
Write off excess purchase price paid over book value of Boom Inc:
Excess of purchase price over book value = $120,000 – (40% x $200,000) = $40,000.
Of $40,000, $20,000 [i.e. 40% of (30,000 + 20,000)] is attributable to higher fair values of land and equipment. Thus, balance $20,000 will be the goodwill. Impairment of goodwill = 10% of $20,000 = $2,000.
Depreciation on increased fair value of equipment (Bang Inc’s share) = 40% x $30,000 / 10 years = $1,200.
No depreciation is charged on land. Land is sold during the year. Bang Inc. will write off its share of increased fair value of land, i.e. $8,000 (i.e. $20,000 x 40%).
The journal entry would be:
Equity in earnings $11,200 (i.e. $2,000 + $1,200 + $8,000)
Investment in Boom Inc. $11,200
Thus, investment income recorded in the statement of income of Bang Inc. concerning investment in Boom Inc. for the year 1 would be $20,800 (i.e. $32,000 - $11,200).
During the previous year, Wall Co. purchased 2,000 shares of Hemp Corp. common stock for $31,500 as an equity investment. The market value of this investment was $29,500 at December 31 of the previous year. Wall sold all of the Hemp common stock for $14 per share on December 15 of the current year, incurring $1,400 in brokerage commissions and taxes. On the sale, Wall should report a realized loss of
A $3,500 B $4,900 C $2,900 D $1,500
Explanation:
The correct answer is (C).
The realized loss reported from the sale of the equity securities is determined as the difference between the proceeds received (i.e., the gross selling price of the shares less any brokerage commissions and taxes incurred in the sale) and the carrying value of the securities. Equity securities are recorded at fair value with unrealized gains and losses included in earnings.
Gross selling price of 2,000 share @ $14 $28,000
Less: brokerage commissions and taxes incurred (1,400)
Proceeds received from sale of securities 26,600
Carrying Value of Securities Sold ($31,500 - $2,000) (29,500)
Realized Loss in current year ($ 2,900)
Grant, Inc. acquired 30% of South Co.’s voting stock for $200,000 on January 2, year 3. Grant’s 30% interest in South gave Grant the ability to exercise significant influence over South’s operating and financial policies. During year 3, South earned $80,000 and paid dividends of $50,000. South reported earnings of $100,000 for the six months ended June 30, year 4, and $200,000 for the year ended December 31, year 4. On July 1, year 4, Grant sold half of its stock in South for $150,000 cash. South paid dividends of $60,000 on October 1, year 4.
Before income taxes, what amount should Grant include in its year 3 income statement as a result of the investment?
A $15,000 B $24,000 C $50,000 D $80,000
B
Explanation:
Grant acquired 30% of South Co. and exercises significant influence, it will account for this investment under equity method. The cost of investment on January 2, Year 3 is $200,000. This is increased by Grant’s share in South’s earnings for Year 3, in this case it is 30% of $80,000 income. This amounts to $24,000 equity in earnings of South Co, which is recognized as a revenue in the statement of income and increase in the investment value by that amount.
Option (A) is incorrect because $15,000 is Grant’s 30% share in $50,000 dividend declared by South. Grant accounts for this investment under equity method. This dividend would reduce the investment value and not be recognized as dividend income in the statement of income.
Option (C) is incorrect because $50,000 is the total dividend declared by South in Year 3.
Option (D) is incorrect because $80,000 is the total income earned by South in Year 3.
On January 1, Year 1, White Car Wash Co. purchased a bond with a face value of $1000, maturing after 7 years. The coupon rate on the bond was stated to be 8%, payable semi-annually on June 30 and December 31. On January 1, Year 5, it was estimated that Tuco, the bond issuer, will be unable to service the rest of the interest payments. However, Tuco promises to pay the principal due on maturity. How much credit loss should be recorded in the books of White? Assume a discount rate of 10%.
Given:
Present Value of $1 at 5% and n=14: 0.505 Ordinary Annuity of $1 at 5% and n=14: 9.8986 Present Value of $1 at 5% and n=6: 0.746 Ordinary Annuity of $1 at 5% and n=6: 5.0757 A $0 B $746 C $203 D $254
Explanation:
The correct answer is (C)
If it is estimated that all of the investments in Held-to-Maturity Securities (principal or/and interest) will not be collected. The difference between the Amortized Cost and Present Value of Interest and Principal that will be collected is recognized as a credit loss and the investment will now be reported at present value of the principal and interest that is expected to be collected. Amortized Cost of the Bond is the Carrying Value of the Bond.
Carrying Value of the Bond on January 1, Year 1:
⇒ $1,000 x Present Value of $1 at 5% for 14 periods + $40 x Present Value of Ordinary Annuity $1 at 5% for 14 periods
⇒ $1,000 x 0.505 + $40 x 9.8986
⇒ $505 + $395.94
⇒ $900.944
Carrying Value of the Bond on January 1, Year 5 (after 8 Periods):
Period Interest Income (5%) Interest Received (4%) Interest Received (4%) Carrying Value 0 $900.94 1 $45.05 40 $5.05 $905.99 2 $45.30 40 $5.30 $911.29 3 $45.56 40 $5.56 $916.86 4 $45.84 40 $5.84 $922.70 5 $46.13 40 $6.13 $928.83 6 $46.44 40 $6.44 $935.27 7 $46.76 40 $6.76 $942.04 8 $47.10 40 $7.10 $949.14 In the given case, Present value of the bond on January 1, Year 5 (after 8 Periods): ⇒ $1,000 x Present Value of $1 at 5% for 6 years ⇒ 1,000 x 0.746 ⇒ $746
White’s credit loss = $949.14 – $746 = $203.14
In the current year, Neil Co. held the following investments in common stock:
25,000 shares of B&K, Inc.’s 100,000 outstanding shares. Neil’s level of ownership gives it the ability to exercise significant influence over the financial and operating policies of B&K.
6,000 shares of Amal Corp.’s 309,000 outstanding shares.
During the year, Neil received the following distributions from its common stock investments:
Nov. 6 $30,000 cash dividend from B&K.
Nov. 11 $1,500 cash dividend from Amal.
Dec. 26 3% common stock dividend from Amal. The closing price of this stock on a national exchange was $15 per share.
What amount of dividend revenue should Neil report for the year?
A $ 1,500 B $ 4,200 C $ 31,500 D $ 34,200
A
Explanation:
Of the dividends received, only the $1,500 cash dividend from Amal should be reported as dividend revenue. Neil owns less than a 2% (i.e., 6,000 / 309,000) interest in Amal; thus, Neil does not have the ability to exercise significant influence over Amal by virtue of the investment, and the investment should be accounted for under the cost method. The investment in B&K should be accounted for under the equity method since Neil’s level of ownership gives it the ability to exercise significant influence over the financial and operating policies of B&K. Thus, the cash dividend from B&K should not be reported as dividend revenue; instead, it reduces the carrying amount of the investment in B&K that is reported in Neil’s balance sheet. No income is recognized from the receipt of the stock dividend from Amal since Neil’s proportionate interest in Amal has not changed and Amal’s underlying assets and liabilities have not changed.
If it is not practicable for an entity to estimate the fair value of a financial instrument, which of the following should be disclosed?
Information pertinent to estimating the fair value of the financial instrument The reasons it is not practicable to estimate fair value A I only B II only C Both I and II D Neither I nor II
C
Explanation:
If it is not practicable to estimate the fair value of a financial instrument, disclosures are required that include: (1) the information pertinent to estimating the fair value of that financial instrument, such as the carrying amount, effective interest rate, and maturity; and (2) the reasons why it is not practicable to estimate fair value.
At the beginning of year 2, a company invested $40,000 in a marketable debt security. At that time the security was appropriately classified as an available-for-sale debt security. At the end of year 2, the security had a fair value of $28,500. The change in fair value is deemed temporary. Assuming no expected credit losses, how should this change in fair value be reported in the financial statements?
A
As a realized loss of $11,500 as part of Net Income
B
As a realized loss of $11,500 as part of Other Comprehensive Income
C
As an unrealized loss of $11,500 as part of Net Income
D
As an unrealized loss of $11,500 as part of Other Comprehensive Income
Explanation:
D
Explanation:
Available-For-Sale (AFS) debt securities are accounted for at fair value, determined on the balance sheet date.
The excess of cost over fair value or fair value over cost for AFS debt securities is recorded as a credit or debit in a Market Adjustment account.
The offsetting unrealized gain or loss is excluded from earnings and should be reported in Other Comprehensive Income until realized.
The company would report the change in fair value as an unrealized loss of $11,500 as part of Other Comprehensive Income.
During the current year, Scott Corp. purchased marketable debt securities and classified them as available-for-sale. Pertinent data follow:
Security Cost Fair value at 12/31 D $ 36,000 $ 40,000 E 80,000 60,000 F 180,000 186,000 $296,000 $286,000 Scott appropriately carries these debt securities at fair value and has no expected credit losses. The amount of unrealized loss on these securities in Scott's current year income statement should be
A $20,000 B $14,000 C $10,000 D $0
Explanation:
The correct answer is (D).
Marketable debt securities classified as available-for-sale (AFS) are to be reported at fair value. The amount by which the aggregate cost of an AFS debt portfolio exceeds its fair value should be accounted for as a credit to the Market Adjustment-AFS account.
In addition, an amount equal to the balance of the Market Adjustment-AFS account should be reported separately within equity in accumulated other comprehensive income.
The balance in the Market Adjustment-AFS account at year-end is a $10,000 credit ($296,000 aggregate cost - $286,000 aggregate market value).
Any changes in the Market Adjustment-AFS account is recognized in other comprehensive income, not in net income.
January 1, Year 5, Elia Company sold a building which had a carrying amount of $350,000, receiving a $125,000 down payment and, as additional consideration, a $400,000 non interest-bearing note due on January 1, Year 8. There was no established exchange price for the building and the note had no ready market. The prevailing rate of interest for a note of this type at January 1, Year 5, was 10%. The present value of 1 at 10% for three periods is 0.75.What amount of interest income should be included in Elia’s Year 5 income statement?
A $0 B $30,000 C $35,000 D $40,000
B
Explanation:
The recorded value of the Note Receivable is $300,000 [400,000 x .75]. Interest income is $30,000 [300,000 x .10]
Alpha company should report the marketable debt securities that it has classified as trading at
A
Lower of cost or market, with holding gains and losses included in earnings.
B
Lower of cost or market, with holding gains included in earnings only to the extent of previously recognized holding losses.
C
Fair value, with holding gains included in earnings only to the extent of previously recognized holding losses.
D
Fair value, with holding gains and losses included in earnings.
D
Explanation:
When an investment is made in debt securities that are publicly traded, and the investment is not large enough to provide the investor with any significant influence over the investee, the accounting for the investment will depend on the below classifications: - Trading or Held-for-trading (HFT) securities. Available-for-sale (AFS) securities. Held-to-maturity (HTM) securities. Trading or Held-for-trading (HFT) securities are debt securities bought and held principally for the purpose of selling them in the near term. They are initially recorded at cost and then carried at Fair Market Value (FMV), as trading securities are marketable and there is intent to sell in the near term.Alpha company should report the marketable debt securities that it has classified as trading at Fair Market value and all holding or unrealized gains and losses are included in earnings.
Anchor Co. owns 40% of Main Co.’s common stock outstanding and 75% of Main’s noncumulative preferred stock outstanding. Anchor exercises significant influence over Main’s operations. During the current period, Main declared dividends of $200,000 on its common stock and $100,000 on its noncumulative preferred stock. What amount of dividend income should Anchor report on its income statement for the current period related to its investment in Main?
A $ 75,000 B $ 80,000 C $ 120,000 D $ 225,000
Explanation:
Dividend income on preferred shares is $75,000 ($100,000 x 75%). When the ownership of the common stock is between 20-50%, it implies that the investor has a significant voting influence over the investee and would apply the equity method. Under the equity method, dividends are recognized as a reduction of the investment and not as dividend income.
Option (B) is incorrect because $80,000 is shown as a reduction in the investment under the equity method.
Options (C) and (D) are incorrect as per the above explanation.
Sork Inc. has a portfolio of equity securities which it does not intend to sell in the near future. At the end of the year, the fair values of the securities exceeded their costs. How should Sork classify these securities and where should the unrealized gain for these securities be recognized?
Classification Recognition of Gain
A Trading Income Statement
B Available-for-Sale Other Comprehensive Income
C Held to Maturity Stockholder’s Equity
D Fair Value through Net Income Income Statement
D
Explanation:
Investments in equity securities with a readily determinable market value are to be classified as Fair Value Through Net Income (FVTNI) - i.e., measured at fair value with changes in the fair value recognized through net income (the AFS classification is no longer applicable where unrealized gains/losses were recognized as OCI).
Options (A) and (B) are incorrect because only debt securities are classified as trading or available-for-sale.
Option (C) is incorrect because only debt securities are classified as held-to-maturity.
Information pertaining to dividends from Wray Corp.’s common stock investments for the current year ended December 31 follows:
On September 8, Wray received a $50,000 cash dividend from Seco, Inc., in which Wray owns a 30% interest. A majority of Wray’s directors are also directors of Seco.
On October 15, Wray received a $6,000 liquidating dividend from King Co. Wray owns a 5% interest in King Co.
Wray owns a 2% interest in Bow Corp., which declared a $200,000 cash dividend on November 27, to stockholders of record on December 15, payable on January 5 of next year.
What amount should Wray report as dividend income in its income statement for the current year ended December 31?
A
$60,000
B
$56,000
C
$10,000
D
$4,000
D
Explanation:
Of the dividends listed, only the $4,000 (i.e., $200,000 × 2%) cash dividend receivable from Bow should be reported as dividend income. Wray owns only a 2% interest in Bow; thus, Wray does not have the ability to exercise significant influence over Bow by virtue of the investment. Therefore, the investment should be accounted for under the cost method. The cash dividend received from Seco should be recorded as a reduction of the carrying amount of the investment in Seco reported in Wray’s balance sheet because the investment should be accounted for under the equity method (i.e., Wray owns a 30% interest in Seco and a majority of Wray’s directors are also directors of Seco). The liquidating dividend received from King should be recorded as a reduction of the carrying amount of the investment in King reported in Wray’s balance sheet.
For an available-for-sale security transferred into the trading category, the portion of the unrealized holding gain or loss at the date of the transfer that has not been previously recognized in earnings shall be
A
Recognized in earnings immediately.
B
Amortized over the period to date of sale.
C
Transferred to other comprehensive earnings.
D
Deferred and recognized when the security is sold.
Explanation:
The correct answer is (A)
As long as a security is held for sale, all the unrealized holding gain/loss resulting from changes in the value of such a security shall be classified under Other Comprehensive Income. However, transferring the security into trading category requires the entire portion of the unrealized gain or loss previously recognized as OCI to be recognized in earnings.
Grant, Inc. acquired 30% of South Co.’s voting stock for $200,000 on January 2, year 3. Grant’s 30% interest in South gave Grant the ability to exercise significant influence over South’s operating and financial policies. During year 3, South earned $80,000 and paid dividends of $50,000. South reported earnings of $100,000 for the six months ended June 30, year 4, and $200,000 for the year ended December 31, year 4. On July 1, year 4, Grant sold half of its stock in South for $150,000 cash. South paid dividends of $60,000 on October 1, year 4.
In Grant’s December 31, year 3, balance sheet, what should be the carrying amount of this investment?
A $200,000 B $209,000 C $224,000 D $230,000
B
Explanation:
Grant acquired 30% of South Co. and exercises significant influence. It will account for this investment under equity method. The investment account for Year 3 will be as follows:
Cost of 30% investment as on January 2, Year 3
$200,000
Add: 30% share of profit for the year (i.e. $80,000 x 30%)
$24,000
Less: 30% share of dividends paid (i.e. $50,000 x 30%)
$15,000
Carrying amount as at December 31, Year 3
$209,000
Option (A) is incorrect because $200,000 is only the cost of Grant’s investment in South Co.
Option (C) is incorrect because $224,000 is the cost of Grant’s investment $200,000 and its proportionate share in South’s earnings for Year 3 $24,000. It does not account for a reduction in value for proportionate share in dividend distributed $15,000.
Option (D) is incorrect because $230,000 is arrived as cost of Grant’s investment $200,000 plus its 30% share in South’s earnings of $100,000 for the first 6 months of Year 4.
When the market value of an investment in securities exceeds its carrying amount, how should each of the following assets be reported at the end of the year?
Trading Marketable Securities Available-For-Sale Marketable Securities
A Market value Market value
B Carrying amount Carrying amount
C Carrying amount Market value
D Market value Carrying amount
A
Explanation:
Marketable securities classified as either trading or available-for-sale are to be accounted for at market.
After being held for 40 days, a 120-day 12% interest-bearing note receivable was discounted at a bank at 15%. The proceeds received from the bank equal A Maturity value less the discount at 12%. B Maturity value less the discount at 15%. C Face value less the discount at 12%. D Face value less the discount at 15%.
B
Explanation:
Determining the proceeds received from discounting a note receivable consists of three steps:
Determine the maturity value of the note. This amount is based on the face amount, the stated rate of interest, and the time to maturity of the note.
Apply the bank’s discount rate to the maturity value of the note to obtain the amount of the discount charged by the bank.
Subtract the discount charged by the bank from the maturity value of the note to obtain the proceeds received from the bank.
Sun Corp. had investments classified as trading debt securities costing $650,000. On June 30 of the current year, Sun decided to hold the investments indefinitely and accordingly reclassified them from trading to available-for-sale on that date. The investment’s fair value was $575,000 on December 31 of the previous year; $530,000 this June 30; and $490,000 on December 31 of the current year. Assuming no expect credit losses, what amount of loss from investments should Sun report in its current year income statement?
A $ 45,000 B $ 85,000 C $120,000 D $160,000
Explanation:
The correct answer is (A).
The transfer of a debt security from the trading category of investments should be accounted for at fair value.
The unrealized holding loss resulting from the decrease in market value while classified as trading debt securities is included in earnings.
As of the date of transfer, June 30 of the current year, the securities had experienced a decline in fair value of $45,000 ($575,000 - $530,000) from the end of the prior year.
Upon transfer to a different category of investment, this unrealized loss is required to be included in earnings. (The decline from $650,000 to $575,000 would have been included as an unrealized loss on the previous year’s income statement.)
CLOSE
Pare, Inc. purchased 10% of Tot Co.’s 100,000 outstanding shares of common stock on January 2 of the current year for $50,000. On December 31, Pare purchased an additional 20,000 shares of Tot for $150,000. There was no goodwill as a result of either acquisition, and Tot had not issued any additional stock during the year. Tot reported earnings of $300,000 for the year. What amount should Pare report in its December 31 balance sheet as investment in Tot?
A $170,000 B $230,000 C $200,000 D $290,000
Explanation:
The correct answer is (C).
When Pare purchased an additional 20,000 shares of Tot at 12/31, it increased its investment in Tot’s common stock from 10% to 30% [i.e., (10,000 shares + 20,000 shares) ÷ 100,000 shares = 30%].
On 12/31, Pare gained the ability to exercise significant influence over the financial and operating policies of Tot and accordingly should report its investment in Tot using the equity method in its 12/31 balance sheet.
The change from the cost method of reporting the investment in Tot to the equity method should be made by prospectively as of the date significant influence is acquired and going forward.
On the December 31 Balance Sheet would be the original investment of $50,000 plus the price paid for additional 20,000 shares - i.e. $150,000 - which will be equal to $200,000.
Pare will begin using equity method from January 1, Year 2 and proportionate earnings will be included from year 2 and not year 1.
Sage, Inc. bought 40% of Adams Corp.'s outstanding common stock on January 2 of the current year for $400,000. The carrying amount of Adams' net assets at the purchase date totaled $900,000. Fair values and carrying amounts were the same for all items except for plant and inventory, for which fair values exceeded their carrying amounts by $90,000 and $10,000, respectively. The plant has an 18-year life. All inventory was sold during the year. Goodwill, if any, will be tested for impairment each year. During the year, Adams reported net income of $120,000 and paid a $20,000 cash dividend. What amount should Sage report in its income statement from its investment in Adams for the year ended December 31? A $48,000 B $42,000 C $36,000 D $32,000
B
Explanation:
The common stock investment should be accounted for under the equity method since Sage has the ability to exercise significant influence over the operating and financial policies of Adams by virtue of the investment. To determine Sage’s reported amount of equity in income in Adams, the cost of the investment must first be allocated as follows:
Purchase price $ 400,000
Percentage of carrying amount acquired ($900,000 x 40%) (360,000)
Payment in excess of purchase price 40,000
Excess payment associated with specific assets:
Plant assets ($90,000 x 40%) 36,000
Inventory ($10,000 x 40%) (4,000)
Goodwill $ 0The amount that Sage should report in its income statement from its investment in Adams can now be determined as follows:
Interest in Adams net income ($120,000 x 40%) $ 48,000
Less: Amortization of excess payment associated with plant assets ($36,000 / 18) (2,000)
Excess payment associated with inventory sold (4,000)
Equity in income of Adams $ 42,000The cash dividends paid by Adams to Sage reduce the carrying amount of the investment in Sage’s balance sheet, they do not effect Sage’s reported amount of equity in income in Adams
On December 29, year 9, BJ Co. sold a marketable debt security that had been purchased on January 4, year 8. BJ owned no other marketable debt security. An unrealized loss was reported in other comprehensive income in year 8. A realized gain was reported in the year 9 income statement. How was the marketable debt security classified, and did its year 8 market price decline exceed its year 9 market price recovery?
Classification Year 8 market price decline exceeded the year 9 market price recovery A Trading Yes B Trading No C Available-For-Sale Yes D Available-For-Sale No
Explanation:
The correct answer is (D).
An unrealized loss in year 8 on investment in marketable debt securities classified as available-for-sale would be shown as an other comprehensive income item rather than as an income statement item.
The market price decline in the year 8 was less than the increase in market price during the year 9 because there was a realized gain reported on the year 9 income statement which means that the security was sold for a price exceeding its cost.
Thus, in year 9, the market price increased enough to recover the unrealized loss experienced in year 8 and to produce a gain on the sale of the investment.
An unrealized loss in year 8 on investment in marketable debt securities classified as trading debt securities would be reported in the income statement rather than in other comprehensive income.
Trading debt securities would be written to fair value at the end of year 8.
If a trading debt security did report a realized gain in the year 9 it would be indeterminable as to whether any market price declined or not in year 8.
A company should report the marketable debt securities that it has classified as trading at
A
Lower of cost or market, with holding gains and losses included in earnings.
B
Lower of cost or market, with holding gains included in earnings only to the extent of previously recognized holding losses.
C
Fair value, with holding gains included in earnings only to the extent of previously recognized holding losses
D
Fair value, with holding gains and losses included in earnings
D
Explanation:
Trading debt securities are reported at fair value and unrealized holding gains and losses are included in current earnings on the Income Statement.
How is a held-to-maturity security reported on the balance sheet when there are expected credit losses?
A Face Value B Amortized Cost C Present Value of Principal D Present Value of Principal and Interest
Explanation:
The correct answer is (D).
If it is estimated that not all of the investments in held-to-maturity securities (principal or/and interest) will be collected, the difference between the amortized cost and present value of Interest and principal that will be collected is recognized as a credit loss and the investment will be reported at present value of the principal and interest that is expected to be collected.
Which of the following securities is subject to the credit loss model of impairment?
A Bonds payable B Investment in common stock C Investment in preferred stock D Bonds receivable
Explanation:
The correct answer is (D).
With the CECL (Current Expected Credit Loss) model, Credit losses expected over the lifetime of a Held-to-Maturity and Available-for-Sale debt Security are to be recognized upon the initial recognition of the asset.
Bonds payable is a liability and not an asset. The CECL model is not applicable to it.
Investments in common and preferred stock are not debt securities.
Bonds receivable is a debt security that is generally held till maturity by the bondholder and can be impaired using the CECL model.
Grant, Inc. acquired 30% of South Co.’s voting stock for $200,000 on January 2, year 3. Grant’s 30% interest in South gave Grant the ability to exercise significant influence over South’s operating and financial policies. During year 3, South earned $80,000 and paid dividends of $50,000. South reported earnings of $100,000 for the six months ended June 30, year 4, and $200,000 for the year ended December 31, year 4. On July 1, year 4, Grant sold half of its stock in South for $150,000 cash. South paid dividends of $60,000 on October 1, year 4.
In its year 4 income statement, what amount should Grant report as gain from the sale of half of its investment?
A $24,500 B $30,500 C $35,000 D $45,500
Explanation:
The correct answer is (B).
Grant acquired 30% of South Co. and exercises significant influence. It will account for this investment under the equity method.
The investment account will be as follows for years Year 3 and Year 4:
Cost of 30% investment as on January 2, Year 3
$200,000
Add: 30% share of profit for the year (i.e. $80,000 x 30%)
$24,000
Less: 30% share of dividends paid (i.e. $50,000 x 30%)
$15,000
Carrying amount as at December 31, Year 3
$209,000
Add: 30% share of profit for six months ended June 30, Year 4 (i.e. $100,000 x 30%)
$30,000
Carrying value as of June 30, Year 4
$239,000
Carrying amount of 50% of investment (i.e. $239,000 x 50%)
$119,500
Proceeds from the sale of 50% of investment
$150,000
Profit on sale of investment
$30,500
Larkin Co. has owned 25% of the common stock of Devon Co. for a number of years, and has the ability to exercise significant influence over Devon. The following information relates to Larkin’s investment in Devon during the most recent year:
Carrying amount of Larkin’s investment in Devon at the beginning of the year $ 200,000
Net income of Devon for the year 600,000
Total dividends paid to Devon’s stockholders during the year 400,000
What is the carrying amount of Larkin’s investment in Devon at year end?
A $100,000 B $200,000 C $250,000 D $350,000
C
Explanation:
Larkin Co would apply equity method to account for its investments. The earnings from Devon Co., will increase the amount of its investment and dividend will reduce the investment account.
Ref
Equity Method
Amount ($)
a
Investment in Devon Co.
200,000
b
Net Income ($600,000 x 25%)
150,000
c
Dividend ($400,000 x 25%)
(100,000)
d
Investment reported at year end (a+b-c)
250,000
Option (A) is incorrect because net income’s proportionate share at 25% is not included, which would increase the investment account.
Option (B) is incorrect because it is the cost of the initial investment, which should be adjusted for proportionate share in net income which would increase the investment account and dividends received at proportionate share of 25% which would reduce the investment account.
Option (D) is incorrect because it should adjust for dividends received at 25% proportionate share which would reduce the investment account.
Long Co. invested in marketable debt securities. At year-end, fair-value changes in this investment were included in Long’s other comprehensive income. How would Long classify this investment?
A Held-to-maturity securities B Trading debt securities C Contra securities D Available-for-sale debt securities
Explanation:
The correct answer is (D).
Available-for-sale debt securities are initially recorded at cost and carried at the fair market value on the balance sheet and any unrealized gain/loss is reported in the statement of comprehensive income and cumulatively accumulates in the balance sheet.
(A) is incorrect because the debt securities that the entity has the positive intent and ability to hold to the maturity date are classified as Held-to-Maturity (HTM). The HTM securities are initially recorded at cost and carried at amortized cost.
(B) is incorrect because trading debt securities are the debt bought and held principally for the purpose of selling them in the near term. They are carried at market value and any unrealized gain/loss at year end are reported in the profit and loss statement.
(C) is incorrect because this is not a classification type.
Note: The updated standard doesn’t change the guidance for classifying and measuring investments in debt securities or loans.
During year 3, Gilman Co. purchased 5,000 shares of the 500,000 outstanding shares of Meteor Corp.’s common stock for $35,000. During year 3, Gilman received $1,800 of dividends from its investment in Meteor’s stock. The fair value of Gilman’s investment on December 31, year 3, is $32,000. Gilman has elected the fair value option for this investment. What amount of income or loss that is attributable to the Meteor stock investment should be reflected in Gilman’s earnings for year 3? A Income of $4,800 B Income of $1,800 C Loss of $1,200 D Loss of $3,000
c
Explanation:
Gilman’s earnings would reflect a loss of $1,200 attributable to the Meteor stock. In electing fair value option, the stock would be valued at $32,000 at December 31, year 3, which denotes a loss of $3,000 from the $35,000 purchase price of the stock during year 3. Gilman also received $1,800 in of dividends from the stock which would be income for year 3. The net between the two transactions is a loss of $1,200.
Wood Co. owns 2,000 shares of Arlo, Inc.’s 20,000 shares of $100 par, 6% cumulative, nonparticipating preferred stock, and 1,000 shares (2%) of Arlo’s common stock. During the current year, Arlo declared and paid dividends of $240,000 on preferred stock. No dividends had been declared or paid the previous year. Also, Wood received a 5% common stock dividend from Arlo when the quoted market price of Arlo’s common stock was $10 per share. What amount should Wood report as dividend income in its year-end income statement?
A $12,000 B $12,500 C $24,000 D $24,500
C
Explanation:
Wood owns 10% of Arlo’s preferred stock and would have received $24,000 ($240,000 dividends × 10%) in dividends in the current year on this preferred stock. Although this amount represents the dividend preference for the previous year, due to the 6% cumulative feature of the stock (2,000 shares x $100 x .06 = $12,000/year), as well as for the current year, the revenue is not recognized until the dividends are actually declared. In the current year the dividends for the current and prior year are both recognized. The 5% stock dividends received are not recognized as income because they are not a distribution, division, or severance of the corporate assets.
Kale Co. purchased bonds at a discount on the open market as an investment and intends to hold these bonds to maturity. Kale should account for these bonds at A Cost. B Amortized cost. C Fair value. D Lower of cost or market.
B
Explanation:
The debt securities that the entity has the positive intent and ability to hold to maturity date are classified as Held-to-Maturity (HTM). The HTM securities are initially recorded at cost and carried at amortized cost. As HTM are not going to be sold, fluctuations in market prices are ignored. The difference between cost and maturity value is amortized over the life of the security. Thus, Kale should account for these bonds at amortized cost. Option (a) is incorrect because HTM are initially recorded at cost, and later carried at amortized cost. Option (c) is incorrect because the fluctuations in market prices are ignored and accounted based on amortized cost not fair value. Option (d) is incorrect because inventory is reported at lower of cost or market not HTM securities which is at amortized cost.
Sun Corp. had investments in debt securities classified as trading debt securities costing $650,000. On June 30 of the current year, Sun decided to hold the investments indefinitely and accordingly reclassified them from trading debt securities to available-for-sale debt securities on that date. The investment’s fair value was $575,000 on December 31 of the previous year; $530,000 this June 30; and $490,000 on December 31 of the current year. Assuming no expected credit losses, what amount should Sun report as a net unrealized loss on investments in debt securities in other comprehensive income at the end of the current year?
A $40,000 B $45,000 C $85,000 D $160,000
Explanation:
The correct answer is (A).
Unrealized holding losses for available-for-sale (AFS) debt securities are excluded from earnings and reported in other comprehensive income until realized.
The decrease in fair value over the time period in which the securities were classified as trading debt securities would have been reported in earnings.
However, the loss occurring after the transfer to the AFS category, $40,000 ($530,000 - $490,000), would be reflected in OCI.
Ace Co. sold to King Co. a $20,000, 8%, 5-year note that required five equal annual year-end payments. This note was discounted to yield a 9% rate to King.
The present value factors of an ordinary annuity of $1 for five periods are as follows:
8% 3.992
9% 3.890
What should be the total interest revenue earned by King on this note?
A $9,000 B $8,000 C $5,560 D $5,050
C
Explanation:
The total amount of interest revenue to be earned by King over the life of the note is determined as the excess of the summation of the required annual year-end payments over the present value of the note discounted to yield 9% to King.
Summation of required annual year-end payments [5 × $5,010 (as determined below)] $ 25,050
Face amount of note $20,000
Divide by: PV factor of an ordinary annuity of $1 for 5 periods at 8% / 3.992
Required equal annual year-end payments under note $ 5,010
Times: PV factor of an ordinary annuity of $1 for 5 periods at 9% × 3.890
Less: present value of note (19,490)
Interest revenue to be earned over the life of the note’s life $ 5,560
On January 2, Kean Co. purchased a 30% interest in Pod Co. for $250,000. On this date, Pod's stockholders' equity was $500,000. The fair value of plant and equipment exceeded its carrying amount by $200,000. Plant and assets were determined to have a remaining 10 year life at December 31. There were no other under valued assets. Pod reported net income of $100,000 for the year, and paid no dividends. Kean accounts for this investment using the equity method. In its December 31, year-end balance sheet, what amount should Kean report as investment in subsidiary? A $210,000 B $220,000 C $270,000 D $280,000
D
Explanation:
Since Kean owns a 30% interest in Pod, the investment should be accounted for under the equity method. Under the equity method, the carrying amount of the investment is increased for Kean’s share of Pod’s income, decreased for Kean’s share of any dividends paid by Pod and decreased by the amortization of the excess payment associated with plant and equipment. To determine the carrying amount of Kean’s investment in Pod at 12/31, the original cost of the investment must first be allocated. The carrying amount of the investment at 12/31 is determined as follows:
On January 1, Year 1, Astor Co. purchased a bond with a face value of $1000, maturing after 7 years, and classified as an available-for-sale debt security on its balance sheet. The coupon rate on the bond was stated to be 8%, payable semi-annually on June 30 and December 31. On January 1, Year 5, the fair value of the said bond was estimated to be $800. What is the impact of the loss on the financial statements of Astor? Assume a discount rate of 10%.
Given:
Present Value of $1 at 5% and n=14: 0.505
Ordinary Annuity of $1 at 5% and n=14: 9.8986
Present Value of $1 at 5% and n=6: 0.746
Ordinary Annuity of $1 at 5% and n=6: 5.0757
Income Statement Other Comprehensive Income
A 0 200
B 100 100
C 149.14 0
D 203.14 0
Explanation:
The correct answer is (C)
Impairment for Available-for-Sale Securities is calculated as the difference between Amortized Cost (i.e. Carrying Value) and the Fair Value of the security. However, Credit Losses on the Income Statement are limited to Amortized Cost – Present Value (calculated the same way as a Held-to-Maturity investment) because if the unrealized loss at any given time is more than the expected credit loss till maturity, the Investor can minimize the loss by holding the security.
Excess losses will be charged to OCI.
Carrying Value of the Bond on January 1, Year 1:
⇒ $1,000 x Present Value of $1 at 5% for 14 periods + $40 x Present Value of Ordinary Annuity $1 at 5% for 14 periods
⇒ $1,000 x 0.505 + $40 x 9.8986
⇒ $505 + $395.94
⇒ $900.94
Carrying Value of the Bond on January 1, Year 5 (after 8 Periods):
Period Interest Income (5%) Interest Received (4%) Interest Received (4%) Carrying Value 0 $900.94 1 $45.05 40 $5.05 $905.99 2 $45.30 40 $5.30 $911.29 3 $45.56 40 $5.56 $916.86 4 $45.84 40 $5.84 $922.70 5 $46.13 40 $6.13 $928.83 6 $46.44 40 $6.44 $935.27 7 $46.76 40 $6.76 $942.04 8 $47.10 40 $7.10 $949.14 In the given case, Present value of the bond on January 1, Year 5 (after 8 Periods): ⇒ $1,000 x Present Value of $1 at 5% for 6 years ⇒ 1,000 x 0.746 ⇒ $746
Expected Credit Loss = Carrying Value – Fair Value = $949.14 - $800 = $149.14
Expected Credit loss = $949.14 - $746 = $203.14.
All of the loss is recognized on the Income Statement.
Under IFRS, what valuation methods are used for intangible assets?
A The cost model or the fair value model. B The cost model or the revaluation model. C The cost model or the fair value through profit or loss model. D The revaluation model or the fair value model.
B
Explanation:
Under IFRS, the cost model or the revaluation model is used to value intangible assets. Revaluation model may be used only for intangibles that are traded with active market prices (as revaluation model requires fair value determination from active market):
Cost Model CV = Cost - Accumulated Amortization (if finite life) -Accumulated Impairment.
Revaluation Model CV = Fair Value from active market @revaluation date -Subsequent Accumulated Amortization (if finite life intangibles) – Subsequent Accumulated Impairment.
Options (a), (c) and (d) are incorrect because the fair value model is not used
Ott Company acquired rights to a patent from Grey under a licensing agreement that required an advanced royalty payment when the agreement was signed. Ott remits royalties earned and due, under the agreement, on October 31 each year. Additionally, on the same date, Ott pays, in advance, estimated royalties for the next year. Ott adjusts prepaid royalties at year-end. Information for the current year ended December 31 is as follows: Date Amount 01/01 Prepaid royalties $ 65,000 10/31 Royalty payment (charged to royalty expense) $ 110,000 12/31 Year-end credit adjustment to royalty expense $ 25,000In its December 31 balance sheet, Ott should report prepaid royalties of A $25,000 B $40,000 C $85,000 D $90,000
D
Explanation:
The balance in the Prepaid Royalties account is $65,000 + $25,000 = $90,000.
On January 1, year 3, Vick Company purchased a trademark for $400,000, having an estimated useful life of 16 years. In January of year 7, Vick paid $60,000 for legal fees in a successful defense of the trademark. Trademark amortization expense for the year ended December 31, year 7, should be A $0 B $25,000 C $28,750 D $30,000
D
Explanation:
Trademarks are amortized over their estimated useful life. Legal fees in a successful defense are capitalized because they offer probable future benefits. They are amortized over the remaining useful life of the trademark.
Amortization of original cost ($400,000 /16 years) $25,000
Amortization of legal defense cost, ($60,000 / 12 remaining years) 5,000
Total amortization expense, year 7 $30,000
During the year, Jase Co. incurred research and development costs of $136,000 in its laboratories relating to a patent that was granted on July 1. Costs of registering the patent equaled $34,000. The patent's legal life is 17 years, and its estimated economic life is 10 years. In its December 31 balance sheet, what amount should Jase report as patent, net of accumulated amortization? A $ 32,300 B $ 33,000 C $161,500 D $165,000
A
Explanation:
Research and development costs are expensed as incurred. Only the costs of acquiring a patent should be capitalized. Thus, only the cost of registering the patent, $34,000, is capitalized. The capitalized cost of an intangible asset, is amortized over the asset’s economic life. One-half year of amortization is $1,700 ($34,000 / 10 years x 1/2 year). ($34,000 - $1,700) = $32,300.
On January 2, Paye Co. purchased Shef Co. at a cost that resulted in recognition of goodwill of $200,000. During the first quarter of the year, Paye spent an additional $80,000 on expenditures designed to maintain goodwill. In its December 31 balance sheet, what amount should Paye report as goodwill? A $80,000 B $195,000 C $200,000 D $280,000
C
Explanation:
The $200,000 of goodwill acquired in connection with the purchase of Shef should be capitalized. The $80,000 of expenditures to maintain the goodwill should be expensed when incurred because costs of developing, maintaining, or restoring goodwill should not be capitalized.
Under a royalty agreement with another company, Wand Co. will pay royalties for the assignment of a patent for three years. The royalties paid should be reported as expense A In the period paid. B In the period incurred. C At the date the royalty agreement began. D At the date the royalty agreement expired.
B
Explanation:
Accrual accounting recognizes expenses in the period they are incurred. Accrual accounting does not recognize expenses merely in the period they are paid. Royalties paid should not be recognized as an expense at the date the royalty agreement began or the date the royalty agreement expires.
After an impairment loss is recognized, the adjusted carrying amount of the intangible asset shall be its new accounting basis. Which of the following statements about subsequent reversal of a previously recognized impairment loss is correct?
A It is prohibited. B It is required when the reversal is considered permanent. C It must be disclosed in the notes to the financial statements. D It is encouraged, but not required.
A
Explanation:
An impairment loss is recognized if the carrying amount of an intangible asset is not recoverable and its carrying amount exceeds its fair value.After an impairment loss is recognized,the adjusted carrying amount of the asset is its new accounting basis.Subsequent reversal of a previously recognized impairment loss is prohibited under US GAAP. Option (b), (c) and (d) are incorrect because US GAAP does not allow reversal of impairment losses.
To be considered intangible, an asset must be which of the following? A Classified as identifiable B Without physical substance C Internally developed D All of the above
B
Explanation:
Intangible assets are assets without physical substance that provide economic benefits through the rights and privileges associated with their possession. Intangibles may be classified as identifiable or unidentifiable and externally acquired or internally developed.
A company is working on a direct response advertising campaign that will likely provide the company future benefits in the form of increased sales over the next two years. The company identified the following costs associated with the advertising campaign:
Catalogs on hand to be mailed to potential customers $100,000
Coupons printed to be mailed to existing customers 50,000
Employee salaries for call center support 45,000
Postage to be paid to mail the catalogs and coupons 25,000
What cost, if any, should be capitalized under IFRS?
A $220,000 B $150,000 C $100,000 D $0
Explanation:
The correct answer is (D)
An expenditure which is expected to yield revenue for more than one accounting year is an example of deferred revenue expenditure. IFRS does not permit capitalization of such expenditures. Hence, the entire expenditure of $220,000 needs to be posted on the Income Statement instead of being capitalized.
Ott Company acquired rights to a patent from Grey under a licensing agreement that required an advance royalty payment when the agreement was signed. Ott remits royalties earned and due, under the agreement, on October 31 each year. additionally, on the same date, Ott pays, in advance, estimated royalties for the next year. Ott adjusts prepaid royalties at year-end. Information for the current year ended December 31 is as follows:
Date
Amount
01/01
Prepaid royalties
$65,000
10/31
Royalty payment (charged to royalty expense)
$110,000
12/31
Year-end credit adjustment to royalty expense
$25,000
In its December 31 balance sheet, Ott should report prepaid royalties of
A $25,000 B $40,000 C $85,000 D $90,000
Explanation:
The correct answer is (D).
This is an Accounting Error question.
Ott follows an incorrect method of accounting.
On 1/1 The balance on Prepaid Royalties Account: $65,000.
Ott makes no adjustments to this account until the year-end.
The 12/31 balance on this account would still be $65,000. On 10/31, the following journal entry was recorded. This included advance for next year’s royalties.
However, under Ott’s system, all such payments are debited to royalty expense when paid, and any necessary adjustments to prepaid royalties are made at year-end.
Royalty Expense 110,000
Cash 110,000
On 12/31, the following journal entry was recorded:
Prepaid Royalty Expense 25,000
Royalty Expense 25,000
The balance in Prepaid Royalties would be $90,000 ($65,000 Opening Balance + $25,000 Current Year Prepaid).
On January 2, Year 1, Lava, Inc. purchased a patent for a new consumer product for $90,000. At the time of purchase, the patent was valid for 15 years; however, the patent's useful life was estimated to be only 10 years due to the competitive nature of the product. On December 31, Year 4, the product was permanently withdrawn from sale under governmental order because of a potential health hazard in the product. What amount should Lava charge against income during year 4, assuming amortization is recorded at the end of each year? A $9,000 B $54,000 C $63,000 D $72,000
c
Explanation:
The patented product was withdrawn from sale under governmental order. Therefore, the unamortized cost of the patent at December 31, year 4, should be charged to income.
Purchase price of patent $ 90,000
Less amortization recorded prior to year 4 ($90,000 x 3/10) (27,000)
Unamortized cost of patent charged to income in year 4 $ 63,000
On July 1, Year 1, Hart signed an agreement to operate as a franchise of Ace Printers for an initial franchise fee of $120,000. The same date, Hart paid $40,000 and agreed to pay the balance in four equal annual payments of $20,000 beginning July 1, Year 2. The down payment is not refundable and no future services are required of the franchiser. Hart can borrow at 14% for a loan of this type. Present and future value factors are as follows:
Present value of $1 at 14% for 4 periods 0.59
Future amount of $1 at 14% for 4 periods 1.69
Present value of an ordinary annuity of $1 at 14% for 4 periods 2.91Hart should record the acquisition cost of the franchise on July 1, Year 1, at
A
$135,200
B
$120,000
C
$ 98,200
D
$ 87,200
c
Explanation:
The acquisition cost of the franchise is determined as follows:
Cash payment, July 1, Year 1 $40,000
Add: Present value of periodic annual payments:
Periodic annual payments $20,000
Times: Present value of an ordinary annuity of 1 at 14% for 4 periods x 2.91 58,200
Franchise acquisition cost $98,200
Alta Co. spent $400,000 during the current year developing a new idea for a product that was patented during the year. The legal cost of applying for a patent license was $40,000. Also, $50,000 was spent to successfully defend the rights of the patent against a competitor. The patent has a life of 20 years. What related patent costs should Alta capitalize?
A $40,000 B $50,000 C $90,000 D $490,000
Explanation:
The correct answer is (C).
The cost of developing a new idea for a product should be expensed as R&D.
The legal cost for applying for the patent license and the amount spent to defend the patent from the competitor are capitalized $90,000 ($40,000 + $50,000)
(a) is incorrect because it does not capitalize the amount spent to successfully defend the rights of the patent.
(b) is incorrect because it does not capitalize the expenses incurred in applying for the patent license.
(d) is incorrect because it capitalizes the cost of developing a new idea for a product, which should be expensed
An entity purchases a trademark and incurs the following costs in connection with the trademark:
One-time trademark purchase price $100,000
Nonrefundable VAT taxes 5,000
Training sales personnel on the use of the new trademark 7,000
Research expenditures associated with the purchase of the new trademark 24,000
Legal costs incurred to register the trademark 10,500
Salaries of the administrative personnel 12,000
Applying IFRS and assuming that the trademark meets all of the applicable initial asset recognition criteria, the entity should recognize an asset in the amount of
A $100,000. B $115,500. C $146,500. D $158,500.
B
Explanation:
A trademark is an intangible asset. The cost of an intangible asset is the cash equivalent price and includes purchase price, nonrefundable taxes, and direct costs of preparing the intangible asset and bringing it to an appropriate condition for its intended use. In this case, the cash equivalent price would include the one-time trademark purchase price, the nonrefundable VAT taxes, and the legal costs incurred to register the trademark ($100,000 + $5,000 + $10,500 = $115,500). The training, research, and salary expenditures would be expensed as incurred.
Gray Co. was granted a patent on January 2, year 1, and appropriately capitalized $45,000 of related costs. Gray was amortizing the patent over its estimated useful life of fifteen years. During year 4, Gray paid $15,000 in legal costs in successfully defending an attempted infringement of the patent. After the legal action was completed, Gray sold the patent to the plaintiff for $75,000. Gray's policy is to take no amortization in the year of disposal. In its year 4 income statement, what amount should Gray report as gain from sale of patent? A $15,000 B $24,000 C $27,000 D $39,000
b
Explanation:
The cost of a successful legal defense of an existing patent is capitalized because it offers probable future benefits.
Original cost of patent (year 1) $ 45,000
Amortization (3 yrs. x $45,000/15) (9,000)
Carrying value 12/31, year 3 $ 36,000
Capitalization of legal costs 15,000
Carrying value of patent (year 4) $ 51,000
Proceeds from sale (year 1) $ 75,000
Carrying value of patent (51,000)
Gain from sale $ 24,000
Under IFRS, which of the following is a criterion that must be met in order for an item to be recognized as an intangible asset other than goodwill?
A
The item’s fair value can be measured reliably.
B
The item is part of the entity’s activities aimed at gaining new scientific or technical knowledge.
C
The item is expected to be used in the production or supply of goods or services.
D
The item is identifiable and lacks physical substance.
d
Explanation:
Under both IFRS and U.S. GAAP, an intangible asset is defined as identifiable nonmonetary assets without physical substance. The recognition criteria for both accounting models require that there be probable future economic benefits and its cost, not its fair value, can be reliably measured. The item need not be expected to be used in the production or supply of goods or services nor be part of the entity’s activities aimed at gaining new scientific or technical knowledge.
Birk Co. purchased 30% of Sled Co.’s outstanding common stock on December 31, 20X0, for $200,000. On that date, Sled’s stockholders’ equity was $500,000, and the fair value of its identifiable net assets was $600,000. On December 31, 20X0, what amount of goodwill should Birk attribute to this acquisition?
A $0 B $20,000 C $30,000 D $50,000
b
Explanation:
The difference between the purchase price paid to the investee and the fair value of its net assets is accounted as goodwill by the investor. On the date of purchase, the fair value of net assets is $600,000. The proportionate amount of the fair value of investment is $180,000 (i.e. 30% of $600,000). Hence, the goodwill amount will be $20,000 (i.e. $200,000 - $180,000). Option (a) is incorrect because goodwill is recorded at $20,000 and not $0. Option (c) is incorrect because it is taking the difference in the proportionate share of stockholders’ equity $150,000 (i.e. 30% of $500,000) with the fair value of investment is $180,000 (i.e. 30% of $600,000). Option (d) is incorrect because it is taking the difference in the proportionate share of stockholders’ equity $150,000 (i.e. 30% of $500,000) instead of using the proportionate share of fair value of the net assets $180,000 (i.e. 30% of $600,000).
Goodwill should be tested for value impairment at which of the following levels?
A Entire business as a whole B Each identifiable long-term asset C Each acquisition unit D Each reporting unit
D
Explanation:
Accounting for goodwill is based on reporting units as an aggregate view of goodwill. Goodwill is tested for impairment at a reporting unit level. A reporting unit is an operating segment or one level below an operating segment, referred to as a component. A component of an operating segment is a reporting unit if the component constitutes a business for which discrete financial information is available and segment management regularly reviews the operating results of that component.Option (a), (b) and (c) are incorrect as per the above explanation
Which of the following would cause an indefinite-lived intangible asset to have an impairment loss?
A
Carrying value is greater than fair value
B
Fair value is greater than carrying value
C
Carrying value plus salvage value is greater than fair value
D
Fair value plus salvage value is greater than carrying value
Explanation:
The correct answer is (A).
Testing for impairment occurs when events or changes in circumstances indicate that the carrying amount of a long-lived asset or asset groups may not be recoverable.
The impairment test is a one-step process:
An indefinite-lived intangible asset is impaired when the fair value is less than its carrying amount.
Goodwill is tested for impairment at least annually, using a one-step process, and the goodwill impairment test may be performed any time during the fiscal year, provided the test is performed at the same time every year.
To identify potential impairment, we compare the reporting unit’s fair value with its carrying amount, including goodwill i.e. performing a recoverability test on the carrying amount of the division’s assets
If the fair value exceeds its carrying amount, the reporting unit’s goodwill is considered not impaired.
If the carrying amount exceeds its fair value, then the Impairment Loss of the reporting unit recognized is calculated as Carrying Value – Fair Value i.e. $100,000,000 - $80,000,000 = $20,000,000.
Options C and D are incorrect because Carrying (and Fair) value plus salvage value is greater than fair (and carrying) value is a nonsense distractor.
During the current year, Orr Co. incurred the following costs:
Research and development services performed by Key Corp. for Orr $150,000
Design, construction, and testing of preproduction prototypes and models 200,000
Testing in search for new products or process alternatives 175,000In its current year income statement, what should Orr report as research and development expense?
A
$150,000
B
$200,000
C
$350,000
D
$525,000
d
Explanation:
All three activities are examples of activities that typically are included in research and development and should be expensed. $150,000 + $200,000 + $175,000 = $525,000.
Which of the following expenditures qualifies for asset capitalization?
A
Cost of materials used in prototype testing
B
Costs of testing a prototype and modifying its design
C
Salaries of engineering staff developing a new product
D
Legal costs associated with obtaining a patent on a new product
D
Explanation:
The external acquisition costs of a patent,which includes the legal costs associated with obtaining a patent on a new product,qualifies for asset capitalization. Cost of materials used in prototype testing, costs of testing a prototype and modifying its design,and salaries of engineering staff developing a new product are all examples of research and development costs.These research and development costs are not capitalized, but instead expensed in the year in which incurred.Costs incurred to legally protect product and process ideas resulting from R&D. Following costs are capitalized:
Costs include costs of patent application.
Costs of purchase if the patent is purchased from another party.
Costs incurred in successful defense of a patent if infringed during its economic life
Option (a), (b) and (c) are incorrect because these are R&D costs which are expensed when incurred
Under IFRS, which of the following statements about intangible assets is correct?
A
Internally generated goodwill cannot be recognized as an asset.
B
Intangible assets within a class may be measured differently using either the cost model or the revaluation model.
C
Research and development costs are capitalized as incurred.
D
Intangible assets with indefinite lives must be amortized annually.
Explanation:
The correct answer is (A)
According to IFRS, internally generated goodwill cannot be recognized as an asset. Goodwill is recognized only if it is externally acquired. Internally developed intangible assets under IFRS are normally recognized only if:
It is probable that the expected future economic benefits that are attributable to the asset will flow to the entity.
It is easy and clear to measure the cost of an asset.
(B), (C) and (D) are incorrect as per IFRS Accounting Standards
Grayson Co. incurred significant costs in defending its patent rights. Which of the following is the appropriate treatment of the related litigation costs?
A
Litigation costs would be capitalized regardless of the outcome of the litigation.
B
Litigation costs would be expensed regardless of the outcome of the litigation.
C
Litigation costs would be capitalized if the patent right is successfully defended.
D
Litigation costs would be capitalized only if the patent was purchased rather than internally developed.
C
Explanation:
Legal fees in a successful defense are capitalized because they offer probable future benefits. They are amortized over the remaining useful life of the patent.
The following information relates to two projects performed by Miley Co. during the year for laboratory research aimed at discovering new knowledge:
Project Costs Likelihood that effort will result in future benefits
$100,000 Probable
$50,000 Reasonably possible
What should Miley report as research and development expenses in its income statement for the year?
A $0 B $50,000 C $100,000 D $150,000
Explanation:
The correct answer is (D).
The R&D costs include:
New knowledge or new technology.
Model or prototype.
Application of new research findings.
Costs incurred for the two projects are expensed as incurred, regardless of the likelihood that efforts will result in future benefits. Miley should report R&D costs of $150,000 ($100,000 + $50,000).
(A) is incorrect because all laboratory research aimed at discovering new knowledge will be reported as R&D costs.
(B) is incorrect because Project I costs of $100,000 should also be expensed regardless of the high probability.
(C) is incorrect because Project II costs of $50,000 should be expensed as R&D costs.
A company recently acquired a copyright that now has a remaining legal life of 30 years. The copyright initially had a 38-year useful life assigned to it. An analysis of market trends and consumer habits indicated that the copyrighted material will generate positive cash flows for approximately 25 years. What is the remaining useful life, if any, over which the company can amortize the copyright for accounting purposes?
A 0 years. B 25 years. C 30 years. D 38 years
Explanation:
Copyrights are amortized over the shorter of useful or legal life. The market trends indicate that the copyrighted materials will generate positive cash flows for approximately 25 years which is shorter than the remaining legal life of 30 years.Thus, Company will amortize copyright for 25 years.Option (a) is incorrect because copyrights are amortized.Option (c) is incorrect because copyrighted material is amortized over 30 years of remaining legal life, while it should be amortized at 25 years useful life.Option (d) is incorrect because copyrighted material was initially amortized over 38years of legal life, now the useful life is reassessed at 25 years which is shorter than the remaining useful legal life, and amortized over 25years.
On January 2, Judd Co. bought a trademark from Krug Co. for $500,000. Judd retained an independent consultant, who estimated the trademark's remaining life to be 50 years. Its unamortized cost on Krug's accounting records was $380,000. In Judd's December 31 balance sheet, what amount should be reported as accumulated amortization? A $7,600 B $9,500 C $10,000 D $12,500
c
Explanation:
The $500,000 acquisition cost of the trademark is amortized over the useful life, resulting in accumulated amortization of $10,000 at 12/31. The unamortized cost on Krug’s books is irrelevant in determining Judd’s acquisition cost.
Which of the following types of assets would typically be reported on a company’s balance sheet as an intangible asset?
A Derivative securities B Cost of research and development C Leasehold improvements made by the lessor D Cost of patent registrations
Explanation:
The correct answer is (D).
Intangible assets are long-lived legal rights and competitive advantages developed or acquired by a business. Intangible assets do not have physical substance. Examples of intangible assets include patents, copyright, goodwill, trademark etc. Patents are costs incurred to legally protect product and process ideas resulting from research and development. Cost of patent registration is capitalized as patent and is reported in the balance sheet as an intangible asset as a patent does not have physical substance.
Which of the following is a true statement regarding the accounting and reporting requirements related to intangible costs under both IFRS and U.S. GAAP?
A
Amortization of intangible assets over their estimated useful lives is required under both U.S. GAAP and IFRS, with one minor exception in U.S. GAAP.
B
Under U.S. GAAP, development costs are capitalized unless addressed by a separate standard.
C
Under IFRS, advertising and promotional costs are capitalized.
D
Under both IFRS and U.S. GAAP, revaluation is not permitted.
A
Explanation: Amortization of intangible assets over their estimated useful lives is required under both U.S. GAAP and IFRS, with one minor exception in U.S. GAAP. Under U.S. GAAP, development costs are expensed as incurred, not capitalized, unless addressed by a separate standard. Under IFRS, advertising and promotional costs are expensed as incurred. Under U.S. GAAP, revaluation is not permitted. Under IFRS, revaluation to fair value of intangible assets other than goodwill is a permitted accounting policy election for a class of intangible assets.
Which of the following types of assets would typically be reported on a company’s balance sheet as an intangible asset?
A Derivative securities. B Cost of research and development. C Leasehold improvements made by the lessor. D Cost of patent registrations.
D
Explanation:
Intangible assets are long-lived legal rights and competitive advantages developed or acquired by a business. Intangible assets do not have physical substance.Examples of intangible assets includes patents, copyright, goodwill, trademark etc.Patents are costs incurred to legally protect product and process ideas resulting from research and development. Cost of patent registration is capitalized as patent and is reported in the balance sheet as an intangible asset as a patent does not have physical substance.
Option (a) is incorrect because derivative securities is not an intangible asset.Derivative securities usually takes the form of an agreement to buy or sell an asset or item (commodity, property, security) at a fixed price on or before a certain date.Derivative securities are traded on exchanges like other financial instruments, and their value varies with the value of the underlying assets. These have physical substance and are not intangibles.
Option (b) is incorrect because even though research and development (R&D) is an intangible, the cost of R&Dis typically expensed as a period cost and is not reported as an asset on the balance sheet. Research is a discovery of new knowledge for anew or significantly improved product or process while development is conversion of the knowledge into plan / design for application.Option (c) is incorrect because a leasehold improvement made by the lessor is capitalized as an asset and depreciated. Only a leasehold improvement made by the lessee is treated as an intangible asset in the lessee’s books.
On June 30, Union, Inc., purchased goodwill of $125,000 when it acquired the net assets of Apex Corp. During the year Union incurred additional costs of developing goodwill, by training Apex employees ($50,000) and hiring additional Apex employees ($25,000). Union's December 31 balance sheet should report goodwill of A $200,000. B $175,000. C $150,000. D $125,000.
D
Explanation:
Only the purchased goodwill of $125,000 from the acquisition of the net assets of Apex should be reported. Costs of developing, maintaining, or restoring goodwill should be expensed when incurred
Under IFRS, which of the following statements about intangible assets is correct?
A
Internally generated goodwill cannot be recognized as an asset.
B
Intangible assets within a class may be measured differently using either the cost model or the revaluation model.
C
Research and development costs are capitalized as incurred.
D
Intangible assets with indefinite lives must be amortized annually
A
Explanation: Goodwill is recognized only in a business combination under both US GAAP and IFRS. With the exception of development costs, internally developed intangibles are not recognized as an asset under either US GAAP or IFRS. Under US GAAP, revaluation is not permitted. Under IFRS, revaluation to fair value of intangible assets other than goodwill is a permitted accounting policy election for a class of intangible assets. Under IFRS, development costs are capitalized when technical and economic feasibility of a project can be demonstrated in accordance with specific criteria. Amortization of intangible assets over their estimated useful lives is generally required under both US GAAP and IFRS. If there is no foreseeable limit to the period over which an intangible asset is expected to generate net cash inflows, the useful life is considered indefinite and the asset is not amortized.
In year 3, a company incurred $500,000 of legal costs defending several patents. Included in that amount was $400,000 of legal costs associated with successful outcomes and $100,000 of legal costs associated with unsuccessful outcomes. What amount of legal costs, if any, should the company expense for year 3?
A $500,000 B $400,000 C $100,000 D $0
c
Explanation:
The costs incurred in successful defense of patents are capitalized. Costs on failure to defend patent are expensed as no legal benefit would exist. Thus, legal costs associated with unsuccessful outcomes is expensed at $100,000.Option (a) is incorrect because it includes legal costs associated with successful defense of the patent which should be capitalized. ($500,000 = $400,000 + $100,000).Option (b) is incorrect because legal costs associated with successful defense of the patent are expensed, instead of capitalizing it. Option (d) is incorrect because the legal costs for unsuccessful out comes are expensed
Which of the following is an adverse qualitative factor related to goodwill? A Unanticipated competition B Loss of key personnel C Adverse action or assessment by a regulator D All of the above
d
Explanation:
All of the factors listed are considered adverse qualitative factors related to testing goodwill impairment.
A company is completing its annual impairment analysis of the goodwill included in one of its cash generating units (CGUs). The recoverable amount of the CGU is $32,000. The company noted the following related to the CGU:
Goodwill Patents Other assets Total
Historical Cost $15,000 $10,000 $35,000 $60,000
Depreciation and amortization ______0 __3,333 __11,667 __15,000
Carrying amount, December 31 $15,000 $ 6,667 $23,333 $45,000
Under IFRS, which of the following adjustments should be recognized in the company’s consolidated financial statements?
A Decrease goodwill by $13,000 B Decrease goodwill by $15,000 C Decrease goodwill by $3,250; patents by $2,167; and other assets by $7,583 D Decrease goodwill by $4,333; patents by $1,926; and other assets by $6,741
a
Explanation:
Under IFRS, a one-step approach requires that an impairment test be done at the cash-generating unit (CGU) level by comparing the CGU’s carrying amount, including goodwill, with its recoverable amount. Any impairment loss on the CGU (amount by which the CGU’s carrying amount, including goodwill, exceeds its recoverable amount) is allocated first to reduce goodwill to zero, then, subject to certain limitations, the carrying amount of other assets in the CGU are reduced pro rata, based on the carrying amount of each asset. In this case, the entire impairment loss of $13,000 ($45,000 – $32,000) would be allocated to goodwill (leaving a balance of $2,000).
Johan Co. has an intangible asset, which it estimates will have a useful life of 10 years, while Abco Co. has goodwill, which has an indefinite life. Which company should report amortization in its financial statements?
Johan Abco A Yes Yes B Yes No C No Yes D No No
B
Explanation:
For identifiable intangibles with definite life straight line method (SLM) is used to amortize the intangible assets over the lower of useful life or legal life. The intangibles with indefinite life cannot be amortized and must be tested for impairment. Therefore only Johan Co. will report amortization in its financial statements.Option (a), (c) and (d) are incorrect because Abco Co., should test goodwill for impairment since it is an Intangible asset with indefinite life.
Under IFRS, an entity that acquires an intangible asset may use the revaluation model for subsequent measurement only if
A
The useful life of the intangible asset can be readily determined.
B
An active market exists for the intangible asset.
C
The cost of the intangible asset can be measured reliably.
D
The intangible asset is a monetary asset.
B
Explanation: Under IFRS, revaluation to fair value of intangible assets other than goodwill is a permitted accounting policy election for a class of intangible assets. Revaluation to fair value requires an active market for the specific type of intangible. Though a reliable useful life and cost are used for amortization, they are not elements required for use of the revaluation model. By definition, intangible assets are nonmonetary assets.
A company reported $6 million of goodwill in last year’s statement of financial position. How should the company account for the reported goodwill in the current year?
A
Determine the current year’s amortizable amount and report the current-year’s amortization expense.
B
Determine whether the fair value of the reporting unit is greater than the carrying amount and report a gain on goodwill in the income statement.
C
Determine whether the fair value of the reporting unit is less than the carrying amount and report an impairment loss on goodwill in the income statement.
D
Determine whether the fair value of the reporting unit is greater than the carrying amount and report the recovery of any previous impairment in the income statement.
c
Explanation:
The intangibles with indefinite life cannot be amortized and must be tested for impairment. Goodwill is an unidentifiable intangible asset that represents capitalized excess earning power. Goodwill is not amortized but tested for impairment annually.If the carrying value of reporting unit is greater than the fair value, an impairment loss for the excess of carrying value of goodwill is reported in the I/S.Option (a) is incorrect because goodwill is not amortized.Option (b) is incorrect because gain on goodwill is not reported.Option (d) is incorrect because restoration of previously recognized impairment loss is not allowed, unless the asset is held for disposal.
Northstar Co. acquired a registered trademark for $600,000. The trademark has a remaining legal life of five years, but can be renewed every 10 years for a nominal fee. Northstar expects to renew the trademark indefinitely. What amount of amortization expense should Northstar record for the trademark in the current year?
A $0 B $15,000 C $40,000 D $120,000
a
Explanation:
A trademark is an identifiable tangible asset.With an externally acquired trademark, normally the acquisition costs are capitalized and amortized over the useful life of the trademark.If an intangible asset has an indefinite life, as is the case with the trademark in this question, it is not amortized but rather tested at least annually for impairment until its useful life is determined to be no longer indefinite.If the trademark had been internally developed the costs would have been expensed as incurred. Option (b) is incorrect because trademark is amortized over 40 years. ($15,000 =$600,000/40). Option (c) is incorrect because trademark is amortized over 15 years. ($40,000 =$600,000/15). Option (d) is incorrect because trademark is amortized over 5 years. ($120,000 =$600,000/5)
West Co. paid $50,000 for an intangible asset other than goodwill. The fair value of the asset is $55,000. West signed a contract to sell the asset for $10,000 in 10 years. What amount of amortization expense should West record each year?
A $4,000 B $4,500 C $5,000 D $5,500
Explanation:
The correct answer is (A).
Amount to be capitalized is equal to the fair value of the consideration given or the fair value of the asset acquired, whichever is more clearly evident. When both are given, the fair value of the consideration given is used i.e. $50,000.
Amortization for an identifiable intangible with a definite life is done using the straight-line method. Amortization per year = (Capitalized amount less value of the asset at the end of its useful life) / Lower of useful life or legal life of the asset.
The amount of an intangible asset to be amortized is the amount initially assigned to that asset ($50,000) less any residual value ($10,000). The amount to be amortized is $40,000 ($50,000 - $10,000) over 10 years, or $4,000 a year.
Note: The residual value is the estimated fair value of the intangible asset at the end of its useful life to the reporting entity less any disposal costs.
An intangible asset with a finite useful life is amortized. Which of the following factors is not considered in the estimate of the finite useful life?
A
The expected use of the asset by the entity
B
Any legal, regulatory, or contractual provisions that may limit the useful life
C
Goodwill
D
The effects of competition
c
Explanation:
Goodwill is not a factor to be considered in the estimate of a finite useful life. Goodwill has an indefinite, not finite, useful life and is not amortized, but rather is tested at least annually for impairment.
OnJune30,Union Inc. purchased goodwill of $125,000 when it acquired the net assets of Apex Corp.During the year, Union incurred additional costs of developing goodwill, by training Apex employees ($50,000) and hiring additional Apex employees($25,000).Union’sDecember31balance sheet should report goodwill of
A $200,000 B $175,000 C $150,000 D $125,000
d
Explanation:
Costs of goodwill from a business combination accounted for as a purchase should be capitalized.However, costs of developing, maintaining, or restoring goodwill should be expensed when incurred.Thus, the good will of $125,000 from the acquisition of the net assets of Apex should be capitalized,while the additional costs of developing good will should be expensed as incurred. Option (a) is incorrect because it includes both additional costs of developing goodwill, by training Apex employees ($50,000) and hiring additional Apex employees($25,000) which should be expensed off. Option (b) is incorrect because it includes cost of training Apex employees ($50,000) which should be expensed off. Option (c) is incorrect because it includes cost of hiring additional Apex employees($25,000) which should be expensed off.
On December 31, 20X0, Bit Co. had capitalized costs for a new computer software product with an economic life of five years. Sales for 20X1 were 30 percent of expected total sales of the software. At December 31, 20X1, the software had a net realizable value equal to 90 percent of the capitalized cost. What percentage of the original capitalized cost should be reported as the net amount on Bit’s December 31, 20X1,balance sheet?
A 70% B 72% C 80% D 90%
a
Explanation:
The capitalized costs of research and development (R&D) is amortized for the greater of (a) cost x ratio of current year revenue of software to the total revenue expected or (b) straight-line method which is cost / economic life of product. The ratio of current year revenue of software to the total revenue expected is given as 30% and the straight-line rate will be 20% (1 / 5 years). Thus, the amortization would be at the greater of the two percentages = 30%. The percentage of original capitalized cost to be reported in the balance sheet would therefore be 70% (i.e.100% - 30%). Capitalized software cost should be reported at the lower of carrying value (70% of original cost) or net realizable value (90% of original cost). Thus, the software cost would be reported at 70%. Options (b), (c) and (d) are incorrect because of above explanation.
A company began developing computer software to be sold as a separate product on January 1, year 1. During the planning, coding, and testing phases, the company incurred $1,300,000 of costs. On June 30, year 1, the product was determined to be technologically feasible. The company began producing product masters of the software and incurred an additional $750,000 of costs from July 1, year 1, through September 30, year 1. After the software was available for release on October 1, year 1, the company incurred an additional $275,000 of costs relating to maintenance and customer support. What amount of software-related costs should be capitalized?
A $275,000 B $750,000 C $1,300,000 D $2,050,000
Explanation:
The correct answer is (B).
Costs associated with computer software may be capitalized once technological feasibility has been established. The costs will continue to be capitalized until the date the software is released for sale. The software was determined to be technologically feasible on June 30th and was released for sale on October 1st. Between July 1st and September 30th, costs of $750,000 are capitalized. All costs incurred prior to June 30th and after October 1st are expensed. Examples of expensed items include sales revenue and R&D.
(A) is incorrect because $275,000 costs incurred after software sales begin are inventories and included in COGS.
(C) is incorrect because $1,300,000 should be expensed as these costs incurred prior to technical feasibility was established.
(D) is incorrect because it includes expenses incurred prior to the technical feasibility was established and the costs incurred after the product was available for release ($2,050,000 = $1,300,000 + $750,000).
Tech Co. bought a trademark on January 2, two years ago. Tech accounted for the copyright as instructed under the appropriate GAAP guidance during the current year. The intangible was being amortized over 40 years. The carrying value at the beginning of the year was $38,000. It was determined that the cash flow will be generated indefinitely at the current level for the trademark. What amount should Tech report as amortization expense for the current year? A $0 B $922 C $1,000 D $38,000
a
Explanation:
An intangible asset with an indefinite useful life is not amortized. If no legal, regulatory, contractual, competitive, economic, or other factors limit the useful life, it is considered to be indefinite. The term indefinite does not mean infinite. Goodwill and intangible assets with indefinite useful lives are not amortized, but rather are tested at least annually for impairment.
Which of the following is a pair of values that are compared to determine the amount of a possible impairment loss on an intangible asset, with an indefinite life, other than goodwill?
A Fair value, present value B Carrying value, book value C Future value, carrying value D Fair value, carrying value
Explanation:
The correct answer is (D).
Indefinite life intangibles should be tested for impairment annually or more frequently if needed. If the circumstances indicating impairment exist, then the indefinite life intangible is tested for impairment by comparing its carrying value with its fair value. If the carrying value of the indefinite life intangible exceeds its fair value then it is impaired and the difference between the two values is recorded as an impairment loss and the value of the asset is reduced by that amount. Thus, the carrying value and the fair value of the indefinite life intangible are compared to determine the amount of possible impairment loss
On January 2, Gant Co. purchased a franchise with a useful life of five years for $60,000 and an annual fee of 1% of franchise revenues. Franchise revenues were $20,000 during the year. Gant projects future revenues of $40,000 next year and $60,000 per year for the following three years. Gant uses the straightline method of amortization. What amount should Gant report as intangible asset-franchise, net of related amortization in its December 31 balance sheet? A $48,000 B $48,160 C $49,920 D $56,000
a
Explanation:
A franchise represents a special right to operate under the name and guidance of another enterprise over a limited geographic area. A franchise is always externally purchased; it cannot be internally developed. Capitalize all significant costs incurred to acquire the franchise (e.g., purchase price, legal fees, etc.). If the acquisition cost of the franchise requires future cash payments, these payments should be capitalized at their present value using an appropriate interest rate. On the other hand, periodic service fees charged as a percentage of revenues are not capitalized; these costs represent a current operating expense of the franchisee. The $60,000 frinchise purchase divided by the five years of useful life would mean the intangible asset-franchise would be amortized $12,000 per year. $60,000 - $12,000 = $48,000.
Anchor Co. is experiencing financial difficulties. Anchor negotiated a settlement of $100,000 in debt owed to Bowden, Inc. in exchange for Anchor’s gross receivables of $100,000. The receivables have an allowance for uncollectible accounts of $25,000. The impact of this transaction on Anchor’s net income is a $25,000
A Increase in bad debt expense. B Gain on restructuring of payables. C Loss on restructuring of payables. D Decrease in bad debt expense
Explanation:
The correct answer is (B)
Anchor Co. is experiencing financial difficulties. Anchor negotiated a settlement of $100,000 in debt owed to Bowden Inc. in exchange for Anchor’s gross accounts receivables of $100,000. This is a troubled debt restructuring, and the debt is settled by a transfer of assets (A/R). The debtor recognizes a gain equal to the excess of the carrying amount of the debt over the carrying amount of the assets. The carrying amount of the receivables transferred is $75,000 because $25,000 is deemed uncollectible. Anchor will recognize a $25,000 gain on restructuring of payables.
On January 1, Stunt Corp. had outstanding convertible bonds with a face value of $1,000,000 and an unamortized discount of $100,000. On that date, the bonds were converted into 100,000 shares of $1 par stock. The market value on the date of conversion was $12 per share. The transaction will be accounted for with the book value method. By what amount will Stunt's stockholders' equity increase as a result of the bond conversion? A $ 100,000 B $ 900,000 C $1,000,000 D $1,200,000
b
Explanation:
Convertible bonds provide the bondholder the option of converting the bond to capital stock, typically common stock. Using the book value method, the conversion of the bonds into common stock is generally recorded by crediting the paid-in capital accounts for the carrying amount of the debt at the date of the conversion, less any cost associated with the conversion. The carrying amount of the bonds on the date of conversion is the $1,000,000 face value less the $100,000 unamortized discount. The market value is not considered when using the book value method. The journal entry:
Bonds Payable 1,000,000
Bond Discount 100,000
Common Stock (100,000 × $1 par) 100,000
APIC (to balance) 800,000The $100,000 credit to common stock and $800,000 credit to APIC would increase stockholders’ equity $900,000.
On December 31, Roth Co. issued a $10,000 face value note payable to Wake Co. in exchange for services rendered to Roth. The note, made at usual trade terms, is due in nine months and bears interest, payable at maturity, at the annual rate of 3%. The market interest rate is 8%. The compound interest factor of $1 due in nine months at 8% is .944. At what amount should the note payable be reported in Roth's December 31 balance sheet? A $10,300 B $10,000 C $ 9,652 D $ 9,440
b
Explanation:
The note payable arose from a transaction with a vendor in the normal course of business and is due in customary trade terms not exceeding one year; therefore, the note can be reported at its face amount of $10,000, despite the fact that the 3% stated interest rate of the note does not approximate the prevailing market interest rate of 8% for similar notes at the date of the transaction.